7 - Topnotch Pharmacology Superexam

  • Uploaded by: Vien Cervantes
  • 0
  • 0
  • January 2021
  • PDF

This document was uploaded by user and they confirmed that they have the permission to share it. If you are author or own the copyright of this book, please report to us by using this DMCA report form. Report DMCA


Overview

Download & View 7 - Topnotch Pharmacology Superexam as PDF for free.

More details

  • Words: 64,038
  • Pages: 83
Loading documents preview...
TOPNOTCH MEDICAL BOARD PREP PHARMACOLOGY SUPEREXAM For inquiries visit www.topnotchboardprep.com.ph or email us at [email protected] DEAR TOPNOTCH FRIENDS: PLEASE FOLLOW THESE INSTRUCTIONS: 1. These questions are previous diagnostic, midterm, and finals exams of Topnotch, almost all of them made by Topnotch Board Exam Topnotchers. 2. Answer this Topnotch Superexam seriously 100-items at a time. Cover the “Explanations” Column. Do not immediately look at the answers from the answer key. That’s not the correct way of answering sample exams. You need to treat these MCQs as exercises and not as handouts. 3. Time yourself. 1.5 hours per 100-item block. 4. After answering each 100-item block, refer to the Topnotch Answer Key for the correct answers. Please be careful of “frameshift mutations” when checking your answers – check every 10 items. (the format of the answer key was designed for you to practice against “frameshift mutations”) 5. The Topnotch Superexams are EXERCISES for the actual med boards. They will not appear verbatim in your future exams. More than knowing what’s the correct answer, it’s more important for you to: a. Know why the other choices are wrong b. Know why the other choices were included in the first place c. Know the explanation to the correct answer 6. Sharpen your mind by answering the Topnotch Superexams. Most of these questions based on past feedback are more difficult than the actual questions in the med boards. In these exams made by Board Exam Topnotchers, if you’re getting a score of 60/100 , that’s already a good score. More than 80/100 is outstanding. Item QUESTION EXPLANATION AUTHOR TOPNOTCH # EXAM 1 A 6-year-old patient was recently diagnosed with Cat Know the doses of anti-TB drugs for BOTH children KRISTEL TANHUI DIAGNOSTIC I PTB. DOTS Treatment is initiated for this patient. and adults. (TOP 3 - AUG EXAM - The dosing should be: Adults: INH 5mg/kg + Rif 10mg/kg + Pyz 25mg/kg + 2015 MED MARCH 2016 A. INH 5mg/kg + Rif 10mg/kg + Pyz 25mg/kg + Eth Eth 15mg/kg BOARDS; 15mg/kg TOPNOTCH MD B. INH 10mg/kg + Rif 15mg/kg + Pyz 35mg/kg + Eth Generally just make sure you are familiar with TB! It FROM LA SALLE) 20mg/kg can come out over and over in different subjects C. INH 15mg/kg + Rif 10mg/kg + Pyz 35mg/kg + Eth from pharma, micro, pedia, im, patho, even gyne. 15mg/kg D. INH 25mg/kg + Rif 10mg/kg + Pyz 25mg/kg + Eth Source: National TB Control Program Manual of 11mg/kg Procedures E. INH 5mg/kg + Rif 20mg/kg + Pyz 15mg/kg + Eth 10mg/kg 2 Which of the following anti-TB drug is matched Anti-TB drug most associated with skin rashes is KRISTEL TANHUI DIAGNOSTIC incorrectly with its adverse effects? streptomycin. (TOP 3 - AUG EXAM - A. Most hepatotoxic – Pyrazinamide 2015 MED MARCH 2016 B. Psychosis – Isoniazid Source: National TB Control Program Manual of BOARDS; C. Most ototoxic – Streptomycin Procedures TOPNOTCH MD D. Most associated with skin rashes due to FROM LA SALLE) hypersensitivity – Rifampicin E. Most associated with visual impairment - Ethambutol 3 A 49 year old made came to the ER with a 3 hour Other than Aspirin, NSAIDs are absolutely KRISTEL TANHUI DIAGNOSTIC history of anginal chest pain. Past medical history is contraindicated to patients with acute MI. (TOP 3 - AUG EXAM - significant only for uncontrolled hypertension. 2015 MED MARCH 2016 Cardiac markers and ECG both confirm the diagnosis Drugs which are administered post MI include betaBOARDS; of acute myocardial infarction probably of the blockers (unless with contraindications), ACE TOPNOTCH MD anterior wall. Currently the patient has stable vital inhibitors (beneficial in preventing cardiac FROM LA SALLE) signs within normal range with persistence of angina. remodeling), Statins (stabilizes the atheromatous The following should be given except: plaque), antiplatelet therapy like aspirin or A. Lisinopril clopidogrel, aldosterone antagonist (epleronone B. Simvastatin only), nitrates (increase O2 supply). C. Ibuprofen D. Metoprolol E. None of the above. All should be administered. 4 Which of the following antibiotic agents is Generally, cell wall inhibitors are –cidal, drugs which KRISTEL TANHUI DIAGNOSTIC bacteriostatic? interfere with DNA are –cidal except sulfonamides (TOP 3 - AUG EXAM - A. Piperacillin which are only static unless they are given with 2015 MED MARCH 2016 B. Vancomycin trimethoprim/pyrimethamine (in this case BOARDS; C. Gentamycin synergism results to cidal activity) TOPNOTCH MD D. Doxycycline FROM LA SALLE) E. Co-trimoxazole Protein synthesis inhibitors are –static except aminoglycosides because they cause frameshift mutations by misreading of the genetic code. Source: Topnotch handout on Pharmacology 5

Which of the following is not an indication for SSRIs? A. Major depressive disorder B. OCD C. Anxiety and panic attacks D. Phobias E. None of the above

SIMILAR TO PREVIOUS BOARD EXAM CONCEPT/PRINCIPLE. SSRIs are the first line drugs for MDD. OCD – SSRI! Source: Topnotch handout on Pharmacology

KRISTEL TANHUI (TOP 3 - AUG 2015 MED BOARDS; TOPNOTCH MD FROM LA SALLE)

DIAGNOSTIC EXAM - MARCH 2016

6

Propylthiouracil and Methimazole are both used in the treatment of hyperthyroidism. They are thioamides which inhibits the enzyme thyroid peroxidase necessary for the synthesis of thyroid hormone. Which of the following is false about propylthiouracil? A. PTU is more highly protein bound than methimazole B. PTU has a longer halflife and requires less frequent dosing than methimazole C. PTU is less potent than methimazole. D. PTU has the added activity of preventing

The halflife of PTU is 1.5hrs and methimazole is 68hrs. PTU is generally given every 6-8hrs while methimazole is given once a day. Thus methimazole is more convenient for patients. Methimazole is also 10 times more potent than PTU. Source: Katzung Basic and Clinical Pharmacology 11th ed p671

KRISTEL TANHUI (TOP 3 - AUG 2015 MED BOARDS; TOPNOTCH MD FROM LA SALLE)

DIAGNOSTIC EXAM - MARCH 2016

TOPNOTCH MEDICAL BOARD PREP PHARMACOLOGY SUPEREXAM Page 1 of 83 For inquiries visit www.topnotchboardprep.com.ph or email us at [email protected]

TOPNOTCH MEDICAL BOARD PREP PHARMACOLOGY SUPEREXAM For inquiries visit www.topnotchboardprep.com.ph or email us at [email protected] Item #

QUESTION

EXPLANATION

AUTHOR

TOPNOTCH EXAM

peripheral conversion of T4 into T3. E. None of the above

7

Which of the following has the most potent mineralocorticosteroid activity? A. Hydrocortisone B. Triamcinolone C. Betamethasone D. Fludrocortisone E. Dexamethasone

Contraindication to thrombolysis - History of CVS hemorrhage at ANY time - History of CVS infarct within the past year - Marked hypertension (>180/110) - Suspicion of aortic dissection - Active internal bleeding (excluding menstruation) Source: Topnotch handout on Pharmacology

KRISTEL TANHUI (TOP 3 - AUG 2015 MED BOARDS; TOPNOTCH MD FROM LA SALLE)

DIAGNOSTIC EXAM - MARCH 2016

8

Which of the following is not true regarding Noncompetitive antagonism? A. No effect on Emax B. No effect on ED50 C. No effect on potency D. A and B E. All of the above

Competitive antagonism: ED50 increases, potency decreases, No effect of maximum efficacy Noncompetitive antagonism: No effect ED50, No effect on potency, decreased Emax Source: Topnotch handout on Pharmacology

KRISTEL TANHUI (TOP 3 - AUG 2015 MED BOARDS; TOPNOTCH MD FROM LA SALLE)

DIAGNOSTIC EXAM - MARCH 2016

9

Which of the following is incorrectly matched? A. Menotropin – FSH and LH analog B. Ocreotide – Somatostatin analog C. Desmopressin – ADH analog D. Ganirelix – GnRH analog E. Levothyroxine – T4

Menotropin – FSH and LH analog Ocreotide – Somatostatin analog Desmopressin – ADH analog Ganirelix – GnRH antagonist Levothyroxine – T4 Source: Topnotch handout on Pharmacology

KRISTEL TANHUI (TOP 3 - AUG 2015 MED BOARDS; TOPNOTCH MD FROM LA SALLE)

DIAGNOSTIC EXAM - MARCH 2016

10

A 45 year old male with history of polyuria, polyphagia and weight loss was recently started on antidiabetic medications after workup confirmed DM type II. 2 months later, he notes increasing weight. Which of the following antidiabetic medications is associated with this? A. Chlorpropramide B. Glimepiride C. Exenetide D. A and B E. All of the above

The first and second generation sulfonylureas are associated with weight gain. Source: Katzung Basic and Clinical Pharmacology 11th ed p748

KRISTEL TANHUI (TOP 3 - AUG 2015 MED BOARDS; TOPNOTCH MD FROM LA SALLE)

DIAGNOSTIC EXAM - MARCH 2016

11

Which of the following is not associated with pulmonary fibrosis? A. Bleomycin B. Busulfan C. Amiodarone D. Methotrexate E. None of the above

All of the following are associated with pulmonary fibrosis. Source: Topnotch handout on Pharmacology

KRISTEL TANHUI (TOP 3 - AUG 2015 MED BOARDS; TOPNOTCH MD FROM LA SALLE)

DIAGNOSTIC EXAM - MARCH 2016

12

What is the treatment for malignant hyperthermia? A. Dantrolene B. Nitroglycerin C. Paracetamol D. Halothane E. All of the above

Source: Topnotch handout on Pharmacology

KRISTEL TANHUI (TOP 3 - AUG 2015 MED BOARDS; TOPNOTCH MD FROM LA SALLE)

DIAGNOSTIC EXAM - MARCH 2016

13

Which of the following is not a 3rd generation cephalosporin? A. Ceftazidime B. Ceftriaxone C. Ceftizoxime D. Cefixime E. Cefepime

1st gen – Cefazolin, Cephalotin, Cephalexin, Cephradine, Cephapirin 2nd gen – Cefaclor, Cefuroxime, Cefoxitin, Cefotetan, Cefamandole, Cefonicid, Cefprozil, Cefometazole 3rd gen – Ceftriaxone, Cefoperazone, Cefotaxime, Cefixime, Ceftazidime, Ceftizoxime, Cefpodoxime proxetil, Cefdinir, Cefditoren, Ceftibuten 4th gen – Cefepime Source: Katzung Basic and Clinical Pharmacology 11th ed

KRISTEL TANHUI (TOP 3 - AUG 2015 MED BOARDS; TOPNOTCH MD FROM LA SALLE)

DIAGNOSTIC EXAM - MARCH 2016

TOPNOTCH MEDICAL BOARD PREP PHARMACOLOGY SUPEREXAM Page 2 of 83 For inquiries visit www.topnotchboardprep.com.ph or email us at [email protected]

TOPNOTCH MEDICAL BOARD PREP PHARMACOLOGY SUPEREXAM For inquiries visit www.topnotchboardprep.com.ph or email us at [email protected] Item # 14

QUESTION Which of the following vasodilators can worsen Diabetes Mellitus? A. Hydralazine B. Diazoxide C. Verapamil D. Nifedipine E. Nitroprusside

EXPLANATION

AUTHOR

TOPNOTCH EXAM DIAGNOSTIC EXAM - MARCH 2016

The mechanism of action of diazoxide is by opening K channels causing hyperpolarization and relaxation of vascular smooth muscles. However, recall the mechanism of release of insulin in the pancreatic beta cells. Glucose enters cells resulting to increased ATP production. This closes ATP dependent K channels resulting to depolarization, entry of Calcium and release of insulin. Diazoxide causes opening of K channels resulting to hyperpolarization and inhibition of insulin release. Diazoxide is actually used in treatment of insulinomas. Source: Katzung Basic and Clinical Pharmacology 11th ed p 180 Statins inhibit the rate-limiting step of cholesterol synthesis. Don’t get confused with HMG CoA synthase which is the rate limiting step of ketogenesis. Source: Katzung Basic and Clinical Pharmacology 11th ed p 612

KRISTEL TANHUI (TOP 3 - AUG 2015 MED BOARDS; TOPNOTCH MD FROM LA SALLE)

KRISTEL TANHUI (TOP 3 - AUG 2015 MED BOARDS; TOPNOTCH MD FROM LA SALLE)

DIAGNOSTIC EXAM - MARCH 2016

This is a case of acute renal failure secondary to statin induced rhabdomyolysis. Statins inhibit cholesterol synthesis. One of the intermediates of cholesterol synthesis is farnesyl. Farnesyl is needed for the synthesis of Coenzyme Q, which is important for ATP production in metabolically active cells like the muscles. Depletion of this can result to rhabdomyolysis. To prevent this, one can administer Coenzyme Q10 with statins. Source: Katzung Basic and Clinical Pharmacology 11th ed p 1122 This is a case of acute renal failure secondary to statin induced rhabdomyolysis. Statins inhibit cholesterol synthesis. One of the intermediates of cholesterol synthesis is farnesyl. Farnesyl is needed for the synthesis of Coenzyme Q, which is important for ATP production in metabolically active cells like the muscles. Depletion of this can result to rhabdomyolysis. To prevent this, one can administer Coenzyme Q10 with statins. Source: Katzung Basic and Clinical Pharmacology 11th ed p 1122 SIMILAR TO PREVIOUS BOARD EXAM CONCEPT/PRINCIPLE. Source: Topnotch Handout on Pharmacology

KRISTEL TANHUI (TOP 3 - AUG 2015 MED BOARDS; TOPNOTCH MD FROM LA SALLE)

DIAGNOSTIC EXAM - MARCH 2016

KRISTEL TANHUI (TOP 3 - AUG 2015 MED BOARDS; TOPNOTCH MD FROM LA SALLE)

DIAGNOSTIC EXAM - MARCH 2016

KRISTEL TANHUI (TOP 3 - AUG 2015 MED BOARDS; TOPNOTCH MD FROM LA SALLE)

DIAGNOSTIC EXAM - MARCH 2016

15

What enzyme do statins inhibit? A. HMG-CoA synthase B. HMG-CoA reductase C. HMG-CoA lyase D. HMG-CoA transferase A. All of the above

16

A 67-year-old male with ischemic heart disease, compliant with his medications is admitted for acute renal failure. Labs reveal hyperkalemia, a rapidly elevating creatinine, and markely increased CK and myoglobin levels. Which of the following medications he was taking could have precipitated the condition? A. Aspirin B. Carvedilol C. Losartan D. Captopril E. Rosuvastatin

17

A 67-year-old male with ischemic heart disease, compliant with his medications is admitted for acute renal failure. Labs reveal hyperkalemia, a rapidly elevating creatinine, and markely increased CK and myoglobin levels. Which of the following medications he was taking could have precipitated the condition? A. Aspirin B. Carvedilol C. Losartan D. Captopril E. Rosuvastatin

18

Which of the following drugs are beneficial in delaying organ remodeling such as in cardiac remodeling in heart failure and reduces progression of DM nephropathy? A. Captopril B. Losartan C. Eplerenone D. A and B only E. All of the above

19

For the treatment of brain tumors, it is important for a drug to be able to penetrate the blood brain barrier. Which of the following antineoplastic agents cannot cross the blood brain barrier? A. Procarbazine B. Dacarbazine C. Streptozocin D. Carmustine E. 5-FU

Procarbazine, Dacarbazine, Nitrosureas (Lomustine, Carmustine, Streptozocin) are chemotherapeutic agents which are able to penetrate the blood brain barrier. Source: Topnotch Handout on Pharmacology

KRISTEL TANHUI (TOP 3 - AUG 2015 MED BOARDS; TOPNOTCH MD FROM LA SALLE)

DIAGNOSTIC EXAM - MARCH 2016

20

Which of the following is not considered a controller drug for asthma? A. Budesonide B. Terbutaline C. Montelukast D. Nedocromil E. Omalizumab

Terbutaline is a bronchodilator used for acute relief of asthma exacerbations. Source: Topnotch handout on Pharmacology

KRISTEL TANHUI (TOP 3 - AUG 2015 MED BOARDS; TOPNOTCH MD FROM LA SALLE)

DIAGNOSTIC EXAM - MARCH 2016

TOPNOTCH MEDICAL BOARD PREP PHARMACOLOGY SUPEREXAM Page 3 of 83 For inquiries visit www.topnotchboardprep.com.ph or email us at [email protected]

TOPNOTCH MEDICAL BOARD PREP PHARMACOLOGY SUPEREXAM For inquiries visit www.topnotchboardprep.com.ph or email us at [email protected] Item # 21

22

23

QUESTION

EXPLANATION

AUTHOR

A 45-year-old man with a history of medicationHydrochlorothiazide, a thiazide diuretic, can LESTER BRYAN controlled hypertension presents to your office with precipitate a gouty attack in predisposed CO (TOP 10 - AUG complaints of a painful, swollen big toe on the left individuals. This is because these agents increase 2015 MED foot. You suspect gout and check his uric acid levels, serum uric acid as a result of competition for the BOARDS; which are elevated. From looking at the list of the organic acid carrier. Loop diuretics can have this TOPNOTCH MD medications the patient is taking, you realize that one effect too. Acetazolamide is a carbonic anhydrase FROM UST) of the medications may be the cause of his current inhibitor; this agent does not have a significant symptoms. Which medication might that be? impact on the levels of uric acid. Amiloride and A. Acetazolamide spironolactone are potassium-sparing diuretics, and B. Amiloride they do not have a significant impact on the levels of C. Spironolactone uric acid either. The same is true for mannitol, an D. Hydrochlorothiazide osmotic diuretic. E. Mannitol A 24-year-old student has been taking over-theDiphenhydramine blocks H1-receptors in the brain, LESTER BRYAN counter diphenhydramine for her allergy symptoms thereby producing sedation. The release of CO (TOP 10 - AUG most of her life. Lately, however, she has had more dopamine and serotonin is modulated via H32015 MED frequent symptoms, so he increased the dose of the receptors. Diphenhydramine readily crosses the BOARDS; medication. She now asks her friend, who is a medical blood–brain barrier. This agent has TOPNOTCH MD student, to explain to her how exactly this agent muscariniccholinergic agonist properties. It is not FROM UST) makes her more sleepy lately. What is the most likely known to contain tryptophan. answer regarding diphenhydramine? A. It blocks H1-receptors in the brain B. It modulates the release of dopamine and serotonin C. It acts peripherally, since it does not cross the blood–brain barrier D. It exerts its effects via muscarinic-cholinergic agonist activity E. It contains tryptophan, which produces sedation Somatostatin interacts with a Somatostatin (growth-inhibiting factor) binds to GiLESTER BRYAN A. Gq-protein–coupled receptor coupled protein receptor, initiating exchange of GTP CO (TOP 10 - AUG B. Gi-protein–coupled receptor for GDP, which inhibits AC and leads to reduced 2015 MED C. Ligand-activated ion channel cAMP production. BOARDS; D. Receptor-activated tyrosine kinase TOPNOTCH MD E. Intracellular nuclear receptor FROM UST)

TOPNOTCH EXAM MIDTERM 1 EXAM - MARCH 2016

MIDTERM 1 EXAM - MARCH 2016

MIDTERM 1 EXAM - MARCH 2016

24

A 50-year man with mild hypertension complains of discomfort in his chest. He has slightly enlarged fat deposits in his breasts with prominent nipples. Which of the following medications might be causing this adverse effect? A. Amiloride B. Spironolactone C. Metolazone D. Hydrochlorothiazide E. Acetazolamide

SIMILAR TO PREVIOUS BOARD EXAM CONCEPT/PRINCIPLE. Spironolactone antagonizes the action of the mineralocorticoid, progesterone, and androgen receptors. Inhibition of androgen receptors can lead to gynecomastia and breast tenderness, most often in men.

LESTER BRYAN CO (TOP 10 - AUG 2015 MED BOARDS; TOPNOTCH MD FROM UST)

MIDTERM 1 EXAM - MARCH 2016

25

Zileuton is useful in the treatment of asthma because By inhibiting 5-lipoxygenase, zileuton reduces it leukotriene biosynthesis; it does not inhibit (and in A. Inhibits prostaglandin biosynthesis fact it might increase) prostaglandin synthesis. B. Inhibits leukotriene synthesis C. Inhibits leukotriene receptors D. Inhibits 12-lipoxygenase E. None of the above.

LESTER BRYAN CO (TOP 10 - AUG 2015 MED BOARDS; TOPNOTCH MD FROM UST)

MIDTERM 1 EXAM - MARCH 2016

26

A 59-year-old female nurse who has been diagnosed Any of the sulfonylureas can cause hypoglycemia with type 2 diabetes is admitted to the emergency which can produce shock-like symptoms. Metformin room. She is tachycardic, tachypneic, and appears and the α-glycosidase inhibitors such as acarbose very disoriented; she does not remember the day of rarely cause ypoglycemia. Glucagon would raise the week or her address or any emergency contact plasma glucose. numbers. She vaguely remembers taking her “sugar medicine” earlier in the day. Which of the following drugs is most likely responsible for her condition? A. Metformin B. Acarbose C. Glipizide D. Glucagon E. Pioglitazone A 53-year-old woman with breast cancer undergoes a Paclitaxel is often used in the treatment of breast as breast-conserving lumpectomy and lymph node well as ovarian and lung cancer. Its main toxicities biopsy. The pathology report returns with mention of are myelosuppression and peripheral neuropathy cancer cells in two of eight lymph nodes removed. that usually manifest as numbness and tingling in Following radiation therapy, chemotherapy is started the distal extremities. Blood in the urine can indicate that includes the use of paclitaxel. Which side effect is hemorrhagic cystitis, a complication of the patient likely to complain of? cyclophosphamide use. Easy bruising can result A. Blood in the urine from mechlorethamine use. Hot flashes are a B. Easy bruising common complaint in patients using tamoxifen. C. Hot flashes Shortness of breath can result from pulmonary D. Shortness of breath fibrosis secondary to busulfan or bleomycin use. E. Numbness and tingling

LESTER BRYAN CO (TOP 10 - AUG 2015 MED BOARDS; TOPNOTCH MD FROM UST)

MIDTERM 1 EXAM - MARCH 2016

LESTER BRYAN CO (TOP 10 - AUG 2015 MED BOARDS; TOPNOTCH MD FROM UST)

MIDTERM 1 EXAM - MARCH 2016

27

TOPNOTCH MEDICAL BOARD PREP PHARMACOLOGY SUPEREXAM Page 4 of 83 For inquiries visit www.topnotchboardprep.com.ph or email us at [email protected]

TOPNOTCH MEDICAL BOARD PREP PHARMACOLOGY SUPEREXAM For inquiries visit www.topnotchboardprep.com.ph or email us at [email protected] Item # 28

29

30

31

32

33

34

QUESTION

EXPLANATION

AUTHOR

A 42-year-old MDS patient presents to the emergency Toxoplasmosis is treated with a combination of LESTER BRYAN room with mental status changes and a headache. A pyrimethamine and sulfadiazine. Ivermectin is used CO (TOP 10 - AUG computed tomography scan is ordered and to treat filariasis, whereas praziquantel is used to 2015 MED demonstrates a ring-enhancing lesion. You decide to treat schistosomiasis. Niclosamide can be used to BOARDS; treat him empirically due to the possibility of treat tapeworm infections, and pyrantel pamoate is TOPNOTCH MD Toxoplasmosis gondii abscess. Which agent should be used to treat many helminth infections. FROM UST) included in his treatment? A. Ivermectin B. Praziquantel C. Sulfadiazine D. Niclosamide E. Pyrantel pamoate A 74-year-old man with a 100-pack/year history of Etoposide is used in the treatment of small-cell lung LESTER BRYAN smoking is evaluated for hemoptysis. A computed carcinomas as well as testicular tumors. Its CO (TOP 10 - AUG tomography (CT) scan of the chest shows numerous mechanism of action is related to its ability to inhibit 2015 MED pulmonary nodules. A nodule on the pleural surface is topoisomerase II. Methotrexate inhibits BOARDS; selected for CT-guided biopsy by the interventional dihydrofolate reductase. Alkylating agents include TOPNOTCH MD radiologist. The biopsy report is small-cell carcinoma mechlorethamine, cyclophosphamide, and FROM UST) of the lung, and chemotherapy containing etoposide ifosfamide. Paclitaxel and docetaxel stabilize is started. This drug works by microtubules and thereby disrupt mitosis. A. Inhibiting topoisomerase II Bleomycin causes DNA chain scission and B. Inhibiting dihydrofolate reductase fragmentation. C. Alkylating double-stranded DNA D. Stabilizing microtubules, with resultant mitotic arrest E. Causing DNA chain scission and fragmentation A young couple present to their primary care Pregnancy and lactation are the states of increased LESTER BRYAN physician stating that they are trying to conceive. iron demands. While increased bleeding tendency, CO (TOP 10 - AUG They would like to know if the future mom-to-be dietary deficiency, and malabsorption are all true 2015 MED needs to be on any supplements. Along with causes of iron deficiency anemia, they are not the BOARDS; recommending a multivitamin with folic acid, the culprits during pregnancy. Iron storage is regulated TOPNOTCH MD doctor also suggests an iron supplement. Pregnant at the level of absorption, and very little of it is lost FROM UST) women develop iron deficiency anemia because of from the body. A. Increased bleeding tendency B. Increased dietary deficiency C. Malabsorption D. Increased iron demands E. Increased excretion Ephedrine can cause increased blood pressure by Ephedrine acts indirectly to release norepinephrine LESTER BRYAN A. Indirect action on cholinergic receptors from nerve terminals, causing effects similar to CO (TOP 10 - AUG B. Blockade of adrenergic receptors those of catecholamines, including elevated blood 2015 MED C. Stimulation of release of epinephrine pressure. This potentially dangerous agent has been BOARDS; D. Inhibition of reuptake of catecholamines removed from the OTC market because of an TOPNOTCH MD E. Direct action on dopamine receptors increasing number of deaths being reported as FROM UST) caused by this agent. An example of an indirectacting cholinergic agonist is edrophonium, which is used for diagnosis of myasthenia gravis. Some adrenoceptor blockers, such as atenolol, are used for the treatment of hypertension. Catecholamine reuptake inhibition is a property of some antidepressant medications. Dopamine receptor agonists are used in the treatment of Parkinson disease. Which of the following is a potential side effect of Agranulocytosis occurs more frequently with LESTER BRYAN clozapine? clozapine than with other agents, requiring routine CO (TOP 10 - AUG A. Cholestatic jaundice blood tests. It is the only agent that improves the 2015 MED B. QT prolongation negative symptoms of schizophrenia. Cholestatic BOARDS; C. Agranulocytosis jaundice and photosensitivity are common with TOPNOTCH MD D. Photosensitivity chlorpromazine. Galactorrhea is a side effect of older FROM UST) E. Galactorrhea high-potency agents that block dopamine. QT prolongation is a complication of agents such as thioridazine and ziprasidone. Which of the following is a good choice to treat newly Buspirone is a partial serotonin 5-HT1A-receptor LESTER BRYAN diagnosed generalized anxiety disorder in a truck agonist that has efficacy comparable to that of CO (TOP 10 - AUG driver? benzodiazepines for the treatment of anxiety, but is 2015 MED A. Alprazolam significantly less sedating. Alprazolam is an BOARDS; B. Triazolam intermediate-acting benzodiazepine used in the TOPNOTCH MD C. Buspirone treatment of generalized anxiety disorder (GAD) but FROM UST) D. Trazodone still has some sedation, which would be undesirable E. Thiopental in this situation. Triazolam is a short-acting benzodiazepine, and trazodone is a heterocyclic antidepressant, both used to induce sleep. Thiopental is a barbiturate sometimes used to induce anesthesia. A 17-year-old girl sees her physician for swollen ABVD is a treatment regimen used for Hodgkin LESTER BRYAN lymph nodes in the supraclavicular region. A core disease and includes adriamycin, bleomycin, CO (TOP 10 - AUG biopsy demonstrates Reed–Sternberg cells and vinblastine, and dacarbazine. R-CHOP is used for 2015 MED fibrotic bands, a finding characteristic of nodular treating non-Hodgkin’s disease. CMF, or BOARDS; sclerosis Hodgkin disease. Which of the following cyclophosphamide, methotrexate, and fluorouracil, TOPNOTCH MD combined regimens might be used in this patient? is used for breast cancer. FOLFOX, a regimen that FROM UST) A. R-CHOP uses 5-fluorouracil, oxaliplatin, and leucovorin, is B. CMF used in the treatment of colon cancer. BEP C. FOLFOX (bleomycin, etoposide, and platinum [cisplatin]) is D. BEP used in the management of metastatic testicular E. ABVD neoplasms.

TOPNOTCH MEDICAL BOARD PREP PHARMACOLOGY SUPEREXAM Page 5 of 83 For inquiries visit www.topnotchboardprep.com.ph or email us at [email protected]

TOPNOTCH EXAM MIDTERM 1 EXAM - MARCH 2016

MIDTERM 1 EXAM - MARCH 2016

MIDTERM 1 EXAM - MARCH 2016

MIDTERM 1 EXAM - MARCH 2016

MIDTERM 1 EXAM - MARCH 2016

MIDTERM 1 EXAM - MARCH 2016

MIDTERM 1 EXAM - MARCH 2016

TOPNOTCH MEDICAL BOARD PREP PHARMACOLOGY SUPEREXAM For inquiries visit www.topnotchboardprep.com.ph or email us at [email protected] Item # 35

36

37

38

39

40

41

QUESTION

EXPLANATION

Which of the following statements regarding the The metabolism of theophylline depends on age; the pharmacokinetics of theophylline is correct? half-life of the drug in children is much shorter than A. It is primarily metabolized by the kidney in adults. The methylxanthines are all well absorbed B. Its metabolism depends on age and are metabolized in the liver. The C. It is poorly absorbed after oral administration PHARMACODYNAMIC porperties of theophylline D. It has a wide therapeutic index include its adenosine-receptor antagonist activity E. It stimulates phosphodiesterase and the inhibition of phosphodiesterase.

AUTHOR LESTER BRYAN CO (TOP 10 - AUG 2015 MED BOARDS; TOPNOTCH MD FROM UST)

A 74-year-old woman who is undergoing Oprelvekin has been shown to reduce the need for LESTER BRYAN chemotherapy for advanced lung cancer presents to platelet transfusions following myelosuppressive CO (TOP 10 - AUG the infusion center for her next treatment. Before chemotherapy. Erythropoietin is used for anemia. 2015 MED each treatment her white count, emoglobin, and Filgrastim and sargramostim are used for BOARDS; platelet counts are checked to make sure she is not neutropenia. Leucovorin is used in patients TOPNOTCH MD experiencing chemotherapy-related cytotoxicity. Her undergoing treatment with methotrexate, to prevent FROM UST) blood sample is run in the analyzer, and her platelet some of its side effects. count is reported to be at a dangerously low level. Which medication is her oncologist likely to prescribe in this situation, along with a platelet transfusion? A. Erythropoietin B. Oprelvekin C. Filgrastim D. Sargramostim E. Leucovorin A 34-year-old carpenter presents to the ER after an Epinephrine is contraindicated as an anesthetic LESTER BRYAN accident in which he inadvertently chopped off the tip adjuvant for surgeries involving most facial CO (TOP 10 - AUG of his index finger. He is taken to the OR for structures, digits, and the penis, because of the risk 2015 MED reattachment of the digit, and after sedation, a local of vascular compromise. This agent causes BOARDS; anesthetic is administered around the site of the decreased blood loss for most other surgeries TOPNOTCH MD injury. The local anesthetic used in the procedure did because of vasoconstriction. Although local FROM UST) not contain any epinephrine, as it usually does for anesthetic agents such as lidocaine or xylocaine can most surgical procedures. The reason for this is: cause mild local tissue swelling, epinephrine does A. Epinephrine causes increased blood loss during not; either way, it is not a contraindication for hand delicate surgery surgery. Epinephrine causes elevated blood B. Epinephrine causes swelling of the tissues, making pressure when administered systemically; however, surgery more challenging it has no systemic side effects when administered C. Epinephrine is contraindicated in emergency locally. surgery D. Epinephrine causes vasoconstriction, which can lead to vascular ischemia in digits E. Epinephrine can cause hypotension when administered with sedative agents A 30-year-old woman has suffered from cyclical Sumatriptan is a 5-HT1D agonist. An example of an LESTER BRYAN migraines for many years. She now presents to her agent known as a 5-HT1A agonist would be CO (TOP 10 - AUG physician asking for a medication designated buspirone, an antianxiety agent. Fluoxetine is an 2015 MED specifically for migraines, not just a general pain example of a serotonin-reuptake inhibitor. BOARDS; reliever. Her physician decided to prescribe Ondansetron, an antinausea medication, is a 5-HT3 TOPNOTCH MD sumatriptan as a trial medication. The patient, who is antagonist. The antipsychotic medication Risperdal FROM UST) a biochemist, would like to know how this medication is an example of a 5-HT2A antagonist. works. A. It is a 5-HT1A agonist B. It is a 5-HT1D agonist C. It blocks reuptake of serotonin D. It is a 5-HT3 antagonist E. It is a 5-HT2A antagonist A 12-year-old boy presents with a rash on the palms Doxycycline, a tetracycline, is the antibiotic of choice LESTER BRYAN and soles of his feet as well as fever and headache. He to treat Rocky Mountain spotted fever, a rickettsial CO (TOP 10 - AUG was camping last weekend and admits to being bitten disease. Streptomycin can be used to treat plague 2015 MED by a tick. His Weil-Felix test result is positive, and brucellosis. Bacitracin is only used topically. BOARDS; suggesting Rocky Mountain spotted fever. What Ciprofloxacin can be used to treat anthrax, and TOPNOTCH MD antibiotic should be given? erythromycin is the most effective drug for the FROM UST) A. Streptomycin treatment of Legionnaires disease. B. Bacitracin C. Ciprofloxacin D. Doxycycline E. Erythromycin A 55-year-old woman is admitted to the surgical Milrinone reduces left ventricular filling pressure LESTER BRYAN intensive care unit after having a coronary artery and thus enhances cardiac output. It is related to the CO (TOP 10 - AUG bypass grafting of four of her coronary vessels. anticholinergic agent biperiden. Milrinone inhibits 2015 MED Overnight she develops hypotension, and her cardiac cardiac phosphodiesterase type 3. It increases cAMP, BOARDS; output, as measured by the Swan-Ganz catheter, is and therefore intracellular calcium. TOPNOTCH MD significantly lower than it had been postsurgery. You FROM UST) decide to give her a dose of milrinone. This results in an increase in her cardiac output. How does milrinone work? A. It is a cholinergic agonist B. It reduces left ventricular filling pressure C. It potentiates cardiac phosphodiesterase type 3 D. It decreases cyclic AMP E. It decreases intracellular calcium Which of the following is true? Choramphenicol is one of the few antibiotics with GEORGE A. Azithromycin has lesser bioavailability when higher oral bioavailability than parenteral form. MICHAEL taken with food SOSUAN (TOP 5 - B. Metronidazole has the same oral and parenteral AUG 2015 MED bioavailability. BOARDS; C. The oral bioavailabity of chloramphenicol is TOPNOTCH MD higher than parenteral form. FROM UST)

TOPNOTCH MEDICAL BOARD PREP PHARMACOLOGY SUPEREXAM Page 6 of 83 For inquiries visit www.topnotchboardprep.com.ph or email us at [email protected]

TOPNOTCH EXAM MIDTERM 1 EXAM - MARCH 2016

MIDTERM 1 EXAM - MARCH 2016

MIDTERM 1 EXAM - MARCH 2016

MIDTERM 1 EXAM - MARCH 2016

MIDTERM 1 EXAM - MARCH 2016

MIDTERM 1 EXAM - MARCH 2016

MIDTERM 2 EXAM - MARCH 2016

TOPNOTCH MEDICAL BOARD PREP PHARMACOLOGY SUPEREXAM For inquiries visit www.topnotchboardprep.com.ph or email us at [email protected] Item #

QUESTION

EXPLANATION

AUTHOR

TOPNOTCH EXAM

42

The following has increased bioavailabity when taken with food, except A. Cefuroxime B. Fusic acid C. Nitrofuratoin D. Griseofulvin E. NOTA

All of the choices has increased bioavailability when taken with food.

GEORGE MICHAEL SOSUAN (TOP 5 - AUG 2015 MED BOARDS; TOPNOTCH MD FROM UST)

MIDTERM 2 EXAM - MARCH 2016

43

The following drugs have concentration dependent killing, except A. Aminoglycosides B. Fluoroquinolones C. Penicillin D. Metronidazole E. NOTA

Concentration dependent or dose-dependent killing means that the higher the concentration, the greater the bactericidal effect. Examples are aminoglycosides, fluoroquinolines, daptomycin, metronidazole, streptogramins. Penicillin employs time-dependent killing

GEORGE MICHAEL SOSUAN (TOP 5 - AUG 2015 MED BOARDS; TOPNOTCH MD FROM UST)

MIDTERM 2 EXAM - MARCH 2016

44

Strongest inhibitor of CYP3A4 amongst the protease inhibitor A. Ritonavir B. Saquinavir C. Lopinavir D. Amprenavir E. Atazanavir

Ritonavir - Strongest

GEORGE MICHAEL SOSUAN (TOP 5 - AUG 2015 MED BOARDS; TOPNOTCH MD FROM UST)

MIDTERM 2 EXAM - MARCH 2016

45

High dose statins are given during ACS, the MOA of which in these cases is: A. Inhibit HMG-CoA reductase B. Stabilize plaque C. Reduce TAG synthesis D. Increase HDL E. AOTA



GEORGE MICHAEL SOSUAN (TOP 5 - AUG 2015 MED BOARDS; TOPNOTCH MD FROM UST)

MIDTERM 2 EXAM - MARCH 2016

46

Single dose drug used in eradication of meningococcal carrer state A. RIfampicin B. Ciprofloxacin C. TMP/SMX D. Chloramphenicol E. NOTA

Drugs used to eradicate meningococcal carrier state: 1. rifampin 2. ciprofloxacin (single dose) 3. ceftriaxone (single dose)

GEORGE MICHAEL SOSUAN (TOP 5 - AUG 2015 MED BOARDS; TOPNOTCH MD FROM UST)

MIDTERM 2 EXAM - MARCH 2016

47

Single dose oral antibacterial agent used to eradicate uncomplicated gonorrhoea A. Ceftriaxone B. Cefixime C. Azithromycin D. Both B and C E. AOTA

Ceftriaxone - single dose IM s

GEORGE MICHAEL SOSUAN (TOP 5 - AUG 2015 MED BOARDS; TOPNOTCH MD FROM UST)

MIDTERM 2 EXAM - MARCH 2016

48

Disruption of cell membrane is not seen with A. Amphotericin B B. Fluconazole C. Nystatin D. Polymxin B E. Griseofulvin

Fluconazole inhibits ergosterol syntehsis

GEORGE MICHAEL SOSUAN (TOP 5 - AUG 2015 MED BOARDS; TOPNOTCH MD FROM UST)

MIDTERM 2 EXAM - MARCH 2016

49

Highly leukogenic anti-neoplastic drugs A. Topoisomerase II inhibitor B. Alkylating agents C. Anthrcycline antibiotics D. Both A and B E. AOTA

Toposiomerase II inhibitors and alkylating agents are the most leukogenic anti-neoplastic agents.

GEORGE MICHAEL SOSUAN (TOP 5 - AUG 2015 MED BOARDS; TOPNOTCH MD FROM UST)

MIDTERM 2 EXAM - MARCH 2016

50

The prokinetic MOA of metoclopramide A. Inhibition of dopamine receptors in the gut B. Inhibition of motilin receptors in the gut C. Inhibition of serotonin receptors in the gut D. AOTA E. Both A and B

Metoclopramide inhibits the D2 receptors in the gut; erythromycin stimulates the molitin receptors in the gut.

GEORGE MICHAEL SOSUAN (TOP 5 - AUG 2015 MED BOARDS; TOPNOTCH MD FROM UST)

MIDTERM 2 EXAM - MARCH 2016

51

The following anti-convulsant is not sedating A. Phenobarbital B. Phenytoin C. Clonazepam D. Gabapentin E. NOTA

Phenytoin is the oldest non-sedating anti-seizure drug

GEORGE MICHAEL SOSUAN (TOP 5 - AUG 2015 MED BOARDS; TOPNOTCH MD FROM UST)

MIDTERM 2 EXAM - MARCH 2016

D. Both B and C E. AOTA

TOPNOTCH MEDICAL BOARD PREP PHARMACOLOGY SUPEREXAM Page 7 of 83 For inquiries visit www.topnotchboardprep.com.ph or email us at [email protected]

TOPNOTCH MEDICAL BOARD PREP PHARMACOLOGY SUPEREXAM For inquiries visit www.topnotchboardprep.com.ph or email us at [email protected] Item # 52

QUESTION

EXPLANATION

AUTHOR

TOPNOTCH EXAM MIDTERM 2 EXAM - MARCH 2016

IOP can be decreased by increasing aqueous outflow with the following drugs A. Timolol B. Latanoprost C. Mannitol D. Dorzolamide E. NOTA

PG analogues decrease IOP by increasing aqueous outflow. CA inhibitors and Beta-blocker decrease aqeuous production. Alpha2 agonist inreases outflow and decreases production.

GEORGE MICHAEL SOSUAN (TOP 5 - AUG 2015 MED BOARDS; TOPNOTCH MD FROM UST)

Ampicillin is eliminated by first-order kinetics. Which of the following statements best describes the process by which the plasma concentration of this drug declines? A. There is only 1 metabolic path for drug elimination B. The half-life is the same regardless of the plasma concentration C. The drug is largely metabolized in the liver after oral administration and has low bioavailability D. The rate of elimination is proportional to the rate of administration at all times E. The drug is distributed to only 1 compartment outside the vascular system Which helminthic infection does not respond to treatment with praziquantel? A. Hydatid disease B. Opisthorchiasis C. Paragonimiasis D. Pork tapeworm infection E. Schistosomiasis

"First-order" means that the elimination rate is proportional to the concentration perfusing the organ of elimination. The half-life is a constant. The rate of elimination is proportional to the rate of administration only at steady state. The order of elimination is independent of the number of compartments into which a drug distributes.

GEORGE MICHAEL SOSUAN (TOP 5 - AUG 2015 MED BOARDS; TOPNOTCH MD FROM UST)

MIDTERM 2 EXAM - MARCH 2016

In hydatid disease, praziquantel has marginal efficacy because it does not affect the inner germinal membrane of Echinococcus granulosus present in hydatid cysts.

GEORGE MICHAEL SOSUAN (TOP 5 - AUG 2015 MED BOARDS; TOPNOTCH MD FROM UST)

MIDTERM 2 EXAM - MARCH 2016

55

This drug that produces a permanent reduction in thyroid activity is A. 131I B. Amiodarone C. Propranolol D. Propylthiouracil E. Triiodothyronine

Radioactive iodine is the only medical therapy that produces a permanent reduction of thyroid activity.

GEORGE MICHAEL SOSUAN (TOP 5 - AUG 2015 MED BOARDS; TOPNOTCH MD FROM UST)

MIDTERM 2 EXAM - MARCH 2016

56

Tolerance is seen in which of the following drugs: A. Nitrates B. Dopamine C. Epinephine D. Aspirin E. AOTA

Tolerance develop with nitrates when used for more than 8 hours without interruption

GEORGE MICHAEL SOSUAN (TOP 5 - AUG 2015 MED BOARDS; TOPNOTCH MD FROM UST)

MIDTERM 2 EXAM - MARCH 2016

57

The only beta-lactam antibiotcs that does not cross react with penicillins A. Ceftriaxone B. Aztreonam C. Meropenem D. Aminoglycosides E. Vancomycin

The monobactams have activity against aerobic Gram negative bacilli and dose not cross react with penicillin.

GEORGE MICHAEL SOSUAN (TOP 5 - AUG 2015 MED BOARDS; TOPNOTCH MD FROM UST)

MIDTERM 2 EXAM - MARCH 2016

58

Which of the following can't be used topically? A. Clindamycin B. Erythromycin C. Amoxicllin D. Fusidic acid E. Mupirocin

Topical clindamycin and erythromycin are used topically againts acne. Topical fusidic acid and mupirocin are used topically againts other superficial bacterial infection.

GEORGE MICHAEL SOSUAN (TOP 5 - AUG 2015 MED BOARDS; TOPNOTCH MD FROM UST)

MIDTERM 2 EXAM - MARCH 2016

59

Fifith Generation cephalosporin with activity against Pseudomonas aeruginosa A. Ceftraline B. Ceftabiprole C. Ceftazidime D. Cefepime E. Cefixime

Both A and B are fifth generation but only B has activity against Pseudomonas.

GEORGE MICHAEL SOSUAN (TOP 5 - AUG 2015 MED BOARDS; TOPNOTCH MD FROM UST)

MIDTERM 2 EXAM - MARCH 2016

60

Blocks the final common pathway of platelet aggregation A. ASA B. Abciximab C. Clopidogrel D. Both A and B E. AOTA

ASA - inhibits COX; Abciximab - GPIIBIIIA inhibitor; Clopidogrel - ADP receptor inhibitor

GEORGE MICHAEL SOSUAN (TOP 5 - AUG 2015 MED BOARDS; TOPNOTCH MD FROM UST)

MIDTERM 2 EXAM - MARCH 2016

61

Which of the ff statements regarding drug elimination is correct? A. Elimination of parent molecules terminats the drug's action for drugs with active metabolites B. First order eliminination occurs when the concentration of the drug decreases exponentially over time C. Zero order kinetics exhibits the characteristic half life of most drugs D. First order kinetics occurs when drugs have saturated their elimination mechanisms

Elimination of parent molecules does not terminate the drug's action for drugs with active metabolites. First order kinetics exhibits the characteristic half life of most drugs. Zero order kinetics occurs when drugs have saturated their elimination mechanisms. Phenobarbital does not display zero roder kinetics

JAN CHRISTIAN FELICIANO (TOP 2 - AUG 2015 MED BOARDS; TOPNOTCH MD FROM UST)

MIDTERM 3 EXAM - MARCH 2016

53

54

TOPNOTCH MEDICAL BOARD PREP PHARMACOLOGY SUPEREXAM Page 8 of 83 For inquiries visit www.topnotchboardprep.com.ph or email us at [email protected]

TOPNOTCH MEDICAL BOARD PREP PHARMACOLOGY SUPEREXAM For inquiries visit www.topnotchboardprep.com.ph or email us at [email protected] Item #

QUESTION

EXPLANATION

AUTHOR

TOPNOTCH EXAM

E. Phenobarbital is an example of a drug that exhibits zero order kinetics

62

27-year old female presents to the ER approximately one hour after swallowing 70 propranolol tablets with suicidal intent. At the time of assessment she is drowsy (GCS 13) with a heart rate of 46 bpm and BP 100/60. What drug/class can be given by virtue of physiologic antagonism? A. Beta agonists B. Alpha agonists C. Glucagon D. Diazepam E. Atropine A patient was put on standrard dose of warfarin therapy, aftter 1 week he suddenly experienced hematochezia. Which of the ff drugs he is also currently taking could have caused this? A. Valproic acid B. Ethanol C. Carbamazepine D. Isoniazid E. Griseofulvin Which of the ff teratogenic drugs causes atrialization of the right ventricle? A. Misoprostol B. Lithium C. Ethanol D. Penicillamine E. Phenytoin

The physiological antagonism is related to the fact that glucagon activates cAMP through nonadrenergic pathways. Because the adrenergic pathways are blocked by propanol, glucagon essentially uses a back door to enhance myocardial activity.

JAN CHRISTIAN FELICIANO (TOP 2 - AUG 2015 MED BOARDS; TOPNOTCH MD FROM UST)

MIDTERM 3 EXAM - MARCH 2016

INH is also an inhibitor. Rifampicin is an inducer.. The patient probably is taking a CYP450 inbibitor such as valproic acid that decreased the metabolism of warafarin leading to its toxic side effectds. All the rest of the drugs are inducers

JAN CHRISTIAN FELICIANO (TOP 2 - AUG 2015 MED BOARDS; TOPNOTCH MD FROM UST)

MIDTERM 3 EXAM - MARCH 2016

Ebstein's anomaly from lithium is a congenital heart defect in which the septal and posterior leaflets of the tricuspid valve are displaced towards the apex of the right ventricle of the heart. There is subsequent 'atrialization' of a portion of the morphologic right ventricle

JAN CHRISTIAN FELICIANO (TOP 2 - AUG 2015 MED BOARDS; TOPNOTCH MD FROM UST)

MIDTERM 3 EXAM - MARCH 2016

A patient came to the ER with diarrhea, tremors and excessive sweating. HR is 50, BP is 120/70, RR is 22. Pupils are 1-2 mm. What drug can be given to the patient? A. Naloxone B. Neostigmine C. Diazepam D. Atropine E. EDTA On a patient with signs of shock, At a dose of 5 mcg/kg/min, dopamine will predominanty have what effect? A. Increased peripheral vascular resistance B. Activation of the RAAS C. Inotropic effect D. Renal vasodilation E. Arrythmias

The patinet is exhibting symptoms of organophosphate poisoning with increased secretion of all body fluids plus bradycardia, miosis and skeletal muscle exictation. Antidote is Atropine and Pralidoxime

JAN CHRISTIAN FELICIANO (TOP 2 - AUG 2015 MED BOARDS; TOPNOTCH MD FROM UST)

MIDTERM 3 EXAM - MARCH 2016

Dopamine has different dose dependent actions. At 1-2 mcg/kg/min, it preferentially activates D1 receptor. At 2-10 mcg, it activates B1 receptor and at >10 mcg, it activates A1 receptors

JAN CHRISTIAN FELICIANO (TOP 2 - AUG 2015 MED BOARDS; TOPNOTCH MD FROM UST)

MIDTERM 3 EXAM - MARCH 2016

67

Which of the ff drugs acts to improve aqueous humor outflow in patients with glaucoma? A. Pilocarpine B. Mannitol C. Apraclonidine D. Timolol E. Acetazlamide

Pilocarpine is a cholinomimeti agent that contracts the ciliary muscle opening the trabecular meshwork. All of the other drugs decreases secretion of aqueous humor

JAN CHRISTIAN FELICIANO (TOP 2 - AUG 2015 MED BOARDS; TOPNOTCH MD FROM UST)

MIDTERM 3 EXAM - MARCH 2016

68

All fo the ff are side effects of lithium therapy EXCEPT? A. Tremors B. Hyperthyroidism C. Nephrogenic diabetic insipidus D. Edema E. Sinus node depression

Lithium decreases thyroid function in most patients exposed to the drug and some show symptoms of hypothyroidism. Tremors is the most common adverse effect and occurs even on therapeutic doses.

JAN CHRISTIAN FELICIANO (TOP 2 - AUG 2015 MED BOARDS; TOPNOTCH MD FROM UST)

MIDTERM 3 EXAM - MARCH 2016

69

This beta blocker has a unique mechanism of action owing to its it has a nitric oxide (NO)-potentiating vasodilatory effect in addition to beta blockade. A. Carvedilol B. Pindolol C. Nadolol D. Esmolol E. Nebivolol

Nebivolol is a β1 receptor blocker with nitric oxidepotentiating vasodilatory effect used in treatment of hypertension

JAN CHRISTIAN FELICIANO (TOP 2 - AUG 2015 MED BOARDS; TOPNOTCH MD FROM UST)

MIDTERM 3 EXAM - MARCH 2016

63

64

65

66

TOPNOTCH MEDICAL BOARD PREP PHARMACOLOGY SUPEREXAM Page 9 of 83 For inquiries visit www.topnotchboardprep.com.ph or email us at [email protected]

TOPNOTCH MEDICAL BOARD PREP PHARMACOLOGY SUPEREXAM For inquiries visit www.topnotchboardprep.com.ph or email us at [email protected] Item # 70

QUESTION

EXPLANATION

AUTHOR

TOPNOTCH EXAM MIDTERM 3 EXAM - MARCH 2016

What is the anti-hypertensive drug of choice for patients with diabetic nephropathy? A. Nifedipine B. Clonidine C. Enalapril D. Atenolol E. Thiazides

ACE inhibitors are renal efferent arteriole dilators decreasing albumin excretion and slowing progression of CKD.

JAN CHRISTIAN FELICIANO (TOP 2 - AUG 2015 MED BOARDS; TOPNOTCH MD FROM UST)

71

Which of the ff electrolyte abnormality will increase digitalis toxicity? A. Hyponatremia B. Hypokalemia C. Hypocalcemia D. Hypermagnesemia E. Hyperphosphatemia

Digitalis toxicity is increased in patients with hypokalemia, hypomagnesemia and hypercalcemia. Potassium and digitalis interact by inhibiting each other’s binding to Na+/K+-ATPase; therefore, hyperkalemia reduces the enzyme-inhibiting actions of cardiac glycosides, whereas hypokalemia facilitates these actions.

JAN CHRISTIAN FELICIANO (TOP 2 - AUG 2015 MED BOARDS; TOPNOTCH MD FROM UST)

MIDTERM 3 EXAM - MARCH 2016

72

What is the drug of choice for the rapid termination of paroxysmal supraventricular tachycardia? A. Adenosine B. Magnesium C. Lidocaine D. Amiodarone E. Verapamil

Adenosine is the drug of choice for paroxysmal SVT. Magnesium is the drug of choice for Torsades de pointes. Lidocaine is the drug of choice for post MI arrythmias and digoxin induced arrythmias.

JAN CHRISTIAN FELICIANO (TOP 2 - AUG 2015 MED BOARDS; TOPNOTCH MD FROM UST)

MIDTERM 3 EXAM - MARCH 2016

73

A child was brought to you with severe itching due to multiple insect bites. The mother requested for a non sedating anti histaminic agent. What drug can you give? A. Chlorpheniramine B. Montelukast C. Meclizine D. Fexofenadine E. Hydroxyzine A 15 year old post partum girl in Fabella hospital was bleeding profusely after giving birth despite oxytocin administration, what class of drug can you give as second line treatment? A. 5-HT1 agonist B. 5-HT2 antagonist C. 5-HT3 antagonist D. 5-HT4 antagonist E. Alpha 1 agonist This drug class used in asthma exerts its effect by contraction of engorged vessels in the bronchial mucosa and reduces bronchial hyperreactivity? A. Beta 2 agonists B. Inhaled corticosteroids C. Methylxanthines D. Mast cell stabilizers E. Antimuscarinic agent

SIMILAR TO PREVIOUS BOARD EXAM CONCEPT/PRINCIPLE. Among the choices only fexofenadine is a 2nd generation antihistamine. Montelukast is a leukotriene receptor antagonist. All the others arer 1st gen antihistamines

JAN CHRISTIAN FELICIANO (TOP 2 - AUG 2015 MED BOARDS; TOPNOTCH MD FROM UST)

MIDTERM 3 EXAM - MARCH 2016

SIMILAR TO PREVIOUS BOARD EXAM CONCEPT/PRINCIPLE. Methylergonovine can be given which is a 5-HT2 receptor antagonist selective to the uterus.

JAN CHRISTIAN FELICIANO (TOP 2 - AUG 2015 MED BOARDS; TOPNOTCH MD FROM UST)

MIDTERM 3 EXAM - MARCH 2016

SIMILAR TO PREVIOUS BOARD EXAM CONCEPT/PRINCIPLE. Corticosteroids do not relax airway smooth muscle directly but reduce bronchial reactivity and reduce the frequency of asthma exacerbations if taken regularly. Their effect on airway obstruction may be due in part to their contraction of engorged vessels in the bronchial mucosa and their potentiation of the effects of βreceptor agonists, but their most important action is inhibition of the infiltration of asthmatic airways by lymphocytes, eosinophils, and mast cells. Ethosuxime blocks the T type Calcium current in the thalamus and is the drug of choice for absence seizures. All the other drugs block Na channels.

JAN CHRISTIAN FELICIANO (TOP 2 - AUG 2015 MED BOARDS; TOPNOTCH MD FROM UST)

MIDTERM 3 EXAM - MARCH 2016

JAN CHRISTIAN FELICIANO (TOP 2 - AUG 2015 MED BOARDS; TOPNOTCH MD FROM UST)

MIDTERM 3 EXAM - MARCH 2016

74

75

76

Which of the ff anticonvulsant agents do not exerts its effect through Na channel blocakade? A. Phenytoin B. Carbamazepine C. Topiramate D. Valproic acid E. Ethosuximide

77

Which of the inhaled anesthetics forms a toxic metabolite, Compound A, that is nephrotoxic at high doses? A. Halothane B. Isoflurane C. Enflurane D. Sevoflurane E. Desflurane

Well known side effects: Halothane-- Hepatitis. Isoflurane- Coronary steal syndrome. Enflurane- Seizures. Sevoflurane- Renal insufficiency. Desflurane- Pulmonary irritant

JAN CHRISTIAN FELICIANO (TOP 2 - AUG 2015 MED BOARDS; TOPNOTCH MD FROM UST)

MIDTERM 3 EXAM - MARCH 2016

78

MRSA, a multi drug resistant organism is kept at bay through administration of vancomycin. Currently, reports of MRSA resistant to vancomycin (VRSA) are being reported. What is the mechanism of resistance to vancomycin? A. Structural change in target PBP B. Formation of extended spectrum beta lactamases C. D ala-ala is modified to D ala-lactate D. Changes in porin structure of outer cell wall E. Extrusion of the drug through P-glycoprotein efflux pump

Structural change in target PBP is mechanism of resistance of MRSA not VRSA. Resistance to vancomycin in vancomycin-resistant S aureus strains is due to modification of the D-Ala-D-Ala binding site of the peptidoglycan building block in which the terminal D-Ala is replaced by D-lactate. This results in the loss of a critical hydrogen bond that facilitates high- affinity binding of vancomycin to its target and loss of activity.

JAN CHRISTIAN FELICIANO (TOP 2 - AUG 2015 MED BOARDS; TOPNOTCH MD FROM UST)

MIDTERM 3 EXAM - MARCH 2016

TOPNOTCH MEDICAL BOARD PREP PHARMACOLOGY SUPEREXAM Page 10 of 83 For inquiries visit www.topnotchboardprep.com.ph or email us at [email protected]

TOPNOTCH MEDICAL BOARD PREP PHARMACOLOGY SUPEREXAM For inquiries visit www.topnotchboardprep.com.ph or email us at [email protected] Item # 79

QUESTION

EXPLANATION

AUTHOR

TOPNOTCH EXAM MIDTERM 3 EXAM - MARCH 2016

This antidiabetic drug because of its unique mechanism of action should be taken just before ingesting the first portion of each meal? A. Exenatide B. Miglitol C. Pramlintide D. Pioglitazone E. Metformin

Acarbose and miglitol are competitive inhibitors of the intestinal α-glucosidases and reduce postmeal glucose excursions by delaying the digestion and absorption of starch and disaccharide. Both acarbose and miglitol are taken in doses of 25–100 mg just before ingesting the first portion of each meal

JAN CHRISTIAN FELICIANO (TOP 2 - AUG 2015 MED BOARDS; TOPNOTCH MD FROM UST)

80

What anti-TB medication readily penetrates into phagocytic cells and can kill organisms that are poorly accessible to many other drugs, such as intracellular organisms and those sequestered in abscesses and lung cavities? A. Isoniazid B. Rifampicin C. Ethambutol D. Pyrazinamide E. Streptomycin

As stated verbatim in Katzung, rifampicin readily penetrates most tissues and penetrates into phagocytic cells. It can kill organisms that are poorly accessible to many other drugs, such as intracellular organisms and those sequestered in abscesses and lung cavities.

JAN CHRISTIAN FELICIANO (TOP 2 - AUG 2015 MED BOARDS; TOPNOTCH MD FROM UST)

MIDTERM 3 EXAM - MARCH 2016

81

1. Which of the following medications is NOT contraindicated in a patient taking RANOLAZINE? a. Losartan b. Itraconazole c. clarithromycin d. metronidazole e. prednisone

Medscape expect questions on never before heard drugs.

ANDREW TIU (TOP 1 - AUG 2015 MED BOARDS; TOPNOTCH MD FROM CIM)

FINAL EXAM - MARCH 2016

82

2. Phoebe Marie, G2 P1 (1001), recently traveled to Palawan and is taking Quinine for malaria. Which of the following medications can she take? a. clindamycin b. doxycycline c. primaquine d. Halofantrine e. all of the above 3. Which of the following drugs is a reversible acetylcholinesterase inhibitor that causes an increase in concentrations of acetylcholine and is metabolized by cholinesterase and excreted mainly by the kidneys and used in Alzheimer's disease? a. Memantine b. Galantamine c. Rivastigmine d. Donepezil e. none of the above 4. Which of the following is the mechanism why Cefuroxime is not used for meningitis? a. poor penetration to CSF b. levels achieved in CSF not adequate to sterilize it c. potential for delayed sterilization and therapeutic failures d. presence of efflux pumps e. none of the above 5. Which of the following is the drug topically used for acne? a. allylamine b. clindamycin c. mupirocin d. bacitracin e. none of the above

Halofantrine is not used for chemoprohylaxis because of its potential for quinidine like cardiotoxicity (QT prolongation) and embryotoxicity. Likewise, doxycycline and primaquine are contraindicated during pregnancy. Katzung Pharmacology Examination and Board Review 10th edition p.453

ANDREW TIU (TOP 1 - AUG 2015 MED BOARDS; TOPNOTCH MD FROM CIM)

FINAL EXAM - MARCH 2016

Memantine - low to moderate affinity uncompetitive NMDA receptor antagonist blocking receptor only under conditions of excessive stimulation without effect of normal neurotransmission Donepezil - acetylcholinesterase inhibitor but metabolized by hepatic P - 450 enzymes Galantamine - increases acetylcholine from surviving presynaptic nerve terminals by modulating nicotinic Ach receptor Medscape http://pedsinreview.aappublications.org/content/2 9/8/264.full

ANDREW TIU (TOP 1 - AUG 2015 MED BOARDS; TOPNOTCH MD FROM CIM)

FINAL EXAM - MARCH 2016

ANDREW TIU (TOP 1 - AUG 2015 MED BOARDS; TOPNOTCH MD FROM CIM)

FINAL EXAM - MARCH 2016

Goodman Gilman 12th edition. Dermatology section

ANDREW TIU (TOP 1 - AUG 2015 MED BOARDS; TOPNOTCH MD FROM CIM)

FINAL EXAM - MARCH 2016

86

6. Which of the following anti folate drugs is a blood schizonticide? a. Atovaquone b. Artesunate c. Halofantrine d. Doxycycline e. Pyrimethamine

Katzung Pharmacology Examination and Board Review 10th edition p.453

ANDREW TIU (TOP 1 - AUG 2015 MED BOARDS; TOPNOTCH MD FROM CIM)

FINAL EXAM - MARCH 2016

87

7. Which of the following is a long acting insulin? a. Lantus b. Lente c. Glargine d. Detemir e. Glulisine

D and E - short acting A and B - intermediate acting Katzung Pharmacology Examination and Board Review 10th edition p.363

ANDREW TIU (TOP 1 - AUG 2015 MED BOARDS; TOPNOTCH MD FROM CIM)

FINAL EXAM - MARCH 2016

88

8. Which of the following is NOT true of Gentamicin? a. killing action continues when plasma levels decline below measurable levels b. lesser efficacy when administered as a single large dose c. in vivo efficacy not directly related to time above MIC d. as plasma level is increased above MIC, it kills an increasing proportion of bacteria at a more rapid rate e. toxicity depends on both critical plasma

has greater efficacy when administered as a single large dose than when given as multiple smaller doses Katzung Pharmacology Examination and Board Review 10th edition p.399

ANDREW TIU (TOP 1 - AUG 2015 MED BOARDS; TOPNOTCH MD FROM CIM)

FINAL EXAM - MARCH 2016

83

84

85



TOPNOTCH MEDICAL BOARD PREP PHARMACOLOGY SUPEREXAM Page 11 of 83 For inquiries visit www.topnotchboardprep.com.ph or email us at [email protected]

TOPNOTCH MEDICAL BOARD PREP PHARMACOLOGY SUPEREXAM For inquiries visit www.topnotchboardprep.com.ph or email us at [email protected] Item #

QUESTION

EXPLANATION

AUTHOR

TOPNOTCH EXAM

concentration and the time that such level is exceeded

89

90

91

92

93

94

95

96

9. Which of the following chemotherapeutic drugs is used for bladder cancer (intravesical administration) and control of malignant pericardial/ pleural/ peritoneal effusions (intracavitary)? a. cyclophosphamide b. thiotepa c. carmustine d. lomustine e. none of the above 10. Which of the following is a mixed FSH and LH agonist and is used to stimulate gonadal function including spermatogenesis and ovulation? a. menotropin b. urofollitropin c. Lutropin d. Somatropin e. Gonadorelin 11. Which of the following is not likely used for treatment of Herpes Simplex virus? a. Acyclovir b. Penciclovir c. Famciclovir d. Ganciclovir e. none of the above

Medscape expect questions on never before heard drugs.

ANDREW TIU (TOP 1 - AUG 2015 MED BOARDS; TOPNOTCH MD FROM CIM)

FINAL EXAM - MARCH 2016

B - FSH only C - LH only D - GH agonist E - GnRH agonist Katzung Pharmacology Examination and Board Review 10th edition p.327

ANDREW TIU (TOP 1 - AUG 2015 MED BOARDS; TOPNOTCH MD FROM CIM)

FINAL EXAM - MARCH 2016

D is used for CMV Katzung Pharmacology Examination and Board Review 10th edition p.429

ANDREW TIU (TOP 1 - AUG 2015 MED BOARDS; TOPNOTCH MD FROM CIM)

FINAL EXAM - MARCH 2016

12. A neonate in the NICU was observed by the Neonatologist fellow to have epicanthal folds, low nasal bridge, short palpebral fissures, flat midface, and indistinct philtrum. Which of the following medications did the mother likely take during pregnancy? a. phenytoin b. carbamazepine c. valproic acid d. ethyl alcohol e. warfarin 13. which of the following is the chemoprophylactic drug of choice for pregnant patients visiting falciparum malaria endemic places? a. chloroquine b. mefloquine c. doxycycline d. atovaquone - proguanil e. artemether - lumefantrine 14. Which of the following is NOT true of phenytoin? a. metabolism is nonlinear b. drug binds extensively to proteins in plasma c. fosphenytoin in the water soluble prodrug for parenteral administration d. metabolism is induced by isoniazid e. drug of choice for grand mal seizures



ANDREW TIU (TOP 1 - AUG 2015 MED BOARDS; TOPNOTCH MD FROM CIM)

FINAL EXAM - MARCH 2016

ANDREW TIU (TOP 1 - AUG 2015 MED BOARDS; TOPNOTCH MD FROM CIM)

FINAL EXAM - MARCH 2016

metabolism is enhanced by presence of inducers of liver metabolism (phenobarbital, rifampin) and inhibited by other drugs (cimetidine, isoniazid) Katzung Pharmacology Examination and Board Review 10th edition p.217

ANDREW TIU (TOP 1 - AUG 2015 MED BOARDS; TOPNOTCH MD FROM CIM)

FINAL EXAM - MARCH 2016

15. A 54 year old male presents to the clinic with a circular lesion on the arm and anesthesia on pin prick of the center of the lesion. Patient was then brought to Cebu Skin Clinic for further evaluation. Dr. AM impression was leprosy tuberculoid type and decides to start the patient with antimycobacterials. Which of the following is true of Dapsone? a. fluorodapsone is the respository form b. it is used as an alternative drug for treatment of PCP pneumonia c. it is not the most active drug for Leprosy d. mechanism of action is to inhibit arabinosyl transferase e. can be given to G6PD patients 16. DC, a 69 year old female, presented with resting tremors and shuffling gait. Upon cleaning her bathroom, she accidentally damaged the bathroom sink thus prompting consult. Having a good understanding of the pathophysiology of Parkinson’s disease, Dr. MC starts the patient with dopamine agonists. Which of the following can be given as monotherapy for mild parkinsonism and has high affinity for D3 receptor? a. bromocriptine b. pramipexole c. ropinirole d. apomorphine

acedapsone is the repository form that provides inhibitory plasma concentrations for several months. It is the most active drug for leprosy. Mechanism of action is inhibition of folic acid synthesis. It is contraindicated to G6PD patients. Katzung Pharmacology Examination and Board Review 10th edition p.415

ANDREW TIU (TOP 1 - AUG 2015 MED BOARDS; TOPNOTCH MD FROM CIM)

FINAL EXAM - MARCH 2016

Katzung Pharmacology Examination and Board Review 10th edition p.249

ANDREW TIU (TOP 1 - AUG 2015 MED BOARDS; TOPNOTCH MD FROM CIM)

FINAL EXAM - MARCH 2016

http://www.who.int/ith/ITH_chapter_7.pdf



TOPNOTCH MEDICAL BOARD PREP PHARMACOLOGY SUPEREXAM Page 12 of 83 For inquiries visit www.topnotchboardprep.com.ph or email us at [email protected]

TOPNOTCH MEDICAL BOARD PREP PHARMACOLOGY SUPEREXAM For inquiries visit www.topnotchboardprep.com.ph or email us at [email protected] Item #

QUESTION

EXPLANATION

AUTHOR

TOPNOTCH EXAM

e. cabergoline

97

17. Which of the following is the regimen for a 29 year old male with a testicular mass? a. ABVD regimen b. paclitaxel + carboplatin c. gemcitabine + erlotinib d. fluorouracil and leucovorin + oxaliplatin e. PEB regimen

A - hodgkin’s lymphoma B - ovarian C - pancreatic D - colorectal Katzung Pharmacology Examination and Board Review 10th edition p.468

ANDREW TIU (TOP 1 - AUG 2015 MED BOARDS; TOPNOTCH MD FROM CIM)

FINAL EXAM - MARCH 2016

98

18. NT a 39 year old male with adult polycystic kidney disease underwent kidney transplant. Which of the following immunophilin inhibitors bind to FK - binding protein 12 and inhibit the mTOR pathway which inhibits the T - cell proliferation response to IL - 2? a. sirolimus b. tacrolimus c. cyclosporin d. cyclophiline e. none of the above 19. A 23 year old female came in for unilateral throbbing headache associated with nausea and photophobia. Which of the following ergot alkaloids has no effects on the dopamine receptor D2 but stimulates the uterine smooth muscle? a. bromocriptine b. ergonovine c. ergotamine d. LSD e. methylergometrine 20. Which of the following prostaglandin derivatives increases outflow of aqueous humor? a. misoprostol b. alprostadil c. dinoprostone d. epoprostenol e. none of the above

Cyclosporine binds to cyclophilin and tacrolimus binds to FKBP. Both complexes inhibit calcineurin, a cytoplasmic phosphatase. Calcineurin regulates the ability of the nuclear factor of activated T cells to translocate to the nucleus and increase the production of key cytokines such as IL 2, 3, and IFN gamma. Cyclophilin and tacrolimus prevent the increased production of cytokines that normally occurs in response to T cell receptor activation. Katzung Pharmacology Examination and Board Review 10th edition p.482 Ergot alkaloids can produce marked and prolonged alpha receptor mediated vasoconstriction. Ergotamine is the prototype and has been a mainstay of treatment of acute attacks of migraine. Katzung Pharmacology Examination and Board Review 10th edition p.160

ANDREW TIU (TOP 1 - AUG 2015 MED BOARDS; TOPNOTCH MD FROM CIM)

FINAL EXAM - MARCH 2016

ANDREW TIU (TOP 1 - AUG 2015 MED BOARDS; TOPNOTCH MD FROM CIM)

FINAL EXAM - MARCH 2016

Latanoprost increases outflow of aqueous humor and reduces intraocular pressure in glaucoma. Katzung Pharmacology Examination and Board Review 10th edition p.176

ANDREW TIU (TOP 1 - AUG 2015 MED BOARDS; TOPNOTCH MD FROM CIM)

FINAL EXAM - MARCH 2016

99

100

101

102

103

104

Thru what route is the usual portal of entry in Page 94 of Topnotch Handout. The usual setting of Inorganic Mercury poisoning? either acute or chronic inorganic mercury poisoning A. Ingestion is thru Inhalation of inorganic mercury vapor. B. Topical application C. Inhalation D. Rectal E. None of the above

ANGELA DIAGNOSTIC PAULINE P. EXAM - AUG CALIMAG2015 LOYOLA (TOP 8 - FEB 2015 MED BOARDS; TOPNOTCH MD FROM UST) Using the Henderson-Hasselbach equation, in a pH Page 2 of Topnotch Handout. According to the ANGELA DIAGNOSTIC above the dissociation constant, a weak acid is Henderson-Hasselbach equation Weak acid above PAULINE P. EXAM - AUG present in the _____________ form. the pKa is unprotonated and charged. CALIMAG2015 A. unprotonated, uncharged LOYOLA (TOP 8 - B. protonated, charged FEB 2015 MED C. protonated, uncharged BOARDS; D. unprotonated, charged TOPNOTCH MD E. unpredictable FROM UST) The rectal route is said to have partial avoidance of Page 3 of Topnotch Handout. The superior rectal the first pass effect. Thru which of the following vein will drain to the Inferior mesenteric vein then venous drainage of the rectum will not bypass the to the portal vein, hence having a first pass effect. first pass effect : The middle rectal vein will drain thru the internal A. Superior rectal vein iliac vein then to the inferior vena cava, while the B. Middle rectal vein inferior rectal vein will drain via the internal C. Inferior rectal vein pudendal vein then thru the internal iliac vein then D. Both B and C thru the inferior vena cava. E. All of the above 15 units of Drug X was given to a patient. If this drug Page 4 of Topnotch Handout. In a drug undergoing undergoes first-order elimination how many half-lifes first-order elimination concentration decreases by will it take for the drug concentration to reach 1.88 50% for every half-life. SO if the drug was 15 units at units: the start, 1st half life- 7.5, 2nd half life 3.75, third A. 1 half life 1.875. B. 2 C. 3 D. 4 E. 5

ANGELA PAULINE P. CALIMAGLOYOLA (TOP 8 - FEB 2015 MED BOARDS; TOPNOTCH MD FROM UST)

DIAGNOSTIC EXAM - AUG 2015

ANGELA PAULINE P. CALIMAGLOYOLA (TOP 8 - FEB 2015 MED BOARDS; TOPNOTCH MD FROM UST)

DIAGNOSTIC EXAM - AUG 2015

TOPNOTCH MEDICAL BOARD PREP PHARMACOLOGY SUPEREXAM Page 13 of 83 For inquiries visit www.topnotchboardprep.com.ph or email us at [email protected]

TOPNOTCH MEDICAL BOARD PREP PHARMACOLOGY SUPEREXAM For inquiries visit www.topnotchboardprep.com.ph or email us at [email protected] Item # 105

106

107

108

109

110

QUESTION

EXPLANATION

Which of the following statements is/are true Page 4 of Topnotch Handout. Binding affinity refers regarding the binding affinity of a drug: to the fraction of receptors bound by a drug plotted A. Kd is the concentration required to bind 100% of against the log of the drug concentration. Kd is the the drug receptors concentration required to bind 50% of the B. The smaller the Kd the lesser the affinity of the receptors. The smaller the Kd, the greater the drug for its receptor affinity of a drug for its receptor, the more potent it C. A smaller Kd means that the drug is less potent is. D. Binding affinity refers to the fraction of receptors bound by a drug plotted against the log of the drug effect E. None of the above A drug is 90% cleared by the liver and 10% by the Page 7 of Topnotch Handout. Normal creatinine kidney. The normal dosage of the drug is 500mg/d. If clearance for healthy men is 97-137 mL/min. this drug is to be given for a patient with a creatinine Normal creatinine clearance for healthy women is clearance of 30ml/min, what should be the corrected 88-128 mL/min. In this patient his crea clearance is dose: A. 15 only 30ml/min. Hence to compute for the corrected B. 50 dose the formula is Corrected dose= Average dose C. 450 (aaplied only to the part of the dose cleared by the D. 465 kidney) x patient's crea clearance/ 100ml/min. In E. 500 this case 500 x 0.10= 50; 30/100= 0.3; ).3 x 50= 15. Corrected dose is 500 x 0.90=450+ 15=465. Which of the following teratogenic drug and effect is Page 9 of Topnotch Handout. correctly paired? Sulfonamides:kernicterus; Misoprostol: Mobius A. Sulfonamides: ototoxicity sequence; Phenytoin: Fetal hydantoin syndrome; B. Penicillamine: cutis laxa Lithium: Ebstein's anomaly. C. Misoprostol: Ebstein's anomaly D. Phenytoin: Mobius sequence E. Lithium: kernicterus

AUTHOR ANGELA PAULINE P. CALIMAGLOYOLA (TOP 8 - FEB 2015 MED BOARDS; TOPNOTCH MD FROM UST)

ANGELA PAULINE P. CALIMAGLOYOLA (TOP 8 - FEB 2015 MED BOARDS; TOPNOTCH MD FROM UST)

TOPNOTCH EXAM DIAGNOSTIC EXAM - AUG 2015

DIAGNOSTIC EXAM - AUG 2015

ANGELA DIAGNOSTIC PAULINE P. EXAM - AUG CALIMAG2015 LOYOLA (TOP 8 - FEB 2015 MED BOARDS; TOPNOTCH MD FROM UST) This drug inhibits the transport of Acetylcholine into Page 11 of Topnotch Handout. Storage of Ach into ANGELA DIAGNOSTIC vesicles for storage by VAT: vesicles is inhibited by vesamicol. PAULINE P. EXAM - AUG A. Hemicholinium CALIMAG2015 B. Botulinum LOYOLA (TOP 8 - C. Neostigmine FEB 2015 MED D. Betanechol BOARDS; E. Vesamicol TOPNOTCH MD FROM UST) A 55 y/o male presented at the ER with complaints of Page 12 of hand out. The patient is experiencing ANGELA DIAGNOSTIC 3 day diarrhea, frequent urination, increased symptoms of organophosphate poisoning. PAULINE P. EXAM - AUG salivation and sweating. You noted his pupils to be Remember the DUMBBELSS mnemonic! Diarrhea, CALIMAG2015 constricted. He works in a factory manufacturing Urination, Miosis, Bronchospasm, Bradycardia, LOYOLA (TOP 8 - pesticides for farms. Which of the following drugs will Excitation (skeletal muscle and CNS), Lacrimation, FEB 2015 MED you give this patient? Sweating and Salivation. The antidote to be given in BOARDS; A. Pyridostigmine organophosphate poisoning is Atropine (Cholinergic TOPNOTCH MD B. Edrophonium antagonist, muscarinic) and Pralidoxime FROM UST) C. Pilocarpine (Cholinesterase regenerator) D. Atropine E. Varenicline Your patient is in cardiogenic shock. Which of the Page 15 of Topnotch Handout. In cardiogenic shock ANGELA DIAGNOSTIC following is the drug of choice to increase cardiac the DOC is Dobutamine. It is a selective Beta 1 PAULINE P. EXAM - AUG contractility without the vasoconstricting effects? Agonist hence it has no Alpha 1 vasoconstricting CALIMAG2015 A. Norepinephrine effects. LOYOLA (TOP 8 - B. Dopamine FEB 2015 MED C. Epinephrine BOARDS; D. Phenylephrine TOPNOTCH MD E. Dobutamine FROM UST)

111

Which of the following beta-blockers are less likely to Page 16 of Topnotch Handout. The two beta blockers ANGELA cause bronchospasm in patients with asthma due to with intrinsic sympathomimetic activity are Pindolol PAULINE P. its partial agonist activity? and Acebutolol. CALIMAGA. Carvedilol LOYOLA (TOP 8 - B. Labetalol FEB 2015 MED C. Pindolol BOARDS; D. Propanolol TOPNOTCH MD E. Nadolol FROM UST)

DIAGNOSTIC EXAM - AUG 2015

112

A 65 y/o male on Clonidine for a few months for his Page 17 of Topnotch Handout. For rebound hypertension abruptly discontinued his medication hypertension due to clonidine withdrawal the DOC because he felt he was feeling better. After 2 days, is Phentolamine. Clonidine is an alpha 2 agonist, during which he had strong headaches. Three days sympathetic outflow blocker. Phentolamine Its after withdrawal, the patient had home blood primary action is vasodilation due to α1 blockade, pressure levels of 140–150/95–105 mmHg. Ten days but can also lead to reflex tachycardia because of after, the patient went to the emergency room with a hypotension and α2 inhibition, which increases symptomatic hypertensive crisis at 220/130 mmHg. sympathetic tone. The primary application for What is the drug of choice to treat the rebound phentolamine is for the control of hypertensive hypertension? emergencies. A. Phentolamine B. Hydrochlorthiazide C. Propanolol D. Hydralazine E. Nicardipine

DIAGNOSTIC EXAM - AUG 2015

ANGELA PAULINE P. CALIMAGLOYOLA (TOP 8 - FEB 2015 MED BOARDS; TOPNOTCH MD FROM UST)

TOPNOTCH MEDICAL BOARD PREP PHARMACOLOGY SUPEREXAM Page 14 of 83 For inquiries visit www.topnotchboardprep.com.ph or email us at [email protected]

TOPNOTCH MEDICAL BOARD PREP PHARMACOLOGY SUPEREXAM For inquiries visit www.topnotchboardprep.com.ph or email us at [email protected] Item # 113

114

115

116

117

118

119

120

QUESTION

EXPLANATION

By which mechanism does Angiotensin converting Page 19 of Topnotch Handout.ACE inhibitors enzyme inhibitors cause cough? produce vasodilation by inhibiting the formation of A. By inhibiting ACE and formation of angiotensin II angiotensin II. ACE also breaks down bradykinin (a B. By blocking the breakdown of bradykinin vasodilator substance). Therefore, ACE inhibitors, by C. By facilitating the release of norepinephrine from blocking the breakdown of bradykinin, increase sympathetic adrenergic nerves bradykinin levels, which can contribute to the D. By inhibiting norepinephrine reuptake vasodilator action of ACE inhibitors. The increase in E. By blocking angiotensin II stimulation of bradykinin is also believed to be responsible for a aldosterone secretion troublesome side effect of ACE inhibitors, namely, a dry cough. Angiotensin II also facilitates the release of norepinephrine from sympathetic adrenergic nerves and inhibits norepinephrine reuptake by these nerves. This effect of angiotensin II augments sympathetic activity on the heart and blood vessels. ACE inhibitors promote renal excretion of sodium and water (natriuretic and diuretic effects) by blocking the effects of angiotensin II in the kidney and by blocking angiotensin II stimulation of aldosterone secretion. This reduces blood volume, venous pressure and arterial pressure. This drug belongs to a class of antiarrhythmic drugs Page 24 of Topnotch Handout. Under the Singhwhich prolongs AP duration, PR interval, QRS Vaughn Williams classification. The class being duration and QT interval: A. Procainamide described are Class 1A Antiarrhythmics. Classified B. Lidocaine underwhich is Procainamide Quinidine and C. Flecainide Disopyramide. D. Propanolol E. Dofetilide A patient was diagnosed with familial combined Page 29 of Topnotch Handout. A synergistic hypercholesterolemia, which of the following combination for patients with familial combined synergistic drug combination will be most suitable for hypercholesterolemia (overproduction of VLDL) are this patient: niacin+resin and statin+fibrate. Fibrate nd resin A. Niacin + Statin combinations are disadvantageous because they B. Statin + Ezetimibe confer an increased risk of cholelithiasis. while C. Niacin + Resin Statin and resin combinations cause impaired statin D. Statin + Resin absorption. Niacin and statin combinations are more E. Fibrate + Resin for familial hypercholesterolemiawhere the defect is in the LDL receptors hence increased LDL. This prostaglandin F2 alpha analog is commonly Page 32 of Topnotch Handout. Latanoprost is used off label for its side effect which is lengthening commonly used off label for eyelash lengthening. It of eyelashes: A. Beraprost is a prostaglandin F2alpha analog that activates FP B. Dinoprostone receptors, increases outflow of aquaeous humor and C. Alprostail reduces intraocular pressure. Its other side effects D. Latanoprost are Blurred vision, eye irritation, or tearing E. Gemeprost darkening of eyelid skin color, increase in brown color in colored part of eye. Which of the following statements is true for SIMILAR TO PREVIOUS BOARD EXAM cromolyn: CONCEPT/PRINCIPLE. Page 33 of Topnotch A. It reduces synthesis of leukotrienes Handout. Cromolyn is a mast cell stabilizer by the B. It has no bronchodilator action prevention of calcium influx, therefore preventing C. It binds IgE antibodies on mast cells degranulation and release of histamine, leukotrienes D. It reduces the expression of cyclooxygenase and other mediators. A is moa of zileuton, C is for E. It prevents vagal-stimulated bronchoconstriction omalizumab, D is for corticosteroids while E is for Ipratropium 9muscarinic receptor antagonist. A 23 y/o female previous RHD patient s/p valve Page 39 of Topnotch Handout. CYP 450 inhibitors replacement is being maintained on warfarin. 3 days prolong/increase warfarin's anticoagulant prior she had symptoms of PUD for which she self effectsince they inhibit drug clearance. Cimetidine is medicated. She now presented at the ER due to black a CYP450 inhibitor. tarry stools, epigastric pain, coffee ground emesis, gum bleeding and dizziness. Laboratory showed prolonged PT. Which of the following drugs caused a drug interaction with warfarin prolonging its anticoagulant effect? A. Cimetidine B. Omeprazole C. Bismuth D. Sucralfate E. Maalox A 30 y/o male patient developed Page 52 of Topnotch Handout. For methemoglobinemia after receiving prilocaine during methemoglobinemia the antidote is methylene blue. a minor surgery. What should be administered to this patient? A. NAC B. Methylene blue C. EDTA D. Dimercaprol E. Deferoxamine Which nondepolarizing neuromuscular blocker Page 53 of Topnotch Handout. Atracurium is broken undergoes rapid breakdown by Hoffman elimination? down to inactive metabolites by (minor) ester A. Vecuronium hydrolysis and spontaneous Hoffman elimination B. Mivacurium (major pathway) to Laudanosine. C. Atracurium D. Tubcurarine E. Rocuronium

AUTHOR ANGELA PAULINE P. CALIMAGLOYOLA (TOP 8 - FEB 2015 MED BOARDS; TOPNOTCH MD FROM UST)

TOPNOTCH EXAM DIAGNOSTIC EXAM - AUG 2015

ANGELA PAULINE P. CALIMAGLOYOLA (TOP 8 - FEB 2015 MED BOARDS; TOPNOTCH MD FROM UST) ANGELA PAULINE P. CALIMAGLOYOLA (TOP 8 - FEB 2015 MED BOARDS; TOPNOTCH MD FROM UST)

DIAGNOSTIC EXAM - AUG 2015

ANGELA PAULINE P. CALIMAGLOYOLA (TOP 8 - FEB 2015 MED BOARDS; TOPNOTCH MD FROM UST) ANGELA PAULINE P. CALIMAGLOYOLA (TOP 8 - FEB 2015 MED BOARDS; TOPNOTCH MD FROM UST) ANGELA PAULINE P. CALIMAGLOYOLA (TOP 8 - FEB 2015 MED BOARDS; TOPNOTCH MD FROM UST)

DIAGNOSTIC EXAM - AUG 2015

ANGELA PAULINE P. CALIMAGLOYOLA (TOP 8 - FEB 2015 MED BOARDS; TOPNOTCH MD FROM UST)

DIAGNOSTIC EXAM - AUG 2015

ANGELA PAULINE P. CALIMAGLOYOLA (TOP 8 - FEB 2015 MED BOARDS; TOPNOTCH MD FROM UST)

DIAGNOSTIC EXAM - AUG 2015

TOPNOTCH MEDICAL BOARD PREP PHARMACOLOGY SUPEREXAM Page 15 of 83 For inquiries visit www.topnotchboardprep.com.ph or email us at [email protected]

DIAGNOSTIC EXAM - AUG 2015

DIAGNOSTIC EXAM - AUG 2015

DIAGNOSTIC EXAM - AUG 2015

TOPNOTCH MEDICAL BOARD PREP PHARMACOLOGY SUPEREXAM For inquiries visit www.topnotchboardprep.com.ph or email us at [email protected] Item # 121

QUESTION These are agents or drugs that bind to a different receptor, producing an effect opposite to that produced by the drug it is antagonizing: A. Competitive antagonists B. Irreversible antagonists C. Chemical antagonists D. Physiologic antagonists E. Partial agonists

EXPLANATION

AUTHOR

TOPNOTCH EXAM MIDTERM 1 EXAM - AUG 2015

Competitive/Reversible antagonist - bind to receptors in a reversible way without activating the effector system; Non-competitive/Irreversible antagonist - cause downward shift of the DRC; Chemical antagonist - interact directly with the drug being antagonized; PHYSIOLOGIC ANTAGONIST is the answer. Examples are histamine and epinephrine, propanolol and thyroid hormone. SIMILAR TO PREVIOUS BOARD EXAM CONCEPT. Phenobarbital, a barbiturate, is a Cytochrome P450 inducer which will increase clearance of other drugs, thus, decreasing their effects. SIMILAR TO PREVIOUS BOARD EXAM CONCEPT.

LYNN DARYL FELICIANO VILLAMATER, MD (TOP 5 - FEB 2015 MED BOARDS; TOPNOTCH MD FROM EAC) LYNN DARYL FELICIANO VILLAMATER, MD (TOP 5 - FEB 2015 MED BOARDS; TOPNOTCH MD FROM EAC)

MIDTERM 1 EXAM - AUG 2015

LYNN DARYL FELICIANO VILLAMATER, MD (TOP 5 - FEB 2015 MED BOARDS; TOPNOTCH MD FROM EAC) LYNN DARYL FELICIANO VILLAMATER, MD (TOP 5 - FEB 2015 MED BOARDS; TOPNOTCH MD FROM EAC) LYNN DARYL FELICIANO VILLAMATER, MD (TOP 5 - FEB 2015 MED BOARDS; TOPNOTCH MD FROM EAC) LYNN DARYL FELICIANO VILLAMATER, MD (TOP 5 - FEB 2015 MED BOARDS; TOPNOTCH MD FROM EAC) LYNN DARYL FELICIANO VILLAMATER, MD (TOP 5 - FEB 2015 MED BOARDS; TOPNOTCH MD FROM EAC)

MIDTERM 1 EXAM - AUG 2015

122

The patient is being maintained on oral anticoagulant. Which of the following if taken by the patient will decrease the effect of the drug he is presently taking? A. Phenobarbital B. Sulfamethoxazole C. Valproic acid D. Ketoconazole E. Vancomycin

123

Which of the following is a mechanism of action of Hydralazine? A. Alters intracellular calcium B. Opens potassium channel causing arteriolar and venular dilatation C. Blocks alpha-1 adrenergic receptor D. Competitively bloocks Nn nicotinic receptor E. Blocks voltage-gated L-type calcium channels The drug of choice for paroxysmal Supraventricular tachycardia is: A. Procainamide B. Verapamil C. Amiodarone D. Lidocaine E. Adenosine

Hydralazine binds to and activates potassium channels on vascular smooth muscle resulting to efflux of potassium and subsequent hyperpolarization of the cell. This prevents calciummediated activation and constriction of smooth muscle, resulting in vasodilation. It dilates arterioles, but not veins. SIMILAR TO PREVIOUS BOARD EXAM CONCEPT. SIMILAR TO PREVIOUS BOARD EXAM CONCEPT.

125

The most common anti-arrythmic drug used for cardiac arrest is: A. Lidocaine B. Adenosine C. Flecainide D. Sotalol E. Amiodarone

Lidocaine is the drug of choice for ventricular arrythmias post-MI, and digoxin-induced arrythmias. It is the least cardiotoxic among conventional anti-arrythmic. SIMILAR TO PREVIOUS BOARD EXAM CONCEPT..

126

Which of the following diuretics may cause hyperkalemia? A. Amiloride B. Ethacrynic acid C. Indapamide D. Mannitol E. Acetozalamide

Amiloride (also Triamterene, Eplerenone, Spironolactone) is a potassium sparing diuretics. All other options are potassium-wasting. SIMILAR TO PREVIOUS BOARD EXAM CONCEPT.

127

Which of the following anesthetic agent has analgesic property and used as an adjunct to other anesthetics? A. Halothane B. Nitric oxide C. Etomidate D. Desflurane E. Fentanyl

Nitrous oxide , not nitric oxide, is used as an anesthetic agent with analgesic property. Fentanyl is often used for anesthesia and analgesia. SIMILAR TO PREVIOUS BOARD EXAM CONCEPT.

128

A 50-year old male with DM type II had been on metformin for nine years and presented with diarrhea, paresthesia, and muscular weakness. On examination, he has hyperactive DTRs. Blood electrolytes showed serum Na 138 mEq/L, Serum K= 3.4 mEq/L, serum Ca=8.2 mg/dL and Mg 0.8 mEq/L. Which of the following medications should you avoid to prevent exacerbation of his symptoms? A. Spironolactone B. Magnesium sulfate C. Furosemide D. Indomethacin E. Acetazolamide Dicycloverine is used for: A. Paralytic ileus B. Biliary colic C. Intestinal spasm D. Constipation E. Urinary retention

This is a case of hypomagnesemia, which can be exacerbated by administration of a loop diuretic Furosemide is a loop diuretic that acts on the thick ascending limb of the loop of henle. This tubular segment is responsible for a significant sodium chloride reabsorption, as well as the site for calcium and magnesium reabsorption. Side effects of Furosemide : Hypokalemia, hypocalcemia, hypomagnesemia, hyperuricemia, dehydration, metabolic alkalosis, ototoxicity, sulfa allergy, nephritis

LYNN DARYL FELICIANO VILLAMATER, MD (TOP 5 - FEB 2015 MED BOARDS; TOPNOTCH MD FROM EAC)

MIDTERM 1 EXAM - AUG 2015

Dicycloverine is an antispasmodic, prescribed for gastrointestinal tract spasm and irritable bowel syndrome. It blocks the activity of acetylcholine on muscarinic receptors. It should not be used in patients with PARALYTIC ILEUS, myasthenia gravis, narrow angle glaucoma, enlarged prostate, or pyloric stenosis. Constipation is a side-effect of this drug. SIMILAR TO PREVIOUS BOARD EXAM CONCEPT.

LYNN DARYL FELICIANO VILLAMATER, MD (TOP 5 - FEB 2015 MED BOARDS; TOPNOTCH MD FROM EAC)

MIDTERM 1 EXAM - AUG 2015

124

129

TOPNOTCH MEDICAL BOARD PREP PHARMACOLOGY SUPEREXAM Page 16 of 83 For inquiries visit www.topnotchboardprep.com.ph or email us at [email protected]

MIDTERM 1 EXAM - AUG 2015

MIDTERM 1 EXAM - AUG 2015

MIDTERM 1 EXAM - AUG 2015

MIDTERM 1 EXAM - AUG 2015

TOPNOTCH MEDICAL BOARD PREP PHARMACOLOGY SUPEREXAM For inquiries visit www.topnotchboardprep.com.ph or email us at [email protected] Item # 130

QUESTION Which of the following is NOT an effect of Naloxone? A. It will cause respiratory depression B. It may cause nausea and vomiting C. It induces abstinence syndrome D. It decreases constipation E. None of the above

131

The combination of metronidazole and alcohol will most likely cause? A. Ataxia B. Blurring of vision C. Dizziness D. Nausea and vomiting E. Pancreatitis

132

The mechanism of action of Nedocromil: A. Activates beta-2 receptors in bronchial smooth muscle B. Prevents calcium influx and stabilizes mast cells preventing release of histamine C. Inhibits 5-lipoxygenase D. Prevents vagal-stimulated bronchoconstriction E. Blocks cysteinyl leukotriene-1 receptor

133

The mechanism of action of Allopurinol : A. Inhibits microtubule assembly B. Decreases macrophage migration and phagocytosis C. Compete with uric acid for reabsorption in the proximal tubules D. Irreversibly inhibits xanthine oxidase E. Increases uric acid excretion The preferred antiseizure drug for pregnant women: A. Valproic acid B. Phenobarbital C. Phenytoin D. Carbamazepine E. Topiramate

134

EXPLANATION

AUTHOR

Naloxone is an opioid antagonist used for opioid overdose. It competitively blocks opioid receptors and rapidly reverses effects of opioid agonists. In individuals who are acutely depressed by an overdose of an opioid, the antagonist effectively normalizes respiration, LOC, pupil size, bowel activity, and awareness of pain. In dependent subjects who appear normal while taking opioids, naloxone or naltrexone almost instantaneously precipitates an abstinence syndrome. SIMILAR TO PREVIOUS BOARD EXAM CONCEPT Metronidazole has a disulfiram-effect, so that nausea and vomiting can occur if alcohol is ingested during therapy. (Katzung) SIMILAR TO PREVIOUS BOARD EXAM CONCEPT.

LYNN DARYL FELICIANO VILLAMATER, MD (TOP 5 - FEB 2015 MED BOARDS; TOPNOTCH MD FROM EAC)

Option A - Beta 2 agonist (Salbutamol, Terbutaline); Option C- leukotriene synthesis inhibitor (Zileuton); Option D - Muscarinic receptor antagonist (Ipratropium); Option E - Leukotriene antagonist (Zafirlukast). Cromolyn, Nedocromil, and Lodoxamide are mast cell stabilizers, which acts by preventing calcium influx and stabilizes mast cells, preventing degranulation and release of histamine, leukotrienes, and other mediators. SIMILAR TO PREVIOUS BOARD EXAM CONCEPT. Option A and B - MOA of Colchicine; Option C and E - MOA of Probenecid; MOA of Allopurinol: its active metabolite irreversibly inhibits xanthine oxidase and lowers production of uric acid. SIMILAR TO PREVIOUS BOARD EXAM CONCEPT.

Phenobarbital is the preferred antiseizure drug in children and pregnant women. SIMILAR TO PREVIOUS BOARD EXAM CONCEPT.

135

Which of the following is an extraluminal amoebicide? A. Metronidazole B. Diloxanide furoate C. Lumefrantrine D. Pentamidine E. Nifurtimox

Tissue amebicides act in the bowel wall and liver: Metronidazole, emetine, chloroquine, tinidazole; Luminal amebicide: Diloxanide furoate, Iodoquinol, Paromomycin. SIMILAR TO PREVIOUS BOARD EXAM CONCEPT

136

Sulfone in the management of leprosy causes? A. Inhibition of folic acid synthesis B. Inhibition of PABA C. Inhibition of DNA synthesis D. Inhibition of protein synthesis E. Inhibition of arabinosyl transferase

Drugs used in leprosy: Dapsone, Rifampicin, and Clofazimine. Dapsone, a sulfone, is the most active drug used against M. leprae. It is bacteriostatic and inhibits folic acid synthesis. Clofazimine acts by binding to guanine bases in bacterial DNA.

137

Which of the following antiretroviral would cause hyperlipidemia and hyperglycemia? A. Enfuvitide B. Indinavir C. Zidovudine D. Delavirdine E. Didanosine

138

Which of the following sulfonylurea is safest drug for elderly diabetics? A. Chlorpropramide B. Tolbutamide C. Tolazamide D. Glyburide E. Glimepiride

Indinavir (Protease Inhibitor) - all Pis/'navirs' may cause hyperlipidemia, hyperglycemia, and insulin resistance as potential adverse events; Efuvirtide is a fusion inhibitor, SE: Hypersensitivity, increased incidence of bacterial pneumonia, injection site reaction; Zidovudine and Didanosine (NRTI) - all NRTI carry the risk of lactic acidosis with hepatic steatosis Because of its short half-life, Tolbutamide is the safest sulfonylurea for elderly diabetics. SIMILAR TO PREVIOUS BOARD EXAM CONCEPT.

139

The danger of long term use of propylthiouracil is: A. Cholestatic jaundice B. Exfoliative dermatitis C. Liver failure D. Agranulocytosis E. Arthralgia

The most dangerous complication of thioamides is agranulocytosis, (granulocyte <500) an infrequent but potentially fatal adverse reaction. Hepatitis can also be fatal. The most common adverse effect of PTU is maculopapular rash. (Katzung). SIMILAR TO PREVIOUS BOARD EXAM CONCEPT.

TOPNOTCH EXAM MIDTERM 1 EXAM - AUG 2015

LYNN DARYL FELICIANO VILLAMATER, MD (TOP 5 - FEB 2015 MED BOARDS; TOPNOTCH MD FROM EAC) LYNN DARYL FELICIANO VILLAMATER, MD (TOP 5 - FEB 2015 MED BOARDS; TOPNOTCH MD FROM EAC)

MIDTERM 1 EXAM - AUG 2015

LYNN DARYL FELICIANO VILLAMATER, MD (TOP 5 - FEB 2015 MED BOARDS; TOPNOTCH MD FROM EAC) LYNN DARYL FELICIANO VILLAMATER, MD (TOP 5 - FEB 2015 MED BOARDS; TOPNOTCH MD FROM EAC) LYNN DARYL FELICIANO VILLAMATER, MD (TOP 5 - FEB 2015 MED BOARDS; TOPNOTCH MD FROM EAC) LYNN DARYL FELICIANO VILLAMATER, MD (TOP 5 - FEB 2015 MED BOARDS; TOPNOTCH MD FROM EAC) LYNN DARYL FELICIANO VILLAMATER, MD (TOP 5 - FEB 2015 MED BOARDS; TOPNOTCH MD FROM EAC) LYNN DARYL FELICIANO VILLAMATER, MD (TOP 5 - FEB 2015 MED BOARDS; TOPNOTCH MD FROM EAC) LYNN DARYL FELICIANO VILLAMATER, MD (TOP 5 - FEB 2015 MED BOARDS; TOPNOTCH MD FROM EAC)

MIDTERM 1 EXAM - AUG 2015

TOPNOTCH MEDICAL BOARD PREP PHARMACOLOGY SUPEREXAM Page 17 of 83 For inquiries visit www.topnotchboardprep.com.ph or email us at [email protected]

MIDTERM 1 EXAM - AUG 2015

MIDTERM 1 EXAM - AUG 2015

MIDTERM 1 EXAM - AUG 2015

MIDTERM 1 EXAM - AUG 2015

MIDTERM 1 EXAM - AUG 2015

MIDTERM 1 EXAM - AUG 2015

MIDTERM 1 EXAM - AUG 2015

TOPNOTCH MEDICAL BOARD PREP PHARMACOLOGY SUPEREXAM For inquiries visit www.topnotchboardprep.com.ph or email us at [email protected] Item # 140

QUESTION

EXPLANATION

AUTHOR

This cell-cycle specific drug is commonly used for testicular cancer. It does not cause myelosuppression, however, dose modification is recommended in the setting of renal dysfunction. What is this chemotherapeutic drug? A. Cyclophosphamide B. Cisplatin C. Busulfan D. Bleomycin E. Etoposide



LYNN DARYL FELICIANO VILLAMATER, MD (TOP 5 - FEB 2015 MED BOARDS; TOPNOTCH MD FROM EAC)

141

You are treating a patient with hyperthyroidism and you prescribed methimazole, you know that this drug inhibits which enzyme? A. Sodium-Iodide co-transporter B. Thyroid peroxidase C. Proteases in the thyroid gland D. All of the above E. None of the above

Thioamides like methimazole and PTU inhibit the organification of iodine to tyrosine via the enzyme thyroid peroxidase.

142

Which of the following anti fungals will enhance the activity of cyclosporine? A. Itraconazole B. Fluconazole C. Ketoconazole D. Griseofulvin E. Terbinafine

Ketoconazole is a potent cytochrome P450 inhibitor and therefore inhibits the metabolism of other drugs and increases their effects, griseofulvin is a cytochrome inducer and thus facilitates the metabolism of other drugs thus decreasing their effect.

143

Which among the following is the most potent opioid A. Ohmefentanyl B. Morphine C. Fentanyl D. Pethidine E. Remifentanil

The opioid ohmefentanyl is the most potent opioid

144

What is the drug of choice in acute arrhythmias A. Metoprolol B. Lidocaine C. Quinine D. Bisoprolol E. None of the above



145

What is the drug of choice in preventing the recurrence of arryhthmia A. Adenosine B. Procainamide C. Lidocaine D. Carvedilol E. None of the above



146

The following antibiotics eliminate intestinal flora thus causing pseudomembranous colitis except A. 3rd gen cephalosporins B. Aminoglycosides C. Clindamycin D. Erythromycin E. None of the above

Erythromycin is not associated with pseudomembranous colitis

147

Why is nitroglycerin preferred given sublingually? A. Giving it orally will require a higher dose B. Giving it orally will produce a greater effect C. It D. Protect it from first-pass metabolism E. None of the above

Sublingual route is preferred as there is less dose needed and it is protected from hepatic first-pass metabolism

148

RH, a 50 year old bank executive was brought to the ER due to loss of consciousness, upon PE you noted his breath smelling like sweet with garlicky odor, what is the appropriate drug for this case A. Dimercaprol B. EDTA C. Hydration D. Deferoxamine E. Flumazenil

The patient has been poisoned with arsenic and the drug of choice is dimercaprol

EDWARD HARRY VALLAJERA, MD (TOP 8 - FEB 2015 MED BOARDS; TOPNOTCH MD FROM PERPETUAL BINAN) EDWARD HARRY VALLAJERA, MD (TOP 8 - FEB 2015 MED BOARDS; TOPNOTCH MD FROM PERPETUAL BINAN) EDWARD HARRY VALLAJERA, MD (TOP 8 - FEB 2015 MED BOARDS; TOPNOTCH MD FROM PERPETUAL BINAN) EDWARD HARRY VALLAJERA, MD (TOP 8 - FEB 2015 MED BOARDS; TOPNOTCH MD FROM PERPETUAL BINAN) EDWARD HARRY VALLAJERA, MD (TOP 8 - FEB 2015 MED BOARDS; TOPNOTCH MD FROM PERPETUAL BINAN) EDWARD HARRY VALLAJERA, MD (TOP 8 - FEB 2015 MED BOARDS; TOPNOTCH MD FROM PERPETUAL BINAN) EDWARD HARRY VALLAJERA, MD (TOP 8 - FEB 2015 MED BOARDS; TOPNOTCH MD FROM PERPETUAL BINAN) EDWARD HARRY VALLAJERA, MD (TOP 8 - FEB 2015 MED BOARDS; TOPNOTCH MD FROM PERPETUAL BINAN)

TOPNOTCH MEDICAL BOARD PREP PHARMACOLOGY SUPEREXAM Page 18 of 83 For inquiries visit www.topnotchboardprep.com.ph or email us at [email protected]

TOPNOTCH EXAM MIDTERM 1 EXAM - AUG 2015

MIDTERM 2 EXAM - AUG 2015

MIDTERM 2 EXAM - AUG 2015

MIDTERM 2 EXAM - AUG 2015

MIDTERM 2 EXAM - AUG 2015

MIDTERM 2 EXAM - AUG 2015

MIDTERM 2 EXAM - AUG 2015

MIDTERM 2 EXAM - AUG 2015

MIDTERM 2 EXAM - AUG 2015

TOPNOTCH MEDICAL BOARD PREP PHARMACOLOGY SUPEREXAM For inquiries visit www.topnotchboardprep.com.ph or email us at [email protected] Item # 149

QUESTION

EXPLANATION

RJLC, a 30 year old female was treated for extraintestinal amebiasis, what is the drug of choice to eliminate the extraintestinal parasites? A. Paromomycin B. Diloxanide furoate C. Metronidazole D. Pyrantel pamoate E. Praziquantel

Metronidazole eliminates the trophozoite forms of the parasite in the intestine or in the tissues.

150

Allopurinol exerts its theraputic effect by: A. Inhibiting excretion of uric acid at the PCT B. Increasing purine catabolism C. Competes with uric acid for excretion in the kidney D. Inhibiting formation of uric acid E. None of the above

Allopurinol inhibits the enzyme xanthine oxidase which metabolizes purines to form uric acid. Therefore its activity inhibits the formation of uric acid

151

This drug reduces the relapse rates for malaria A. Quinine B. Arthemeter-lumefantrine C. Mefloquine D. Doxycycline E. Primaquine

Primaquine eradicates hypnozoites in the liver responsible for causing relapse

152

Chronic administration of phenobarbital causes metabolic tolerance via which mechanism? A. Stimulation of aldehyde dehydrogenase B. Stimulation of alcohol dehydrogenase C. Stimulation of MEOS D. Stimulation of Monoamine oxidase E. None of the above

Stimulation of the microsomal ethanol oxidizing system is implicated in the development of tolerance to phenobarbital aside from playing a minor role in individuals with chronic alcoholism. The rest have no role in the development of tolerance to phenobarbital

153

Most common side effect of clomiphene citrate A. Hot flushes B. Constipation C. Headache D. Allergic skin reactions E. None of the above

Hot flushes are the most common side effect of use of clomiphene citrate, the rest are occasional side effects

154

Which of the following antimalarial drugs may precipitate a hemolytic crisis in a patient with G6PD A. Lumefantrine B. Doxycycline C. Artemether D. Quinine E. None of the above

Quinine is precipitates hemolytic crisis in patients with malaria with G6PD deficiency.

155

A man is being treated for intestinal strongyloidiasis, you know that the drug of choice is: A. Praziquantel B. Ivermectin C. Melarsoprol D. Pyrimethamine-sulfa E. Trimethoprim-sulfamethoxazole

Ivermectin is the DOC in the treatment of strongyloidiasis

156

SMA, a 68 year old female with hypertension is taking captopril, you know that it is effective in hypertension because it inhibits: A. Cathepsin C B. Dipeptidyl dipeptidase-2 C. Dipeptidyl dipeptidase-4 D. Peptidyl dipeptidase E. Procarboxypeptidase

Peptidyl dipeptidase is otherwise known as ACE which is the enzyme inhibited by ACE inhibitors like captopril.

157

Intake of potassium supplements is contraindicated in patients taking which of the following? A. Ethacrynic acid B. Amiloride C. Hydrochlorothiazide D. Acetazolamide E. None of the above

Amiloride is a potassium sparing diuretic which therefore spares renal excretion of potassium raising the potassium levels and therefore is contraindicated in patients taking potassium supplements.

AUTHOR EDWARD HARRY VALLAJERA, MD (TOP 8 - FEB 2015 MED BOARDS; TOPNOTCH MD FROM PERPETUAL BINAN) EDWARD HARRY VALLAJERA, MD (TOP 8 - FEB 2015 MED BOARDS; TOPNOTCH MD FROM PERPETUAL BINAN) EDWARD HARRY VALLAJERA, MD (TOP 8 - FEB 2015 MED BOARDS; TOPNOTCH MD FROM PERPETUAL BINAN) EDWARD HARRY VALLAJERA, MD (TOP 8 - FEB 2015 MED BOARDS; TOPNOTCH MD FROM PERPETUAL BINAN) EDWARD HARRY VALLAJERA, MD (TOP 8 - FEB 2015 MED BOARDS; TOPNOTCH MD FROM PERPETUAL BINAN) EDWARD HARRY VALLAJERA, MD (TOP 8 - FEB 2015 MED BOARDS; TOPNOTCH MD FROM PERPETUAL BINAN) EDWARD HARRY VALLAJERA, MD (TOP 8 - FEB 2015 MED BOARDS; TOPNOTCH MD FROM PERPETUAL BINAN) EDWARD HARRY VALLAJERA, MD (TOP 8 - FEB 2015 MED BOARDS; TOPNOTCH MD FROM PERPETUAL BINAN) EDWARD HARRY VALLAJERA, MD (TOP 8 - FEB 2015 MED BOARDS; TOPNOTCH MD FROM PERPETUAL BINAN)

TOPNOTCH MEDICAL BOARD PREP PHARMACOLOGY SUPEREXAM Page 19 of 83 For inquiries visit www.topnotchboardprep.com.ph or email us at [email protected]

TOPNOTCH EXAM MIDTERM 2 EXAM - AUG 2015

MIDTERM 2 EXAM - AUG 2015

MIDTERM 2 EXAM - AUG 2015

MIDTERM 2 EXAM - AUG 2015

MIDTERM 2 EXAM - AUG 2015

MIDTERM 2 EXAM - AUG 2015

MIDTERM 2 EXAM - AUG 2015

MIDTERM 2 EXAM - AUG 2015

MIDTERM 2 EXAM - AUG 2015

TOPNOTCH MEDICAL BOARD PREP PHARMACOLOGY SUPEREXAM For inquiries visit www.topnotchboardprep.com.ph or email us at [email protected] Item # 158

EXPLANATION

AUTHOR

Injection of a small dose of Ach will cause what? A. Reflex bradycardia B. Hypotension C. Splanchnic stimulation D. All of the above E. None of the above

Ach causes depression of the SA and AV nodes thus slowing the heart rate, hypotension through the relaxation of arterioles, splanchnic stimulation through the stimulation of the muscarinic receptors

159

Though 1% is systemically absorbed when Ipratropium Bromide is given via nebulization, what is its net effect? A. Reflex tachycardia B. Bronchodilation C. Bronchoconstriction D. Laryngeal spasm E. None of the above

Bronchodilation is still the net effect of ipratropium, the systemic effect is negligent as the drug is administered to its site of action which is the lungs.

160

KVM, a 29 year old female was brought to the ER due to ingestion of insecticide, you ordered pralidoxime to be given, what does it do? A. Induces the enzyme Acetylcholinesterase B. Regenerates acetylcholinesterase if given within 68 hours after ingestion C. Breaks the bond between organophosphate and acetylcholinesterase D. B and C only E. All of the above A patient came in for consult due to infertility. It was found out that the patient is having ovulation disorder so you decided to give here clomiphene. It is important to tell to the patient that the most common adverse effect of clomiphene is: A. hot flushes B. bleeding C. constipation D. headache E. nausea and vomiting

Pralidoxime regenerates acetylcholinesterase and is only effective when the bond between the organophosphate and acetylcholinesterase hasn't matured which takes 6-8 hours, afterwhich, pralidoxime even if given is no longer effective.

EDWARD HARRY VALLAJERA, MD (TOP 8 - FEB 2015 MED BOARDS; TOPNOTCH MD FROM PERPETUAL BINAN) EDWARD HARRY VALLAJERA, MD (TOP 8 - FEB 2015 MED BOARDS; TOPNOTCH MD FROM PERPETUAL BINAN) EDWARD HARRY VALLAJERA, MD (TOP 8 - FEB 2015 MED BOARDS; TOPNOTCH MD FROM PERPETUAL BINAN)

SIMILAR TO PREVIOUS BOARD EXAM CONCEPT/PRINCIPLE. Katzung 11th edition page 719.. "the most common adverse effects in patients treated with this drug are hot flushes, which resemle those experienced by menopausal patients."

HAROLD JAY S. BAYTEC, MD (TOP 10 - FEB 2015 MED BOARDS; TOPNOTCH MD FROM FEU)

MIDTERM 3 EXAM - AUG 2015

162

It is the antiarrythmic of choice for termination of ventricular tachycardia and prevention of ventricular fibrillation after cardioversion in the setting of acute ischemia. A. adenosine B. esmolol C. lidocainne D. procainamide E. Amiodarone

HAROLD JAY S. BAYTEC, MD (TOP 10 - FEB 2015 MED BOARDS; TOPNOTCH MD FROM FEU)

MIDTERM 3 EXAM - AUG 2015

163

Jan Deo is a newly diagnosed with essential hypertension. He has no other co morbidities. Aside from lifestyle modification, he was prescribed with Enalapril as his maintainance. Which of the following BEST describes the mechanism of action of the drug. A. blocks Angiotensin receptor which will prevent rise in BP B. inhibits peptidyl dipeptidase C. direct vasodilator D. reduces calcium uptake of smooth muscles E. none of the above A patient was given Nedocromil for asthma. Which of the following is the mechanism of action of the drug? A. Inhibits the release of histamine B. Direct relaxation of smooth muscle C. Inhibits leukotriene pathway D. mast cell stabilizer E. none of the above

SIMILAR TO PREVIOUS BOARD EXAM CONCEPT/PRINCIPLE. Katzung 11th edition page 239. "Lidocaine is the agent of choice for termination of ventricular tachycardia and prevention of ventricular fibrillation after cardioversion the setting of acute ischemia. However, routine prophylactic use of lidocaine in this setting may actually increase total mortality, possibly by increasing incidence of asystole, and is not the standard of care. Most physicians administer IV lidocaine only to patients with arrythmia" Enalapril is an ACE inhibitor. One of the many alternative names for ACE is peptidyl dipeptidase. SIMILAR TO PREVIOUS BOARD EXAM CONCEPT/PRINCIPLE

HAROLD JAY S. BAYTEC, MD (TOP 10 - FEB 2015 MED BOARDS; TOPNOTCH MD FROM FEU)

MIDTERM 3 EXAM - AUG 2015

SIMILAR TO PREVIOUS BOARD EXAM CONCEPT/PRINCIPLE. Katzung 11th edition page 349

HAROLD JAY S. BAYTEC, MD (TOP 10 - FEB 2015 MED BOARDS; TOPNOTCH MD FROM FEU)

MIDTERM 3 EXAM - AUG 2015

SIMILAR TO PREVIOUS BOARD EXAM CONCEPT/PRINCIPLE. Katzung 11th edition page 672. PTU is preferable because it is more strongly protein-bound and therefore crosses the placcenta less readily.

HAROLD JAY S. BAYTEC, MD (TOP 10 - FEB 2015 MED BOARDS; TOPNOTCH MD FROM FEU)

MIDTERM 3 EXAM - AUG 2015

SIMILAR TO PREVIOUS BOARD EXAM CONCEPT/PRINCIPLE. Katzung 11th edition page 672. All of the choices are mechanisms of action of PTU except for the blocking of uptake of iodide by the gland. Inhibition of thyroid peroxidase-catalyzed reaction and the blocking of iodine organification.

HAROLD JAY S. BAYTEC, MD (TOP 10 - FEB 2015 MED BOARDS; TOPNOTCH MD FROM FEU)

MIDTERM 3 EXAM - AUG 2015

161

164

165

166

QUESTION

Among the thioamide antithyroid drugs, which of the following is the one preferred for pregnant women because it crosses the placental barrier less readily giving lesser effect to the fetus A. potassium iodide B. Ipodate C. carbimazole D. methimazole E. PTU All of the following are mechanism of action of propylthiouracil EXCEPT: A. inhibits thyroid peroxidase-catalyzed reactions B. Blocks iodine organification C. Blocks coupling of the iodotyrosinase D. Block uptake of iodide by the gland E. inhibits peripheral deiodination

TOPNOTCH MEDICAL BOARD PREP PHARMACOLOGY SUPEREXAM Page 20 of 83 For inquiries visit www.topnotchboardprep.com.ph or email us at [email protected]

TOPNOTCH EXAM MIDTERM 2 EXAM - AUG 2015

MIDTERM 2 EXAM - AUG 2015

MIDTERM 2 EXAM - AUG 2015

TOPNOTCH MEDICAL BOARD PREP PHARMACOLOGY SUPEREXAM For inquiries visit www.topnotchboardprep.com.ph or email us at [email protected] Item # 167

QUESTION

EXPLANATION

Fluoxetine (prozac) is one of the most commonly use antidepressants worldwide. Which of the following is the mechanism of action of the drug? A. Selectively inhibits serotonin reuptake B. Inhibits serotonin and norepinephrine reuptake C. selectively inhibits reuptake of norepinephrine D. directly antagonize serotonin E. directly antagonize epinephrine

SIMILAR TO PREVIOUS BOARD EXAM CONCEPT/PRINCIPLE. Katzung 11th edition page 513. SSRIs like Fluoxetine are agents that have their primary action the inhibition of serotonin transporter.

HAROLD JAY S. BAYTEC, MD (TOP 10 - FEB 2015 MED BOARDS; TOPNOTCH MD FROM FEU)

168

Which among the following drugs deliver its action through selectively inhibiting reuptake of serotonin? A. venlafaxine B. imipramine C. selegiline D. fluoxetine E. trazodone

SIMILAR TO PREVIOUS BOARD EXAM CONCEPT/PRINCIPLE. Katzung 11th edition page 514. Fluoxetine is a SSRI. Venlafaxine is a SNRI. Imipramine is a TCA. Selegiline is MAOi. Trazadone is a serotonin antagonist.

HAROLD JAY S. BAYTEC, MD (TOP 10 - FEB 2015 MED BOARDS; TOPNOTCH MD FROM FEU)

MIDTERM 3 EXAM - AUG 2015

169

All of the following drugs are considered diseasemodifying antirheumatic drugs (DMARDs) except: A. Abatacept B. Azathioprine C. Chloroquine D. Methotrexate E. Ketoprofen

ketoprofen is a non selective COX inhibitor

HAROLD JAY S. BAYTEC, MD (TOP 10 - FEB 2015 MED BOARDS; TOPNOTCH MD FROM FEU)

MIDTERM 3 EXAM - AUG 2015

170

Colchicine is a widely used drug in the treatment of gout. Which among the following describes the mechanism of action of the drug? A. Binding to intracellular tubulin to excert anti inflammatory effect B. It increases the excretion of uric acid through urine C. inhibits xanthine oxidase catalized reactions D. non purine xanthine oxidase inhibitor E. none of the above A patient came in at the ER having severe headache and heat intolerance. Examination revealed enlarged thyroid, high blood pressure and severe tachycardia. On history, his relative told you that he also has frequent attacks of asthma. In your findings, you are suspecting that he is having thyroid storm. Which of the following drugs can be given immediately to relieve hypertension and tachycardia? A. propanolol B. esmolol C. diltiazem D. nifedipine E. clonidine A post stroke patient is taking in warfarin. Which among the following drugs will most likely increase the possibility of having warfarin toxicity when added in his management? A. Smoking B. Barbiturates C. Carbamazepine D. Isoniazid E. Ethanol A G7P7 (6107) came in for consult. She asks you that she wants to take oral contraceptive pills as family planning. If given in conjunction with OCP, which among the following will decrease the effectivity of OCP? A. Isoniazid B. Cimetidine C. ketoconazole D. erythromycin E. Rifampicin

B. are uricosuric agents like probenecid and sulfinpyrazone… C. allopurinol….D. Febuxostat SIMILAR TO PREVIOUS BOARD EXAM CONCEPT/PRINCIPLE

HAROLD JAY S. BAYTEC, MD (TOP 10 - FEB 2015 MED BOARDS; TOPNOTCH MD FROM FEU)

MIDTERM 3 EXAM - AUG 2015

Take note, the patient has asthma… Katzung 11th edition chapter 38 page 677. “If propanolol is contraindicated by the presence of severe heart failure or asthma, hypertension and tachycardia may be controlled with diltiazem, 90-120mg orally three or four times daily or 5-10 mg/h by IV infusion.” SIMILAR TO PREVIOUS BOARD EXAM CONCEPT/PRINCIPLE. Katzung 11th edition page 677

HAROLD JAY S. BAYTEC, MD (TOP 10 - FEB 2015 MED BOARDS; TOPNOTCH MD FROM FEU)

MIDTERM 3 EXAM - AUG 2015

Isoniazid in a P450 inhibitor which will delay the excretion of warfarin when given. Other choices are P450 inducers

HAROLD JAY S. BAYTEC, MD (TOP 10 - FEB 2015 MED BOARDS; TOPNOTCH MD FROM FEU)

MIDTERM 3 EXAM - AUG 2015

Rifampicin is a CYP450 inducer which hastens the excretion of OCP

HAROLD JAY S. BAYTEC, MD (TOP 10 - FEB 2015 MED BOARDS; TOPNOTCH MD FROM FEU)

MIDTERM 3 EXAM - AUG 2015

174

This drug inhibits intestinal absorption of phytosterol and cholesterol which will eventually decrease the level of LDL. A. rosuvastatin B. ezetimibe C. Niacin D. Gemfibrozil E. Simvastatin



HAROLD JAY S. BAYTEC, MD (TOP 10 - FEB 2015 MED BOARDS; TOPNOTCH MD FROM FEU)

MIDTERM 3 EXAM - AUG 2015

175

All of the following reactions are considered Phase 2 reactions in drug metabolism except: A. glucoronidation B. hydoxylation C. Glutathione conjugation D. sulfation E. Methylation

other choices are phase 1 reaction

HAROLD JAY S. BAYTEC, MD (TOP 10 - FEB 2015 MED BOARDS; TOPNOTCH MD FROM FEU)

MIDTERM 3 EXAM - AUG 2015

171

172

173

AUTHOR

TOPNOTCH MEDICAL BOARD PREP PHARMACOLOGY SUPEREXAM Page 21 of 83 For inquiries visit www.topnotchboardprep.com.ph or email us at [email protected]

TOPNOTCH EXAM MIDTERM 3 EXAM - AUG 2015

TOPNOTCH MEDICAL BOARD PREP PHARMACOLOGY SUPEREXAM For inquiries visit www.topnotchboardprep.com.ph or email us at [email protected] Item # 176

QUESTION

EXPLANATION

Which of the following correctly describes FDA Drug category B? A. Either animal studies revealed adverse effects on the fetus and there are no controlled studies in women or studies in women and animals are not available B. Control studies in women fail to demonstrate a risk to the fetus in the first trimester and the possibility of fetal harm is remote C. there is positive evidence of human fetal risk but the benefits from use in pregnant women may be acceptable despite the risk D. Animal reproduction studies have shown an adverse effect that was not confirmed in controlled studies in women in the first trimester E. None of the above Which of the following antineoplastic drugs can cause hemorrhagic cystitis? A. cyclophosphamide B. cisplatin C. vincristine D. methotrexate E. Bleomycin

A: category C------ B. Category A-------- C. Category D

HAROLD JAY S. BAYTEC, MD (TOP 10 - FEB 2015 MED BOARDS; TOPNOTCH MD FROM FEU)

cisplatin are nephrotoxic and ototoxic. Vincristine causes peripheral neuropathy. Methotrexate usually causes myelosuppression. Bleomycin is usually associated with pulmonary fibrosis

HAROLD JAY S. BAYTEC, MD (TOP 10 - FEB 2015 MED BOARDS; TOPNOTCH MD FROM FEU)

MIDTERM 3 EXAM - AUG 2015

178

A cancer patient is scheduled for methotrexate chemotherapy. Which of the following rescue agent/s is used to alleviate the toxic effect of the drug? A. Dexrazoxane B. Mesna C. Leucovorin D. Amifostine E. None of the above

dexrazoxane is a rescue therapy for doxorubicin; mesna is for cyclophosphamide; amifostine is for cisplatin

HAROLD JAY S. BAYTEC, MD (TOP 10 - FEB 2015 MED BOARDS; TOPNOTCH MD FROM FEU)

MIDTERM 3 EXAM - AUG 2015

179

Which of the following antibiotics is a protein synthesis inhibitor at 30S sub unit? A. Ciprofloxacin B. azithromycin C. clindamycin D. linezolid E. Doxycycline

Ciprofloxacin is a fluoroquinolone which inhibits topoisomerase. The other choices inhibits at 50s subunit.

HAROLD JAY S. BAYTEC, MD (TOP 10 - FEB 2015 MED BOARDS; TOPNOTCH MD FROM FEU)

MIDTERM 3 EXAM - AUG 2015

180

Which of the following cephalosporins can be used to patients with Pseudomonas infection? A. cefuroxime B. cefazolin C. cefoxitin D. Ceftazidime E. Ceftriaxone

antipseudomonal cephalosporins are ceftazidime, cefepime, and cefoperazone

HAROLD JAY S. BAYTEC, MD (TOP 10 - FEB 2015 MED BOARDS; TOPNOTCH MD FROM FEU)

MIDTERM 3 EXAM - AUG 2015

181

What is the mechanism of action of metronidazole? A. Inhibits DNA replication by binding to DNA gyrase B. Reactive reduction by ferredoxin forming free radicals C. Forms multiple reactive intermediates when acted upon by bacterial enzyme D. Unknown E. None of the above 59 year old male, hypertensive for 22 years, came in the ER due to difficulty of breathing. PE revealed crackles on bilateral lower lobes of the lungs and bipedal edema. Which of the drugs used in CHF can prolong survival of patients? A. carvedilol B. atenolol C. digitalis D. furosemide E. A and B

A- fluoroquinolones. C- nitrofurantoin.

JEAN PAOLO M. DELFINO, MD (TOP 10 - FEB 2015 MED BOARDS; TOPNOTCH MD FROM FATIMA)

FINAL EXAM - AUG 2015

my mnemonics: improves QUANTITY of life in CHF- ABS (ACEI/ARBS, Beta-Blockers, Spironolactone). Improves QUALITY of life- Digital Film (Digitalis, Furosemide). Only 3 beta blockers are found to be useful in CHF: Carvedilol, metoprolol and bisopolol.

JEAN PAOLO M. DELFINO, MD (TOP 10 - FEB 2015 MED BOARDS; TOPNOTCH MD FROM FATIMA)

FINAL EXAM - AUG 2015

183

Your patient requires a local anesthetic with long duration of action. What will you give? A. procaine B. lidocaine C. cocaine D. bupivacaine E. None of the above

SIMILAR TO PREVIOUS BOARD EXAM CONCEPT/PRINCIPLE.. my mnemonics for this: BeT-long. Local anesthetics with long duration of action are Bupivacaine and Tetracaine. The longest acting is Ropivacaine ("at the end of a long ROPe")

JEAN PAOLO M. DELFINO, MD (TOP 10 - FEB 2015 MED BOARDS; TOPNOTCH MD FROM FATIMA)

FINAL EXAM - AUG 2015

184

What anti-retroviral agent causes peripheral neuropathy and pancreatitis? A. Zidovudine B. Zalcitabine C. Didanosine D. Abacavir E. Foscarnet

SIMILAR TO PREVIOUS BOARD EXAM CONCEPT/PRINCIPLE.. Zidovudine causes bone marrow suppression. Abacavir causes hypersensitivity. Didanosine causesperipheral neuropathy and pancreatitis. Stavudine and Zalcitabine causes peripheral neuropathy.

JEAN PAOLO M. DELFINO, MD (TOP 10 - FEB 2015 MED BOARDS; TOPNOTCH MD FROM FATIMA)

FINAL EXAM - AUG 2015

177

182

AUTHOR

TOPNOTCH MEDICAL BOARD PREP PHARMACOLOGY SUPEREXAM Page 22 of 83 For inquiries visit www.topnotchboardprep.com.ph or email us at [email protected]

TOPNOTCH EXAM MIDTERM 3 EXAM - AUG 2015

TOPNOTCH MEDICAL BOARD PREP PHARMACOLOGY SUPEREXAM For inquiries visit www.topnotchboardprep.com.ph or email us at [email protected] Item # 185

QUESTION

EXPLANATION

AUTHOR

TOPNOTCH EXAM FINAL EXAM - AUG 2015

Which of the following anti-microbial agent is bacteriostatic? A. Co-trimoxazole B. Pyrazinamide C. Metronidazole D. Aminoglycoside E. Tetracycline

Bacteriostatic agents: All protein synthesis inhibitors except aminoglycoside; ethambutol, nitrofurantoin; those with anti-metabolite mechanism of action- sulfonamide and trimethoprim (but they become bactericidal when combined).

JEAN PAOLO M. DELFINO, MD (TOP 10 - FEB 2015 MED BOARDS; TOPNOTCH MD FROM FATIMA)

186

Drug of choice for hydatid disease? A. Niclosamide B. Praziquantel C. Albendazole D. Mebendazole E. Pyrantel pamoate

Albendazole is the drug of choice for hydatid disease and cysticercosis.

JEAN PAOLO M. DELFINO, MD (TOP 10 - FEB 2015 MED BOARDS; TOPNOTCH MD FROM FATIMA)

FINAL EXAM - AUG 2015

187

Which of these drugs will be the fastest to be eliminated from the body? A. 100mg drug with zero order kinetics; elimination rate is 25mg/2hrs B. 100mg drug with first order kinetics; elimination t 1/2 is 2 hours C. 150mg drug with zero order kinetics; elimination rate is 25mg/hr D. 150mg drug with first order kinetics; elimination t 1/2 is 2 hours E. All of these will be eliminated at the same time Local anesthetics are not effective in inflamed infected tissues because? A. The cationic form is increased B. Protonated form predominates C. The drug is more water soluble D. A and B E. All of the above



JEAN PAOLO M. DELFINO, MD (TOP 10 - FEB 2015 MED BOARDS; TOPNOTCH MD FROM FATIMA)

FINAL EXAM - AUG 2015

Local anesthetics are weak bases, therefore, in an acidic environment like in infected tissues, the protonated or cationic form of the drug predominates. This is water soluble so the drug undergoes rapid clearance rather than being able to cross biological membranes.

JEAN PAOLO M. DELFINO, MD (TOP 10 - FEB 2015 MED BOARDS; TOPNOTCH MD FROM FATIMA)

FINAL EXAM - AUG 2015

189

The following are adverse effects of amiodarone except? A. hyperthyroidism B. hypothyroidism C. photodermatitis D. pulmonary fibrosis E. None of the above

JEAN PAOLO M. DELFINO, MD (TOP 10 - FEB 2015 MED BOARDS; TOPNOTCH MD FROM FATIMA)

FINAL EXAM - AUG 2015

190

This opioid antagonist is used in opioid and alcohol dependence A. Nalmefene B. Naltrexone C. Naloxone D. Nalbuphine E. A and C

Adverse effects of amiodarone: bradycardia, heart block, pulmonary fibrosis, elevated liver enzymes, photodermatitis, corneal microdeposit, halos in peripheral visual field, optic neuritis, hypothyroidism (blocks the peripheral conversion of T4 to T3) and hyperthyroidism (because it is also a potential source of large amounts of inorganic iodine) Naltrexone reduces craving in alcohol dependence. Note: there are probably 4 questions about opiates in our boards.

JEAN PAOLO M. DELFINO, MD (TOP 10 - FEB 2015 MED BOARDS; TOPNOTCH MD FROM FATIMA)

FINAL EXAM - AUG 2015

191

What is the mechanism of action of PTU? A. Blocks iodination and organification B. Inhibits iodide trapping C. Blocks coupling reaction D. Blocks peripheral conversion of T4 to T3 E. Inhibits secretion

There are 5 steps in thyroid hormone synthesis. 1. iodide trapping; 2. iodination/organification; 3. coupling; 4. secretion; 5. peripheral conversion. Steps 2 to 5 are actually blocked by PTU but the main effect among these is the inhibition of peripheral conversion.

JEAN PAOLO M. DELFINO, MD (TOP 10 - FEB 2015 MED BOARDS; TOPNOTCH MD FROM FATIMA)

FINAL EXAM - AUG 2015

192

This 3rd generation cephalosporin can cross the blood brain barrier A. ceftizoxime B. cefpodoxime C. ceftibuten D. cefaclor E. cefuroxime

All 3rd generation cephalosporins can cross the blood brain barrier except: ceftibuten, cefpodoxime, cefixime, cefoperazone.

JEAN PAOLO M. DELFINO, MD (TOP 10 - FEB 2015 MED BOARDS; TOPNOTCH MD FROM FATIMA)

FINAL EXAM - AUG 2015

193

These drugs increase the outflow of aqueous humor used in glaucoma except? A. timolol B. latanoprost C. epinephrine D. pilocarpine E. physostigmine

Drugs that increase outflow of aqueous humor: pilocarpine, physostigmine, latanoprost, epinephrine. Drugs that decrease aqueous humor secretion; mnemonics- TAMAD: Timolol, Acetazolamide, Mannitol, Apraclonidine, Dorzolamide.

JEAN PAOLO M. DELFINO, MD (TOP 10 - FEB 2015 MED BOARDS; TOPNOTCH MD FROM FATIMA)

FINAL EXAM - AUG 2015

194

Patient came in the clinic complaining of amenorrhea and galactorrhea. History revealed intake of an antipsychotic drug. What could have caused the symptoms of the patient? A. Clozapine B. Olanzapine C. Risperidone D. Haloperidol E. Chlorpromazine

Risperidone causes hyperprolactinemia.

JEAN PAOLO M. DELFINO, MD (TOP 10 - FEB 2015 MED BOARDS; TOPNOTCH MD FROM FATIMA)

FINAL EXAM - AUG 2015

188

TOPNOTCH MEDICAL BOARD PREP PHARMACOLOGY SUPEREXAM Page 23 of 83 For inquiries visit www.topnotchboardprep.com.ph or email us at [email protected]

TOPNOTCH MEDICAL BOARD PREP PHARMACOLOGY SUPEREXAM For inquiries visit www.topnotchboardprep.com.ph or email us at [email protected] Item # 195

QUESTION

EXPLANATION

Which has the highest partition coefficient among the inhalation anesthetics? A. methoxyflurane B. Nitrous oxide C. isoflurane D. sevoflurane E. desflurane

Methoxyflurane has the highest partition coefficient which means that it has the slowest onset and recovery.

JEAN PAOLO M. DELFINO, MD (TOP 10 - FEB 2015 MED BOARDS; TOPNOTCH MD FROM FATIMA)

Patient is diagnosed with Lennox-Gaustat Syndrome and is taking maintenance drug. He later on developed urolithiasis. The patient is probably taking what drug? A. topiramate B. lamotrigine C. ethosuximide D. carbamazepine E. levetiracetam Escitalopram is an antidepressant agent that belongs to which drug class? A. SSRI B. SNRI C. TCA D. MAOI E. 5HT2 antagonist

Topiramate is used as an adjunct in Lennox-Gaustat Syndrome. Its adverse effects are unusual/weird for anti-seizure drugs (ex. Urolithiasis, myopia, glaucoma)

JEAN PAOLO M. DELFINO, MD (TOP 10 - FEB 2015 MED BOARDS; TOPNOTCH MD FROM FATIMA)

FINAL EXAM - AUG 2015

Drugs that belong to SSRI: fluoxetine, sertraline, paroxetine, fluvoxamine, citalopram, escitalopram

JEAN PAOLO M. DELFINO, MD (TOP 10 - FEB 2015 MED BOARDS; TOPNOTCH MD FROM FATIMA)

FINAL EXAM - AUG 2015

198

7 year old male was noted to have difficulty sustaining attention in tasks or play activities in school and at home. He often leaves his seat inside the classroom and runs about and climbs excessively. What is the treatment of choice for this case? A. sibutramine B. methylphenidate C. diazepam D. behavioral therapy E. magnesium pemoline

Methylphenidate is the drug of choice for ADHD in children 6 years old and above. Mechanism of action is unknown but it mainly acts as a CNS stimulant similar to amphetamines.

JEAN PAOLO M. DELFINO, MD (TOP 10 - FEB 2015 MED BOARDS; TOPNOTCH MD FROM FATIMA)

FINAL EXAM - AUG 2015

199

What drug acts on epithelial sodium channels in cortical collecting ducts and is used to reduce potassium excretion in cases of hypokalemia? A. furosemide B. acetazolamide C. spironolactone D. triamterene E. hydrochlorothiazide

Triamterene and Amiloride are potassium sparing diuretics that act on epithelial sodium channels (ENaC) in cortical collecting duct, causing decreased Na reabsorption and K excretion. Spironolactone and eplerenone are also potassium sparing diuretics but they act on aldosterone receptors.

JEAN PAOLO M. DELFINO, MD (TOP 10 - FEB 2015 MED BOARDS; TOPNOTCH MD FROM FATIMA)

FINAL EXAM - AUG 2015

200

Which of the following beta-blockers have local anesthetic effect? A. Nadolol B. Penbutolol C. Pindolol D. Carteolol E. Atenolol

Beta-blockers with partial agonist effect on adrenergic receptors have intrinsic sympathetic activity: LAPPCC (Labetalol, Acebutolol, Penbutolol, Pindolol, Carteolol, Celiprolol). Beta-blockers with membrane stabilizing activity have local anesthetic effect: LAMPP (Labetalol, Acebutolol, Metoprolol, Pindolol, Propranolol)

JEAN PAOLO M. DELFINO, MD (TOP 10 - FEB 2015 MED BOARDS; TOPNOTCH MD FROM FATIMA)

FINAL EXAM - AUG 2015

201

In the stomach, aspirin will exist predominantly as the ______________ form. A. Hydrophilic, ionized, polar B. Lipophilic, unionized, non-polar C. Hydrophilic, unionized, non-polar D. Lipophilic, ionized, polar E. Hydrophilic, unionized, polar

Review the Henderson-Hasselbach equation. Aspirin (acetylsalicyclic acid) is acidic in nature. In an acidic environment like the stomach, it will exist predominantly as the lipid-soluble, unionized, nonpolar form.

GRACE ARVIOLA, MD (TOP 3 - AUG 2014 MED BOARDS; TOPNOTCH MD)

DIAGNOSTIC EXAM - FEB 2015

202

In the CHOP regimen for non-Hodgkin's lymphoma, which chemotherapeutic drug acts by inhibiting the mitotic spindle? A. Cyclophosphamide B. Hydroxyurea C. Oncovin D. Prednisone E. Doxorubicin

Oncovin is vincristine.

GRACE ARVIOLA, MD (TOP 3 - AUG 2014 MED BOARDS; TOPNOTCH MD)

DIAGNOSTIC EXAM - FEB 2015

203

A patient with leprosy is taking a drug that casts a red color to the skin. He is probably taking: A. Dapsone B. Rifampin C. Clofazimine D. Ethambutol E. Phenylbutazone

Rifampin causes red discoloration of body fluids but not the skin.

GRACE ARVIOLA, MD (TOP 3 - AUG 2014 MED BOARDS; TOPNOTCH MD)

DIAGNOSTIC EXAM - FEB 2015

204

What part of the pharmacokinetics of penicillin is affected by the drug probenecid? A. Glomerular filtration B. Tubular reabsorption C. Tubular secretion D. Hepatic uptake E. Protein-binding with albumin

Probenecid raises plasma concentration of penicillin by inhibiting its tubular secretion in the renal tubules.

GRACE ARVIOLA, MD (TOP 3 - AUG 2014 MED BOARDS; TOPNOTCH MD)

DIAGNOSTIC EXAM - FEB 2015

196

197

AUTHOR

TOPNOTCH MEDICAL BOARD PREP PHARMACOLOGY SUPEREXAM Page 24 of 83 For inquiries visit www.topnotchboardprep.com.ph or email us at [email protected]

TOPNOTCH EXAM FINAL EXAM - AUG 2015

TOPNOTCH MEDICAL BOARD PREP PHARMACOLOGY SUPEREXAM For inquiries visit www.topnotchboardprep.com.ph or email us at [email protected] Item # 205

QUESTION

EXPLANATION

AUTHOR

TOPNOTCH EXAM DIAGNOSTIC EXAM - FEB 2015

Hoffman elimination is exhibited by: A. Succinylcholine B. Diazepam C. Phenobarbital D. Phenytoin E. Atracurium

Hoofman elimination aka exhaustive methylation

GRACE ARVIOLA, MD (TOP 3 - AUG 2014 MED BOARDS; TOPNOTCH MD)

206

In a patient suffering from organophosphate poisoning with a cardiac rate of 45, what drug should be given? A. Neostigmine B. Atropine C. Epinephrine D. Pancuronium E. Edrophonium

Because of its antimuscarinic effect on the heart, atropine (the antidote for organophosphate poisoning) has a positive chronotropic effect.

GRACE ARVIOLA, MD (TOP 3 - AUG 2014 MED BOARDS; TOPNOTCH MD)

DIAGNOSTIC EXAM - FEB 2015

207

Patients receiving cyclophosphamide should also be given: A. Mesna B. Leucovorin C. Vitamin B6 D. Thiamine E. Amifostine

Leucovorin for methotrexate. Pyridoxine for isoniazid. Thiamine for alcoholics.

GRACE ARVIOLA, MD (TOP 3 - AUG 2014 MED BOARDS; TOPNOTCH MD)

DIAGNOSTIC EXAM - FEB 2015

208

Which statement is correct regarding the aminoglycosides? A. They work in anaerobic conditions. B. They exhibit concentration-dependent killing. C. Most drugs of this class are myelosuppresive. D. They cannot be given to patients allergic to penicillins. E. They are acidic in nature. What is the second messenger system for the receptors blocked by metoprolol? A. cAMP B. IP3 C. DAG D. cGMP E. PLC

Aminoglycosides only work in arebic conditions. They do not have cross-reactivity with the penicillins. Most drugs are nephrotoxic and ototoxic.

GRACE ARVIOLA, MD (TOP 3 - AUG 2014 MED BOARDS; TOPNOTCH MD)

DIAGNOSTIC EXAM - FEB 2015

Beta-2 receptors are coupled to the cAMP second messenger system.

GRACE ARVIOLA, MD (TOP 3 - AUG 2014 MED BOARDS; TOPNOTCH MD)

DIAGNOSTIC EXAM - FEB 2015

210

In the treatment of Parkinson's disease, what drug is given with levodopa to increase the drug's bioavailability? A. Entacapone B. Selegiline C. Carbidopa D. Fluoxetine E. Pyridoxine

Carbidopa peripherally inhibits the enzyme DOPA decarboxylase, the enzyme that converts DOPA to dopamine. Dopamine itself is unable to cross the blood-brain barrier but DOPA can. Inhibition of the enzyme results in more DOPA available to enter the CNS. Within the CNS, DOPA is converted to dopamine by the same enzyme. However, it is not inhibited by carbidopa at this location.

GRACE ARVIOLA, MD (TOP 3 - AUG 2014 MED BOARDS; TOPNOTCH MD)

DIAGNOSTIC EXAM - FEB 2015

211

Which drug used in the treatment of depression possess a tricyclic ring? A. Imipramine B. Fluoxetine C. Trazodone D. Venlafaxine E. Bupropion

Imipramine is a tricyclic antidepressant.

GRACE ARVIOLA, MD (TOP 3 - AUG 2014 MED BOARDS; TOPNOTCH MD)

DIAGNOSTIC EXAM - FEB 2015

212

Which antiretroviral drug acts through the M2 ion channel? A. Efavirenz B. Nevirapine C. AZT D. Amantadine E. Indinavir

Amantadine is also effective for influenza.

GRACE ARVIOLA, MD (TOP 3 - AUG 2014 MED BOARDS; TOPNOTCH MD)

DIAGNOSTIC EXAM - FEB 2015

213

Drug X has greater affinity for albumin than Drug Y. Considering all other parameters are the same, what significant drug-drug interaction is expected to take place when both drugs are administered simultaneously? A. Drug X will have a higher plasma concentration than Drug Y. B. Drug Y will not be filtered by the renal glomerulus. C. Drug Y will not be available to interact with its receptors in the tissues. D. The free plasma drug concentration of Drug Y will increase. E. Drug X can exert its full biologic activity. Dissociative anesthesia is effectively achieved by the combination of: A. Thiopental, diazepam, ketamine B. Ketamine, nitrous oxide, fentanyl C. Succinylcholine, thiopental, diazepam D. Nitrous oxide, halothane, fentanyl E. Succinylcholine, halothane, thiopental

Only the free, unbound drug can interact with receptors, exert biologic activity, and be excreted by the kidneys.

GRACE ARVIOLA, MD (TOP 3 - AUG 2014 MED BOARDS; TOPNOTCH MD)

DIAGNOSTIC EXAM - FEB 2015



GRACE ARVIOLA, MD (TOP 3 - AUG 2014 MED BOARDS; TOPNOTCH MD)

DIAGNOSTIC EXAM - FEB 2015

209

214

TOPNOTCH MEDICAL BOARD PREP PHARMACOLOGY SUPEREXAM Page 25 of 83 For inquiries visit www.topnotchboardprep.com.ph or email us at [email protected]

TOPNOTCH MEDICAL BOARD PREP PHARMACOLOGY SUPEREXAM For inquiries visit www.topnotchboardprep.com.ph or email us at [email protected] Item # 215

QUESTION

EXPLANATION

AUTHOR

TOPNOTCH EXAM DIAGNOSTIC EXAM - FEB 2015

Among the inhaled anesthetics, nitrous oxide has the _______ MAC and the ______ potency. A. Lowest, lowest B. Lowest, highest C. Highest, lowest D. Highest, highest E. Intermediate, intermediate

The MAC is a measure of an inhaled anesthetic's potency. The relationship is inverse.

GRACE ARVIOLA, MD (TOP 3 - AUG 2014 MED BOARDS; TOPNOTCH MD)

216

Which properties of digoxin make it an effective treatment in cases of heart failure? A. Positive inotrope, positive chronotrope B. Positive inotrope, negative chronotrope C. Negative inotrope, negative chronotrope D. Negative inotrope, positive chronotrope E. None of the above

Digoxin also slows AV conduction (negative dromotrope)

GRACE ARVIOLA, MD (TOP 3 - AUG 2014 MED BOARDS; TOPNOTCH MD)

DIAGNOSTIC EXAM - FEB 2015

217

In anticoagulation, warfarin therapy is usually overlapped with heparin for the first 1-2 days. Why is this so? A. To achieve supraoptimal anticoagulation during critical periods of illness as warfarin and heparin have synergistic effects. B. To prevent bleeding as heparin partially counteracts warfarin's hemorrhagic property. C. Warfarin is metabolized slowly thus leading to a delay in anticoagulation if heparin is not also given. D. To compensate for warfarin's initial prothrombotic property. E. Heparin decreases the clearance of warfarin thus achieving greater plasma drug concentration of warfarin. A commercial product for colds contain the following: Phenylephrine, chlorpheniramine, and paracetamol. Which acts as the decongestant? A. Paracetamol B. Chlorpheniramine C. Phenylephrine D. Both chlorpheniramine and phenylephrine E. This product does not contain a decongestant.

Warfarin acts by vitamin K anatagonism. The initial prothrombotic effect of warfarin is because the synthesis of protein S, an anticoagulant, also depends on vitamin K. Thus, protein S levels decline initially and the effect of protein C (for which it serves as a co-factor) is also diminshed.

GRACE ARVIOLA, MD (TOP 3 - AUG 2014 MED BOARDS; TOPNOTCH MD)

DIAGNOSTIC EXAM - FEB 2015

Phenylephrine constricts blood vessels through stimulation of adrenergic receptors. This acts as a decongestant.

GRACE ARVIOLA, MD (TOP 3 - AUG 2014 MED BOARDS; TOPNOTCH MD)

DIAGNOSTIC EXAM - FEB 2015

219

Which antimalarial drug should not be given to a patient with glucose-6-phosphate dehydrogenase deficiency? A. Chloroquine B. Quinine C. Atovaquone-proquanil D. Mefloquine E. Primaquine



GRACE ARVIOLA, MD (TOP 3 - AUG 2014 MED BOARDS; TOPNOTCH MD)

DIAGNOSTIC EXAM - FEB 2015

220

What drug should be given to an immunocompromised patient with CMV retinitis? A. Acyclovir B. Vidarabine C. Indinavir D. Ganciclovir E. AZT

Ganciclovir is the drug of choice for CMV. Foscarnet is an alternative.

GRACE ARVIOLA, MD (TOP 3 - AUG 2014 MED BOARDS; TOPNOTCH MD)

DIAGNOSTIC EXAM - FEB 2015

221

What is the pharmacologic basis of using Timolol in open angle glaucoma? A. Increaed outflow via dilatation of uveoscleral veins B. It leads to opening of trabecular meshwork C. It suppresses the ciliary epithelium from producing aqueous humor D. lt decrease production of aqueous humor by decreasing levels of bicarbonate through cAMP mediated pathway. E. none of the above A 7 yo male was brought to ER secondary to symptoms suggestive of botulinum toxin ingestion, which of the following are likely included in the symptoms of the patient? A. Muscle fasciculation B. Bronchoconstriction C. Increased bowel movement D. blurred near vision E. hypertension Which of the following will increase the arrythmogenic side effect of digoxin? A. Decrease sympathetic discharge B. Increase serum Potassium C. Decrease serum Calcium D. Increase Parasympathetic discharge E. None of the above

MOA: letter A- alpha agonist and epinephrine; B- Cholinomimetics ( physostigmine, pilocarpine) D- there is no such thing cAMP dependent decrease in bicarbonate; however inhibition of Carbonic anhydrase leads to decrease humor production via decreased bicarbonate levels.

LEAN ANGELO SILVERIO, MD (TOP 4 - AUG 2014 MED BOARDS; TOPNOTCH MD), MD

MIDTERM EXAM 1 - FEB 2015

Botulinum toxin inhibits the release of vesicular contents from all types of cholinergic nerve endings. Since both ganglionic/autonomic and peripheral motor synapse are affected. Patient will present with hypotension and lack of muscular contraction. Blurred near vision is a result of paralysis of ciliary muscle aka cycloplegia

LEAN ANGELO SILVERIO, MD (TOP 4 - AUG 2014 MED BOARDS; TOPNOTCH MD), MD

MIDTERM EXAM 1 - FEB 2015

hypokalemia, hypomagnesemia, hypercalcemia, and increase vagal tone increases the risk for digoxin induced arrhytmia

LEAN ANGELO SILVERIO, MD (TOP 4 - AUG 2014 MED BOARDS; TOPNOTCH MD), MD

MIDTERM EXAM 1 - FEB 2015

218

222

223

TOPNOTCH MEDICAL BOARD PREP PHARMACOLOGY SUPEREXAM Page 26 of 83 For inquiries visit www.topnotchboardprep.com.ph or email us at [email protected]

TOPNOTCH MEDICAL BOARD PREP PHARMACOLOGY SUPEREXAM For inquiries visit www.topnotchboardprep.com.ph or email us at [email protected] Item # 224

225

QUESTION

EXPLANATION

AUTHOR

TOPNOTCH EXAM MIDTERM EXAM 1 - FEB 2015

Which of the following is correct regarding isosorbide mononitrate? A. Can be use along with PDE inhibitors compared to other forms of nitroglycerin B. It has a similar mechanism of action with Nitroprusside by phosphorylation of myosin light chain phosphate C. It has no direct effect on cardiac muscle D. the primary mechanism for decrease in BP is due to decrease in afterload E. B and C are correct This drug when given IV and supraphysiologic dose will cause conduction block on AV node via increasing K efflux? A. Verapamil IV B. Adenosine C. Fleicanide D. Amiodarone E. None of the above

ISMN is the oral form of nitroglycerin. Its mechanism is by activating guanylyl cyclase causing an increase in cGMP eventually leading dephosphorylation of MLC. Its antihypertensive effect is primarily via venodilation. It has no direct inotropic or chronotropic effect to the heart. all forms of nitroglycerin should not concomitantly use with anyform of PDEI ( sildenafil) since it will cause profound hypotension and hypoperfusion of critical organs

LEAN ANGELO SILVERIO, MD (TOP 4 - AUG 2014 MED BOARDS; TOPNOTCH MD), MD

Verapamil can also decrease AV node conduction however via blocking L type Ca channels and not K channels. Fleicanide is a class IC drug that affects primarily Na channels present in both atria and ventricles. Amiodarone is the most efficacious of all antiarrhythmic drug since it blocks Na, Ca, K and Beta receptors. the question is specific on its action in reference to AV node. the best answer is adenosine. antiarrhytmic drugs that affects the K channel at phase 3 causes prolongation of Action Potential leading prolonged QT interval. These drugs are of the Class IA and III. Class IA drugs - procainamide, quinidine, disopyramide. Class III- Ibutilide, sotalol, dofetilide

LEAN ANGELO SILVERIO, MD (TOP 4 - AUG 2014 MED BOARDS; TOPNOTCH MD), MD

MIDTERM EXAM 1 - FEB 2015

LEAN ANGELO SILVERIO, MD (TOP 4 - AUG 2014 MED BOARDS; TOPNOTCH MD), MD

MIDTERM EXAM 1 - FEB 2015

Naratriptan and its prototype Sumatriptan are 5HT 1D agonist use for the treatment of acute migraine and cluster headache. Tegaserod indicated for constipation is a 5HT4 partial agonist.Ondansetron is a 5-HT3 antagonist ise for antiemesis in post operative and postchemotherapy patients. Ketanserin is a 5HT2 antagonist use for controlling the systemic effects of carcinoid syndrome. SIMILAR TO PREVIOUS BOARD EXAM CONCEPT/PRINCIPLE last aug 2014. Epoprostenol is a Prostacyclin analog use for dialysis to prevent platelet aggregation and also for pulmonary hypertension. Latanoprost or PGF2a analog is the one use for glaucoma. Misoprostol or RU 486 (PGE1 analog) has an off label indication for induction of labor. it primarily acts as an abortifacient. Corticosteroids exerts its anti inflammatory effect by inhibiting the Phospholipase A2 enzyme which is responsible for generation of arachidonic acid from membrane lipids. Phospholipase C is not connected with corticosteroids and its function is to generate IP3 and DAG. Cyclooxygenase and thromboxane synthase is primarily inhibit by Coxibs and NSAIDS.

LEAN ANGELO SILVERIO, MD (TOP 4 - AUG 2014 MED BOARDS; TOPNOTCH MD), MD

MIDTERM EXAM 1 - FEB 2015

LEAN ANGELO SILVERIO, MD (TOP 4 - AUG 2014 MED BOARDS; TOPNOTCH MD), MD

MIDTERM EXAM 1 - FEB 2015

LEAN ANGELO SILVERIO, MD (TOP 4 - AUG 2014 MED BOARDS; TOPNOTCH MD), MD

MIDTERM EXAM 1 - FEB 2015

226

Which of the following drugs can cause prolonged QT interval leading to Torsade de Pointes arrhytmia? A. Esmolol B. Propafenone C. Procainamide D. Verapamil E. None of the above

227

Which in the following drugs is classified as 5HT1D agonist? A. Tegaserod B. Ketanserin C. Ondansetron D. Naratriptan E. None of the above

228

The primary indication of Epoprostenol? A. Open angle glaucoma - increase outflow of aqueous humor B. Severe pulmonary hypertension C. Postpartum bleeding D. All of the above E. None of the above

229

What is the site of action of corticosteroids in terms of its antiinflammatory effect? A. Cyclooxygenase 1 and 2 B. Phospholipase C C. Phospholipase A2 D. Thromboxane synthase E. All of the above

230

the major systemic side effects of systemic corticosteroids are much more likely to occur if at least the duration of treatment is more than_____? A. 14 days B. 4 weeks C. 3 weeks D. 20 days E. 2 months

Katzung Review of pharmacology 8th ed pp 171. .

LEAN ANGELO SILVERIO, MD (TOP 4 - AUG 2014 MED BOARDS; TOPNOTCH MD), MD

MIDTERM EXAM 1 - FEB 2015

231

What is the most serious side effect of Methanol ingestion? A. Retinal damage B. Hepatotoxicity C. Blood dyscrasia D. Nephrotoxicity E. Neurotoxicity- status epilepticus

LEAN ANGELO SILVERIO, MD (TOP 4 - AUG 2014 MED BOARDS; TOPNOTCH MD), MD

MIDTERM EXAM 1 - FEB 2015

232

which of the following antiseizure drugs can inhibit cytochrome isoenzymes leading to increase plasma concentration of other drugs? A. Phenobarbital B. Lorazepam C. Oxcarbazepine D. Valproic acid E. None of the above

Methanol or wood alcohol is metabolized by alcohol dehydrogenase to produce a product called Formaldehyde and formic acid. This metabolite causes severe acidosis and prominent retinal damage. Nephrotoxicity is a major side effect of ethylene glycol poisoning via its oxalic acid metabolite. treatment for both alcohol is by administration of Fomepizole ( inhibitor alcohol dehydrogenase) and ethanol ( competitive substrate for alcohol dehydrogenase). Oxcarbazepine, carbamazepine, Barbiturates, Phenytoin are all drug metabolizing inducers. Only Valproic acid in the choice is an inhibitor. Benzodiazepines do not affect the function of the cytochromes.

LEAN ANGELO SILVERIO, MD (TOP 4 - AUG 2014 MED BOARDS; TOPNOTCH MD), MD

MIDTERM EXAM 1 - FEB 2015

TOPNOTCH MEDICAL BOARD PREP PHARMACOLOGY SUPEREXAM Page 27 of 83 For inquiries visit www.topnotchboardprep.com.ph or email us at [email protected]

TOPNOTCH MEDICAL BOARD PREP PHARMACOLOGY SUPEREXAM For inquiries visit www.topnotchboardprep.com.ph or email us at [email protected] Item # 233

QUESTION

AUTHOR

TOPNOTCH EXAM MIDTERM EXAM 1 - FEB 2015

Katzung Review of pharmacology 8th ed pp 233. Both Levodopa and bromocriptine can cause exacerbation of schizophrenia by increasing brain dopamine levels. However, only bromocriptine can cause ergot related side effects.

LEAN ANGELO SILVERIO, MD (TOP 4 - AUG 2014 MED BOARDS; TOPNOTCH MD), MD

PPAR alpha ligands is the MOA of fibric acid derivatives ( Gemfibrozil and fenofibrate). It increases the supersaturation of bile leading cholelithiasis. Risk for hepatotoxicity only significantly increase if gemfibrozil is combined with HMG CoA inhibitor. Cutaneous flushing and hyperuricemia are toxicity profile of Niacin. Intestinal malabsorption is a primary side effect of resins ( cholestyramine, colestipol, colsevelam) Aminoglycoside exerts concentration dependent killing and post antibiotic effect. It also has curare like action at high dose leading to respiratory paralysis. Aminoglycosides needs Oxygen dependent transport to exerts inhibitory effect 30s ribosome. therefore, it has no activity to anaerobes. acetylation by plasmid mediated group transferase is the major mode of resistance.

LEAN ANGELO SILVERIO, MD (TOP 4 - AUG 2014 MED BOARDS; TOPNOTCH MD), MD

MIDTERM EXAM 1 - FEB 2015

LEAN ANGELO SILVERIO, MD (TOP 4 - AUG 2014 MED BOARDS; TOPNOTCH MD), MD

MIDTERM EXAM 1 - FEB 2015

AIDS patient with a CD4 count of less than 50/UL are prone to have Mycobacterium Avum Intracellulare infection. The only approved prophylactic treatment are daily dose of clarithromycin with or without rifabutin and once weekly dose of azithromycin.

LEAN ANGELO SILVERIO, MD (TOP 4 - AUG 2014 MED BOARDS; TOPNOTCH MD), MD

MIDTERM EXAM 1 - FEB 2015

Which in the following choices can be primarily use for the treatment of hemolymphatic stage of trypanosomiasis and for Pneumocystis jiroveci infection A. Sodium Stibogluconate B. Metronidazole C. Nifurtimox D. Pentamidine E. Cotrimoxazole What is the drug of choice for all forms of schistosomiasis and paragonimiasis? A. Diethylcarbamazine B. Praziquantel C. Ivermectin D. Albendazole E. Piperazine

Sodium stibogluconate - treatment of all forms of leishmaniasis infection; Metronidazole is not effective in the said infection, it is use for giardia and trichomonas infection. Nifurtimox is for chagas diseae ( T cruzi infection). Cotrimoxazole is used for P jiroveci but not effective in Trypanosomiasis infection.

LEAN ANGELO SILVERIO, MD (TOP 4 - AUG 2014 MED BOARDS; TOPNOTCH MD), MD

MIDTERM EXAM 1 - FEB 2015

DEC is the drug of choice for loa loa infection, it is also an alternative drug for elephantiasis. Ivermectin - DOC for onchocerciasis, cutaneous larva migransm and strongyloides. Albendazole- ascariasis, hookworm, pinworm, whipworm, cysticercosis and hydatid disease. Piperazine is an alternative drug for ascariasis.

LEAN ANGELO SILVERIO, MD (TOP 4 - AUG 2014 MED BOARDS; TOPNOTCH MD), MD

MIDTERM EXAM 1 - FEB 2015

239

In what phase of the cell cycle does vinca alkaloid exerts its effect? A. G0 B. G1 C. S D. G2 E. M

Vinca alkaloids ( vinblastine, vincristine, vinorelbine) blocks the formation of mitotic spindle by preventing the assembly of tubin dimers into microtubules. Therefore it acts on the M phase.

LEAN ANGELO SILVERIO, MD (TOP 4 - AUG 2014 MED BOARDS; TOPNOTCH MD), MD

MIDTERM EXAM 1 - FEB 2015

240

Which of the following drugs/drug regimen is primarily used for Hodgkin's Lymphoma A. ABVD- adriamycin, bleomycin, vincristine, dacarbazine, prednisone B. PEB - cisplatin, etoposide, bleomycin C. Gemcitabine + 5 FU + radiation D. EMACO- etoposide, methotrexate, actinomycin D, cyclophosphamide, vincristine E. all of the above 25 year old female came in with chief complaint of cough of 3 weeks duration, this was accompanied by night sweats, weight loss, body malaise. PE revealed slightly pale palpebral conjunctiva, multiple bilateral posterior cervical lymphadenopathies but with no adventitious breath sounds on bilateral lung fields. The rest of the physical examination was unremarkable, which of the following drugs is considered to be the most active for her condition? A. Rifampicin B. Isoniazid C. Pyrazinamide D. Ethambutol E. streptomycin

B- testicular cancer, C- pancreatic cancer, D- choriocarcinoma. Aug 2014 SIMILAR TO PREVIOUS BOARD EXAM CONCEPT/PRINCIPLE.

LEAN ANGELO SILVERIO, MD (TOP 4 - AUG 2014 MED BOARDS; TOPNOTCH MD), MD

MIDTERM EXAM 1 - FEB 2015

SIMILAR TO PREVIOUS BOARD EXAM CONCEPT/PRINCIPLE.,Considered as the most active anti-TB drug, in fact the only drug given to treat latent tuberculosis

KEVIN BRYAN LO, MD (TOP 7 - AUG 2014 MED BOARDS; TOPNOTCH MD)

MIDTERM 2 EXAM - FEB 2015

234

235

236

237

238

241

Which of the following antiparkinsonian drugs can cause exacerbation of psychosis and produce ergot related effects such as pulmonary fibrosis and erythromelalgia A. Amantadine B. Bromocriptine C. Entacapone D. Haloperidol E. None of the above Prolonged treatment with a PPAR alpha ligand in the treatment of hyperlipidemia will increase the risk for the following condition? A. Cutaneous flushing B. Hyperuricemia leading to Gout C. Intestinal malabsorption D. Gallstones E. hepatotoxicity

EXPLANATION

Which of the following is not true in the pharmacologic profile of Aminoglycosides? A. The primary mechanism of resistance is due to plasmid mediated group transferases B. Bactericidal action continuous even if the plasma drug concentration is below measuring levels C. It does not have an activity against anaerobic organisms D. high dose can cause respiratory paralysis E. none of the above Which of the following is a drug of choice for daily dose prophylaxis in AIDS patient with a CD4 count of 50/UL? A. Cotrimoxazole B. Kanamycin C. Azithromycin D. Erythromycin E. Clarithromycin

TOPNOTCH MEDICAL BOARD PREP PHARMACOLOGY SUPEREXAM Page 28 of 83 For inquiries visit www.topnotchboardprep.com.ph or email us at [email protected]

TOPNOTCH MEDICAL BOARD PREP PHARMACOLOGY SUPEREXAM For inquiries visit www.topnotchboardprep.com.ph or email us at [email protected] Item # 242

QUESTION

EXPLANATION

A 56 year old patient diagnosed with rheumatic heart disease underwent mitral valve replacement and is placed on oral anticoagulant Warfarin, she comes to your clinic for a follow up check up and complaints of heartburn and epigastric pain, which of the following medications will you avoid prescribing? A. Cimetidine B. Aluminum Magnesium Hydroxide C. lansoprazole D. ranitidine E. sucralfate

SIMILAR TO PREVIOUS BOARD EXAM CONCEPT/PRINCIPLE, which asked us directly which of the following is a CYP inhibitor: cimetidine, Warfarin is majorly metabolized by CYP 2C9, cimetidine inhibits this enzyme among others which inhibits warfarin metabolism which increases its toxicity

KEVIN BRYAN LO, MD (TOP 7 - AUG 2014 MED BOARDS; TOPNOTCH MD)

Which of the following medications is considered as an inhibitor of glucocorticoid synthesis which could be used in the treatment of cushing's syndrome and some types of cancers? A. Methylprednisolone B. anastrozole C. Danazol D. Aminogluthetimide E. Flutamide Which of the following drugs can cause cataract formation? A. allopurinol B. pyrazinamide C. propanolol D. omeprazole E. ciprofloxacin

SIMILAR TO PREVIOUS BOARD EXAM CONCEPT/PRINCIPLE, I couldn’t believe this was actually asked, aminoglutethimide inhibits desmolase blocking conversion of cholesterol to pregnenolone reduces synthesis of all hormonally active steroids, anastrozole inhibits only estrogen synthesis, danazol is an androgen, flutamide is androgen antagonist

KEVIN BRYAN LO, MD (TOP 7 - AUG 2014 MED BOARDS; TOPNOTCH MD)

MIDTERM 2 EXAM - FEB 2015

SIMILAR TO PREVIOUS BOARD EXAM CONCEPT/PRINCIPLE, one of the hardes pharma questions we encountered, we did not know at that time that allopurinol was associated with cataracts :)

KEVIN BRYAN LO, MD (TOP 7 - AUG 2014 MED BOARDS; TOPNOTCH MD)

MIDTERM 2 EXAM - FEB 2015

245

Which of the following chemotherapeutic regimens could be used for the treatment of advanced hodgkin's disease? A. FOLFOX B. MOPP C. CHOP D. 7+3 E. BEP

SIMILAR TO PREVIOUS BOARD EXAM CONCEPT/PRINCIPLE - MOPP nitrogen mustard, oncovin, procarbazine, prednisone, take time to know the common regimens for other cancers as well, ex: folfox colorectal, CHOP nonhodgkins, BEP testicular, 7+3 AML

KEVIN BRYAN LO, MD (TOP 7 - AUG 2014 MED BOARDS; TOPNOTCH MD)

MIDTERM 2 EXAM - FEB 2015

246

Which of the following medications causes mydriasis but without cycloplegia? A. tropicamide B. homatropine C. cylopentolate D. phenylephrine E. atropine

SIMILAR TO PREVIOUS BOARD EXAM CONCEPT/PRINCIPLE also a difficult one, only phenylephrine among the choices causes mydriasis without affecting accomodation

KEVIN BRYAN LO, MD (TOP 7 - AUG 2014 MED BOARDS; TOPNOTCH MD)

MIDTERM 2 EXAM - FEB 2015

247

When a pregnant 18 year old female delivered her child, her baby was noted to have characteristic small palpebral fissures, smooth philtrum and thin upper lip, which of the following substances when ingested by the mother during pregnancy could potentially be the cause of such changes? A. phenytoin B. warfarin C. ethanol D. captopril E. isotretinoin What is the predominant reason why nitroglycerin is generally preferred not to be given via the oral route? A. could potentially cause dangerous systemic hypotension B. is considerably less effective when given through this route C. is immediately inactivated by exposure to low gastric pH D. there is high first pass effect E. food in the stomach significantly retards absorption Which of the following is an inhalational anesthetic of choice for asthmatic patient because it causes the least bronchospasm? A. halothane B. desflurane C. sevoflurane D. thiopental E. propofol

SIMILAR TO PREVIOUS BOARD EXAM CONCEPT/PRINCIPLE, know the common syndromes and teratogen, answer here is fetal alcohol syndrome

KEVIN BRYAN LO, MD (TOP 7 - AUG 2014 MED BOARDS; TOPNOTCH MD)

MIDTERM 2 EXAM - FEB 2015

SIMILAR TO PREVIOUS BOARD EXAM CONCEPT/PRINCIPLE, high first pass effect is the best answer, low bioavailability through the oral route due to liver metabolism

KEVIN BRYAN LO, MD (TOP 7 - AUG 2014 MED BOARDS; TOPNOTCH MD)

MIDTERM 2 EXAM - FEB 2015

SIMILAR TO PREVIOUS BOARD EXAM CONCEPT/PRINCIPLE, halothane generally is the inhaled agent of choice, sevoflurane is debatable, desflurane is pungent, the others are IV anesthetics

KEVIN BRYAN LO, MD (TOP 7 - AUG 2014 MED BOARDS; TOPNOTCH MD)

MIDTERM 2 EXAM - FEB 2015

Which of the following drugs given contains an effect which suppresses nausea and vomiting ? A. propanolol B. procarbazine C. chlorambucil D. chlorpromazine E. chloramphenicol

SIMILAR TO PREVIOUS BOARD EXAM CONCEPT/PRINCIPLE, difficult because chlorpromazine usually is used as a antipsychotic but has antiemetic effects belonging to the phenothiazine class along with other drugs used to control nausea vomiting like promethazine

KEVIN BRYAN LO, MD (TOP 7 - AUG 2014 MED BOARDS; TOPNOTCH MD)

MIDTERM 2 EXAM - FEB 2015

243

244

248

249

250

AUTHOR

TOPNOTCH MEDICAL BOARD PREP PHARMACOLOGY SUPEREXAM Page 29 of 83 For inquiries visit www.topnotchboardprep.com.ph or email us at [email protected]

TOPNOTCH EXAM MIDTERM 2 EXAM - FEB 2015

TOPNOTCH MEDICAL BOARD PREP PHARMACOLOGY SUPEREXAM For inquiries visit www.topnotchboardprep.com.ph or email us at [email protected] Item # 251

QUESTION

EXPLANATION

AUTHOR

TOPNOTCH EXAM MIDTERM 2 EXAM - FEB 2015

Which of the following medications could be given to high risk or immunocompromised patients with RSV infections? A. Acyclovir B. Fosamprenavir C. Indinavir D. Ganciclovir E. Ribavirin

SIMILAR TO PREVIOUS BOARD EXAM CONCEPT/PRINCIPLE, acyclovir and ganciclovir for herpes group, indinavir and fosamprenavir for hiv

KEVIN BRYAN LO, MD (TOP 7 - AUG 2014 MED BOARDS; TOPNOTCH MD)

252

Using drugs to block which of the following chemical mediators could potentially decrease leukocyte chemotaxis? A. prostaglandin H B. cyclooxygenase C. bradykinin D. leukotrienes E. serotonin

SIMILAR TO PREVIOUS BOARD EXAM CONCEPT/PRINCIPLE, but leukotrienes could also serve as chemotactic factors

KEVIN BRYAN LO, MD (TOP 7 - AUG 2014 MED BOARDS; TOPNOTCH MD)

MIDTERM 2 EXAM - FEB 2015

253

A patient is taking fenofibrate for elevated trigylceride levels, which of the following laboratory examinations should the doctor request to monitor for potential side effects upon follow up after 3 to 6 months? A. CBC with differential count B. urinalysis C. AST, ALT D. fasting blood glucose levels E. serum total cholesterol and triglycerides

SIMILAR TO PREVIOUS BOARD EXAM CONCEPT/PRINCIPLE, fibrates can potentially increase liver enzymes

KEVIN BRYAN LO, MD (TOP 7 - AUG 2014 MED BOARDS; TOPNOTCH MD)

MIDTERM 2 EXAM - FEB 2015

254

Which of the following medications can be used as part of a regimen for migraine prophylaxis? A. ibuprofen B. acetaminophen C. sumatriptan D. propranolol E. dihydroergotamine

SIMILAR TO PREVIOUS BOARD EXAM CONCEPT/PRINCIPLE, prophylaxis: pizotifen, TCA, anticonvulsants and beta blockers, methysergide, flunarizine

KEVIN BRYAN LO, MD (TOP 7 - AUG 2014 MED BOARDS; TOPNOTCH MD)

MIDTERM 2 EXAM - FEB 2015

255

Which of the following medications can decrease conduction through the AV node? A. nifedipine B. verapamil C. lithium D. prazosin E. dypyridamole

SIMILAR TO PREVIOUS BOARD EXAM CONCEPT/PRINCIPLE, verapamil is a non dihydropyridine calcium channel blocker which exerts its actions more on the heart decreasing AV nodal conduction

KEVIN BRYAN LO, MD (TOP 7 - AUG 2014 MED BOARDS; TOPNOTCH MD)

MIDTERM 2 EXAM - FEB 2015

256

A 24 year old female is taking oral contraceptives as family planning method. Which of the following drugs is considered a CYP450 enzyme inducer which can cause rapid metabolism of other drugs such as oral contraceptives which can potentially decrease their effectiveness? A. isoniazid B. trimethoprim sulfamethoxazole C. griseofulvin D. Amiodarone E. ketoconazole Which of the following chemotherapeutic agents can produce pulmonary fibrosis as toxicity? A. bleomycin B. doxorubicin C. irinotecan D. etoposide E. carmustine

SIMILAR TO PREVIOUS BOARD EXAM CONCEPT/PRINCIPLE, what came out as an enzyme inducer in ours was phenobarbital, griseofulvin is an inducer all the others are inhibitors, memorize at least the most common of them inducers and inhibitors, topnotch handouts are good reference

KEVIN BRYAN LO, MD (TOP 7 - AUG 2014 MED BOARDS; TOPNOTCH MD)

MIDTERM 2 EXAM - FEB 2015

SIMILAR TO PREVIOUS BOARD EXAM CONCEPT/PRINCIPLE, know the characteristic adverse effects and toxicities, the peculiar ones, the ones that make the drugs famous, pulmo fibrosis = bleomycin

KEVIN BRYAN LO, MD (TOP 7 - AUG 2014 MED BOARDS; TOPNOTCH MD)

MIDTERM 2 EXAM - FEB 2015

258

Which among the following cardiac drugs can decrease preload, afterload and contractility? A. digoxin B. captopril C. amlodipine D. hydrochlorthiazide E. propranolol

KEVIN BRYAN LO, MD (TOP 7 - AUG 2014 MED BOARDS; TOPNOTCH MD)

MIDTERM 2 EXAM - FEB 2015

259

Which of the following antibiotics are highly protein bound which can act to displace other substances from albumin binding and increase free drug levels A. chloramphenicol B. ciprofloxacin C. cotrimoxazole D. tetracycline E. gentamicin

know the mechanism understand them, know and master normal physiology, digoxin is a negative chronotropic but positive inotropic inc contractility, captopril inhibits ACE, acts on the RAAS decreases preload by decreasing aldosterone and sodium water retention hence decrease blood volume dec venous return, amlodipine calcium antagonist predominantly on the vessels decrease afterload by dilating resistance vessels, hydrochlorthiazide decreases preload by increasing secretion of both sodium water, propranolol, non selective beta blocker, decreases renin secretion hence RAAS through B1 blockage this includes angiotensin 2 so decreased both preload afterload, decreases contractilitty through B1 blockade, SIMILAR TO PREVIOUS BOARD EXAM CONCEPT/PRINCIPLE, sulfa drugs are usually highly protein bound drugs which can displace other drugs making the free drug available to easily permeate cell membranes or bind to receptors, it can also displace bilirubin from albumin raising bilirubin levels in neonates

KEVIN BRYAN LO, MD (TOP 7 - AUG 2014 MED BOARDS; TOPNOTCH MD)

MIDTERM 2 EXAM - FEB 2015

257

TOPNOTCH MEDICAL BOARD PREP PHARMACOLOGY SUPEREXAM Page 30 of 83 For inquiries visit www.topnotchboardprep.com.ph or email us at [email protected]

TOPNOTCH MEDICAL BOARD PREP PHARMACOLOGY SUPEREXAM For inquiries visit www.topnotchboardprep.com.ph or email us at [email protected] Item # 260

QUESTION

EXPLANATION

AUTHOR

TOPNOTCH EXAM MIDTERM 2 EXAM - FEB 2015

Which of the following drugs with mechanism of action that inhibits protein synthesis by inhibiting binding of the tRNA to the mRNA ribosome complex A. tetracycline B. clindamycin C. gentamycin D. erythromycin E. chloramphenicol

SIMILAR TO PREVIOUS BOARD EXAM CONCEPT/PRINCIPLE, please know the mechanism of action of each drug class

KEVIN BRYAN LO, MD (TOP 7 - AUG 2014 MED BOARDS; TOPNOTCH MD)

If a single dose of a known drug with first order elimination is given intravenously, how long will it take for 75% of the drug to be eliminated if the halflife is 1.5 hours? A. 1.5 hours B. 2 hours C. 2.5 hours D. 3 hours E. 6 hours Which of the following teratogenic effects is associated with valproic acid use in pregnancy? A. Craniofacial anomalies B. Neural tube defects C. Ebstein anomaly D. Fetal hydantoin syndome E. Microcephaly

Review first-order elimination. 50% is one half-life, 75% is 2 X half-life and so on..

RAYMUND MARTIN LI, MD (TOP 1 - AUG 2014 MED BOARDS; TOPNOTCH MD)

MIDTERM 3 EXAM - FEB 2015

A is carbamazepine; C is Lithium, D is phenytoin

RAYMUND MARTIN LI, MD (TOP 1 - AUG 2014 MED BOARDS; TOPNOTCH MD)

MIDTERM 3 EXAM - FEB 2015

You are assigned to go on duty as an intern of the toxicology department of the hospital. You recall that activated charcoal will NOT be effective adsorbing which of the following? A. Iron B. Theophylline C. Phenobarbital D. Amitryptiline E. Digoxin Which of the following Cholinomimetic drugs are used exclusively in the treatment of Alzheimer's disease? A. Donepezil B. Physostigmine C. Rivastigmine D. A and B E. A and C

Iron, lithium, cyanide, alcohol are poorly adsorbed by activated charcoal

RAYMUND MARTIN LI, MD (TOP 1 - AUG 2014 MED BOARDS; TOPNOTCH MD)

MIDTERM 3 EXAM - FEB 2015

Donepezil, Rivastigmine, Tacrine used exclusively in Alzheimer's

RAYMUND MARTIN LI, MD (TOP 1 - AUG 2014 MED BOARDS; TOPNOTCH MD)

MIDTERM 3 EXAM - FEB 2015

You prescribe a cancer patient who is undergoing chemotherapy with allopurinol to address hyperuricemia. Upon reviewing his chart, you realize caution is needed because of risk of toxicity. Which of the following chemotherapeutic drus is most likely being given to the patient? A. Azathioprine B. Methotrexate C. Fluorouracil D. Cytarabine E. All of the above Adrenoreceptor blockers such as acebutolol and pindolol are unique for having intrinsic sympathetic activity. What is meant by this characteristic? A. has a membrane-stabilizing activity B. has partial agonist activity C. has receptor selectivity D. A and B E. All of the above

Azathioprine/Mercaptopurine metabolism is inhibited by allopurinol.

RAYMUND MARTIN LI, MD (TOP 1 - AUG 2014 MED BOARDS; TOPNOTCH MD)

MIDTERM 3 EXAM - FEB 2015



RAYMUND MARTIN LI, MD (TOP 1 - AUG 2014 MED BOARDS; TOPNOTCH MD)

MIDTERM 3 EXAM - FEB 2015

267

Which of the following drugs that used in gout has been associated with cataract formation? A. Colchicine B. Allopurinol C. Probenecid D. Indomethacin E. Acetaminophen

SIMILAR TO PREVIOUS BOARD EXAM CONCEPT/PRINCIPLE

RAYMUND MARTIN LI, MD (TOP 1 - AUG 2014 MED BOARDS; TOPNOTCH MD)

MIDTERM 3 EXAM - FEB 2015

268

You are treating a patient HIV-patient for an opportunistic viral infection affecting the eyes. If you suspect viral strains that are thymidine-kinase deficient, you decide to choose an antiviral drug that is exclusively phosphorylated by host cell kinases but which doesn't require viral kinase phosphorylation. Which of the following will you choose? A. Acyclovir B. Ganciclovir C. Cidofovir D. Foscarnet E. Lamivudine

Cidofovir is phosphorylated by host kinases while Foscarnet doesn’t reqauire phosphorylation. Both are used in thymidine-deficient resistant strains of CMV

RAYMUND MARTIN LI, MD (TOP 1 - AUG 2014 MED BOARDS; TOPNOTCH MD)

MIDTERM 3 EXAM - FEB 2015

261

262

263

264

265

266

TOPNOTCH MEDICAL BOARD PREP PHARMACOLOGY SUPEREXAM Page 31 of 83 For inquiries visit www.topnotchboardprep.com.ph or email us at [email protected]

TOPNOTCH MEDICAL BOARD PREP PHARMACOLOGY SUPEREXAM For inquiries visit www.topnotchboardprep.com.ph or email us at [email protected] Item # 269

270

271

272

273

274

275

QUESTION

EXPLANATION

AUTHOR

TOPNOTCH EXAM MIDTERM 3 EXAM - FEB 2015

A 63 year old male patient presenting with intermittent complaints of chest pain upon walking several blocks or climbing several flights of stairs is prescribed with nitroglycerin to used on an as needed basis. What will be the expected physiologic changes as a result of this drug? A. Increased HR and and decreased left ventricular end-systolic volume B. Decreased HR and decreased left ventricular endsystolic volume C. Decreased HR and increased left ventricular endsystolic volume D. Increased HR and increased left ventricular endsystolic volume E. None of the above Which of the following mechanism indicates highlevel resistance in a TB patient being treated with Isoniazid? A. deletions in inhA gene that encodes target enzyme B. changes in drug sensitivity to RNA polymerase C. expression of drug efflux system D. expression of inactivating enzymes E. deletion in katG gene involved in the bioactivation of the drug A drug with diuretic activity is being studied based on its effects on electrolyte levels in the urine. It was found to moderately increase urine NaCl and urine K while decreasing urine Ca with associated slight increase in body pH and minimal change in urine HCO3-. This drug has characteristics similar to that of? A. Loop diuretic B. Carbonic anhydrase inhibitor C. K-sparing diuretic D. Thiazide diuretic E. Osmotic diuretic Which of the following steroid antagonists inhibits cytochrome P450 enxymes necessary for all steroids? A. Ketoconazole B. Aminogluthetimide C. Metyrapone D. Spinorolactone E. Mifepristone

Nitroglcerin decreases afterload, thus, decreasing end systolic volume. Compensatory sympathetic activity increases heart rate.

RAYMUND MARTIN LI, MD (TOP 1 - AUG 2014 MED BOARDS; TOPNOTCH MD)

E is INH high level resistance; A is INH low level resistance; B is rifampicin resistance

RAYMUND MARTIN LI, MD (TOP 1 - AUG 2014 MED BOARDS; TOPNOTCH MD)

MIDTERM 3 EXAM - FEB 2015

Thiazide increases Na and K excretion and causes metabolic alkalosis. It increases Ca reabsorption

RAYMUND MARTIN LI, MD (TOP 1 - AUG 2014 MED BOARDS; TOPNOTCH MD)

MIDTERM 3 EXAM - FEB 2015

Ketoconazole is a potent cytochrome p450 inhibitor. Aminoglutethimide inhibits conversion of cholesterol to pregnenolone. Metyrapone inhibits cortisol synthesis. Spinorolactone inhibits aldosterone. Mifepristone inhibits progesterone

RAYMUND MARTIN LI, MD (TOP 1 - AUG 2014 MED BOARDS; TOPNOTCH MD)

MIDTERM 3 EXAM - FEB 2015

A patient with warfarin as a maintenance medication is treated for pneumonia. Prothrombin time monitoring during treatment of the respiratory infection reveals elevated INR but it decrease back to baseline levels after treatment with the antibiotic is completed. Which of the following is most likely given? A. Amoxicillin B. Ceftriaxone C. Clindamycin D. Erythromycin E. TMP-SMX A COPD patient with severe concomitant heart condition is being prescribed a drug that will relieve his dyspnea and shortness of breath. Which of the following bronchodilators will be the primary choice? A. Salmeterol B. Salbutamol C. Terbutaline D. Atropine E. Ipratropium A physician prescribes a diabetic patient with an antidiabetic agent that activates a nuclear receptor that increases transcription of GLUT-4 transporters in adipose tissue. The drug prescribed is most likely: A. Methotrexate B. Insulin C. Pioglitazone D. Glyburide E. Sitagliptin

Erythromycin inhibits metabolism of warfarin causing an increase In INR

RAYMUND MARTIN LI, MD (TOP 1 - AUG 2014 MED BOARDS; TOPNOTCH MD)

MIDTERM 3 EXAM - FEB 2015

Ipratoropium is an anticholinergic used in COPD because of less cardiac effects

RAYMUND MARTIN LI, MD (TOP 1 - AUG 2014 MED BOARDS; TOPNOTCH MD)

MIDTERM 3 EXAM - FEB 2015

Thiozolidinediones - activates PPAR, a nuclear receptor to improve sensitivity to insulin

RAYMUND MARTIN LI, MD (TOP 1 - AUG 2014 MED BOARDS; TOPNOTCH MD)

MIDTERM 3 EXAM - FEB 2015

TOPNOTCH MEDICAL BOARD PREP PHARMACOLOGY SUPEREXAM Page 32 of 83 For inquiries visit www.topnotchboardprep.com.ph or email us at [email protected]

TOPNOTCH MEDICAL BOARD PREP PHARMACOLOGY SUPEREXAM For inquiries visit www.topnotchboardprep.com.ph or email us at [email protected] Item # 276

QUESTION

EXPLANATION

AUTHOR

TOPNOTCH EXAM MIDTERM 3 EXAM - FEB 2015

A patient presented to your clinic with complaints of dyspnea and chronic cough. He states that he frequently gets short of breath and is unable to take a deep breath. History is significant for prolonged treatment of rheumatoid arthritis. Chest X-ray revealed fine reticulonodular densities. Which drug is most likely responsible? A. Glucocorticoids B. Methotrexate C. Infliximab D. Cyclosporine E. Anakinra If you need to give a skeletal muscle relaxant to a burn patient, you will AVOID prescribing which of the following drugs? A. Pancuronium B. Baclofen C. Dantrolene D. Vecuronium E. Succinylcholine

Methotrexate is known to cause pulmonary fibrosis

RAYMUND MARTIN LI, MD (TOP 1 - AUG 2014 MED BOARDS; TOPNOTCH MD)

Succinylcholine is contraindicated in patient with rhabdomyolysis, hyperkalemia, burn patients, malignant hyperthermia

RAYMUND MARTIN LI, MD (TOP 1 - AUG 2014 MED BOARDS; TOPNOTCH MD)

MIDTERM 3 EXAM - FEB 2015

Which of the following medications exert its anticoagulant effect through the inactivation of thrombin? A. Fondaparinux B. Unfractionated heparin C. Low molecular weight heparin D. Apixaban E. All of the above One of the following drugs is effective in reducing the risk of ulcers in chronic NSAID users. Diarrhea is a frequent side effect. Which is it? A. Sucralfate B. Ranitidine C. Omeprazole D. Misoprostol E. Cimetidine

Fondaparinux and LMW heparin combines with antithrombin and selectively inhibits factor X but NOT thrombin. Apixaban is a direct Xa inhibitor

RAYMUND MARTIN LI, MD (TOP 1 - AUG 2014 MED BOARDS; TOPNOTCH MD)

MIDTERM 3 EXAM - FEB 2015



RAYMUND MARTIN LI, MD (TOP 1 - AUG 2014 MED BOARDS; TOPNOTCH MD)

MIDTERM 3 EXAM - FEB 2015

280

Knowledge of potency and solubility is important in general anesthetic medications. Which of the following characteristics refers to high potency? A. Low minimum alveolar concentration B. High minimum alveolar concentration C. Low blood:gas partition coefficient D. High blood:gas partition coefficient E. None of the above

Low MAC - high potency; High MAC - low potency; Low blood:gas coefficient - low solubility; High blood:gas coeeficient - high solubility

RAYMUND MARTIN LI, MD (TOP 1 - AUG 2014 MED BOARDS; TOPNOTCH MD)

MIDTERM 3 EXAM - FEB 2015

281

Which of the following anti hypertensive medications when taken during pregnancy can cause hypocalvaria in the fetus? A. Carvedilol B. Amlodipine C. Captopril D. HCTZ E. Hydralazine

Intake of ACE inhibitors can produce teratogenic effect in the fetus which includes renal dysgenesis and hypoplastic skulls (hypocalvaria) thus it is contraindicated during pregnancy

ERIC ROYD TALAVERA, MD (TOP 1 - AUG 2014 MED BOARDS; TOPNOTCH MD)

FINAL EXAM - FEB 2015

282

A 29 year old woman presented with amenorrhea, galactorrhea and loss of sexual libido. On evaluation her serum prolactin was noted to be markedly elevated. Which of the following medications is most useful in the treatment of her condition? A. Bromocriptine B. Cimetidine C. Sumatriptan D. Ergotamine E. Ondansetron

Bromocriptine is an effective dopamine agonist in the CNS with the advantage of oral activity. The drug inhibits prolactin secretion by activating pituitary dopamine receptors

ERIC ROYD TALAVERA, MD (TOP 1 - AUG 2014 MED BOARDS; TOPNOTCH MD)

FINAL EXAM - FEB 2015

283

A 47 year old obese male, who is a heavy alcoholic beverage drinker, was recently diagnosed with diabetes mellitus. He was started on Metformin 500 mg/tab TID. This patient is at risk of developing which of the following complications? A. Hypoglycemia B. Disulfiram like reaction C. Congestive heart failure D. Lactic acidosis E. Diarrhea

Metformin is associated with lactic acidosis. It should be avoided or used with extreme caution in patients who are heavy alcoholics since acute ethanol ingestion increaes the risk of lactic acidosis

ERIC ROYD TALAVERA, MD (TOP 1 - AUG 2014 MED BOARDS; TOPNOTCH MD)

FINAL EXAM - FEB 2015

277

278

279

TOPNOTCH MEDICAL BOARD PREP PHARMACOLOGY SUPEREXAM Page 33 of 83 For inquiries visit www.topnotchboardprep.com.ph or email us at [email protected]

TOPNOTCH MEDICAL BOARD PREP PHARMACOLOGY SUPEREXAM For inquiries visit www.topnotchboardprep.com.ph or email us at [email protected] Item # 284

285

286

287

288

289

290

QUESTION

EXPLANATION

AUTHOR

TOPNOTCH EXAM FINAL EXAM - FEB 2015

A 65 year old female presented to the ER due to progressive dyspnea. She is a known hypertesive but is poorly compliant with medications. On history, patient claims to experience orthopnea, paroxysmal nocturnal dyspnea and easy fatigability. On PE, her BP is 80/50. There is prominent neck vein distention. S3 gallop, bibasal crackle and a grade 3 bipedal edema were also appreciated. A 2d echo was done which showed a depressed ejection fraction of 32%. Which of the following medications should not be given at this time for this patient ? A. Furosemide B. Digoxin C. Metoprolol D. Dobutamine E. None of the above A 75 year old male was brought to the ER due to sudden onset of right sided weakness accompanied by a progressive deterioration in sensorium. On PE, BP was elevated at 220/100. Neurologic exam showed a GCS of 11, an MMT of 2/5 on the both right upper and lower extremities and a positive babinski on the right. A CT scan showed a massive intracerebral hemorrhage involving the left basal ganglia. A diuretic was started to decrease the patient's intracranial pressure. Which of the following is an associated adverse effect of this medication ? A. Ototoxicity B. Hyperlipidemia C. Gynecomastia D. Pulmonary Edema E. Thrombocytosis Which of the following is an orally active direct thrombin inhibitor? A. Apixaban B. Rivaroxaban C. Enoxaparin D. Fondaparinux E. Dabigatran

This is a case of an acute decompensated heart failure. Due to the marked systolic dysfunction as evident by a depressed ejection fraction, beta blockers are contraindicated at this time as it may further lower the cardiac output.

ERIC ROYD TALAVERA, MD (TOP 1 - AUG 2014 MED BOARDS; TOPNOTCH MD)

Mannitol is used to decrease the ICP in patient's with hemorrhagic stroke. It acts as an osmotic diuretic thus removes water from the intracellular compartment. This rapid fluid shifting may cause hyponatremia and pulmonary edema and as the water is excreted, hypernatremia may follow.

ERIC ROYD TALAVERA, MD (TOP 1 - AUG 2014 MED BOARDS; TOPNOTCH MD)

FINAL EXAM - FEB 2015

Apixaban and Rivaroxaban are both oral Factor Xa inhibitors. Enoxaparin and Fondaparinux are both SC factor Xa inhibior.

ERIC ROYD TALAVERA, MD (TOP 1 - AUG 2014 MED BOARDS; TOPNOTCH MD)

FINAL EXAM - FEB 2015

A 48 year old female, a known case of Acute Myelogenous Leukemia (AML), who is on reinduction chemotherapy was started on anti viral prophylaxis with Valacyclovir 500 mg/tab TID. What is the mechanism of action of this anti viral medication? A. Inhibition of Viral DNA polymerase B. Blockage of M2 proton channel C. Inhibition of neuraminidase D. Inhibition of viral reverse transcriptase E. Prevents fusion of virus with the host cellular membrane Which of the following drugs of abuse is non addictive? A. Amphetamine B. Cocaine C. Morphine D. LSD E. Heroin

Valaciclovir is a prodrug, an esterified version of aciclovir that has greater oral bioavailability (about 55%) than aciclovir (10–20%). It is phophorylated three times and this form acts as a competitive substrate for DNA polymerase and it leads to chain termination

ERIC ROYD TALAVERA, MD (TOP 1 - AUG 2014 MED BOARDS; TOPNOTCH MD)

FINAL EXAM - FEB 2015

Some drugs of abuse do not lead to addiction. This is the case for substances that alter perception without causing sensation of reward and euphoria. These include LSD, Phencyclidine and ketamine

ERIC ROYD TALAVERA, MD (TOP 1 - AUG 2014 MED BOARDS; TOPNOTCH MD)

FINAL EXAM - FEB 2015

A 70 year old male who is diagnosed with Colon Cancer Stage IV with metastasis to the liver who is currently on FOLFOX regimen was started on a monoclonal antibody which binds to the vascular endothelial growth factor (VEGF) receptor. What is the medication that was given to the patient? A. Imatinib B. Sorafenib C. Trastuzumab D. Bevacizumab E. Erlotinib Which of the following medications can be given once a week for prophylaxis against bactermia caused by M. avium complex (MAC) in AIDS patients? A. Azithromycin B. Kanamycin C. Ethambutol D. Rifampicin E. Cycloserine

Imatinib is a tyosine kinase inhibitor used in CML. Sorafenib inhibit multiple tyrosne kinase receptors and are primarily used in HCC. Trastuzumab inhibits cells that overexpress the Her-2/neu recepto in Breast CA. Erlotinib inhibits epidermal growth factor receptor and is used for non small cell lung CA and pancreatic cancer

ERIC ROYD TALAVERA, MD (TOP 1 - AUG 2014 MED BOARDS; TOPNOTCH MD)

FINAL EXAM - FEB 2015

because of its long elimination half life (3-4 days), weekly administration of azithromycin has proved to be equivalent to daily administration of clarithromycin when used for prophylaxis against MAC in AIDS

ERIC ROYD TALAVERA, MD (TOP 1 - AUG 2014 MED BOARDS; TOPNOTCH MD)

FINAL EXAM - FEB 2015

TOPNOTCH MEDICAL BOARD PREP PHARMACOLOGY SUPEREXAM Page 34 of 83 For inquiries visit www.topnotchboardprep.com.ph or email us at [email protected]

TOPNOTCH MEDICAL BOARD PREP PHARMACOLOGY SUPEREXAM For inquiries visit www.topnotchboardprep.com.ph or email us at [email protected] Item # 291

292

293

294

295

296

297

298

QUESTION

EXPLANATION

AUTHOR

A 37 year old female diagnosed with chronic schizophrenia maintained on an antipsychotic medication was brought to the clinic due to a 3 day history of fever. PE showed presence of tonsillar exudates. A CBC was requested which showed a markedly decreased WBC count. The medication was immediately discontinued by the patient's physician. What is the anti-psychotic medication that could have caused this problem? A. Haloperidol B. Quetiapine C. Clozapine D. Risperidone E. Olanzapine What is the major advantage of Etomidate over other intravenous anesthetics? A. Has greater analgesic effects B. Causes minimal cardiovascular and respiratory depression C. Relatively longer duration of effect D. Has a lower incidence of postoperative nausea and vomiting E. Has a rapid onset of recovery when compared with Propofol A 24 year old female G1P0, 12 weeks AOG, sought consult due to purulent vaginal discharge and dyspareunia. Gram stain of the vaginal discharge showed intracellular gram negative diplococci. The patient disclosed that she had a severe allergic reaction to amoxicillin a year ago. Which of the following is the safest antibiotic for this case ? A. Doxycyline B. Ciprofloxacin C. Ceftriaxone D. Gentamicin E. Azithromycin Which of the following is a major adverse effect associated with use of fenofibrates? A. Constipation B. Gallstones C. Hyperuricemia D. Cardiac arrythmia E. Liver damage

Clozapine can cause agranulocytosis in a small but significant number of patients (1-2%). This serious, potentially fatal effect can develop rapidly usually between the 6th to 18th week of treatment. It appears to be reversible upon discontinuation of treatment. Because of this risk, patient on clozapine must have weekly blood counts for the first 6 months of treatment and every 3 weeks thereafter

ERIC ROYD TALAVERA, MD (TOP 1 - AUG 2014 MED BOARDS; TOPNOTCH MD)

Etomidate has no analgesic effects. Distribution is rapid from the brain to highly perfused tissue hence a relatively short duration of its anesthetic effect. It has a high incidence of postoperative nausea and vomiting, pain and myoclonic activity. Initial recovery is less rapid compared with propofol

ERIC ROYD TALAVERA, MD (TOP 1 - AUG 2014 MED BOARDS; TOPNOTCH MD)

FINAL EXAM - FEB 2015

Cephalosporins should be avoided in patients with history of severe allergic reaction to penicillin. FQ and doxycycline are avoided in pregnancy since it can cause cartilage and bone damage. Azithromycin, a macrolide antibiotic is the safest drug for this case.

ERIC ROYD TALAVERA, MD (TOP 1 - AUG 2014 MED BOARDS; TOPNOTCH MD)

FINAL EXAM - FEB 2015

SIMILAR TO PREVIOUS BOARD EXAM CONCEPT/PRINCIPLE. A major toxicity of fenofibrates is increased risk of gallstone which may be due to enhanced biliary excretion

ERIC ROYD TALAVERA, MD (TOP 1 - AUG 2014 MED BOARDS; TOPNOTCH MD)

FINAL EXAM - FEB 2015

A 45 year old male presented to the ER due to sudden onset of right eye pain accompanied by blurring of vision. On examination there was note of ciliary injection over the right eye, IOP pressure was noted to be elevated. A diagnosis of acute angle closure glaucoma was made. Which of the following ophthalmic medications should be avoided ? A. Brinzolamide B. Timolol C. Apraclonidine D. Atropine E. All of the above Which of the following beta blockers possess intrinsic sympathomimetic activity? A. Metoprolol B. Esmolol C. Pindolol D. Acebutolol E. Both C and D

Atropine is a mydriatic and cyloplegic and can further increase IOP hence worsen angle closure glaucoma.

ERIC ROYD TALAVERA, MD (TOP 1 - AUG 2014 MED BOARDS; TOPNOTCH MD)

FINAL EXAM - FEB 2015

Acebutolol and Pindolol have partial agonist activity (ISA)

ERIC ROYD TALAVERA, MD (TOP 1 - AUG 2014 MED BOARDS; TOPNOTCH MD)

FINAL EXAM - FEB 2015

Which of the following drugs is used to induce ovulation in anovulatory women by selectively blocking estrogen receptors in the pituitary, thus increasing FSH and LH output? A. Raloxifene B. Clomiphene C. Fulvestrant D. Anastrozole E. Progestins A 39 year old female, known case of Graves' disease but poorly compliant with medications, presented to the ER due to palpitations and shortness of breath. She was hooked to a cardiac monitor which showed supraventricular tachycardia. Which of the following drugs would be most suitable for this case ? A. Esmolol B. Quinidine C. Flecainide D. Disopyramide E. Lidocaine

SIMILAR TO PREVIOUS BOARD EXAM CONCEPT/PRINCIPLE

ERIC ROYD TALAVERA, MD (TOP 1 - AUG 2014 MED BOARDS; TOPNOTCH MD)

FINAL EXAM - FEB 2015

beta blockers are the most effective agents in acute thyrotixc arrythmias . Esmolo is a rapid parenteral beta blocker

ERIC ROYD TALAVERA, MD (TOP 1 - AUG 2014 MED BOARDS; TOPNOTCH MD)

FINAL EXAM - FEB 2015

TOPNOTCH MEDICAL BOARD PREP PHARMACOLOGY SUPEREXAM Page 35 of 83 For inquiries visit www.topnotchboardprep.com.ph or email us at [email protected]

TOPNOTCH EXAM FINAL EXAM - FEB 2015

TOPNOTCH MEDICAL BOARD PREP PHARMACOLOGY SUPEREXAM For inquiries visit www.topnotchboardprep.com.ph or email us at [email protected] Item # 299

QUESTION

EXPLANATION

AUTHOR

TOPNOTCH EXAM FINAL EXAM - FEB 2015

A 67 year old woman, known diabetic for 20 years and maintained on Gliclazide, complained of severe bloating and post prandial fullness. Evaluation was done and she was assessed to have diabetic gastropathy. Which of the following medications would be most appropriate for this case? A. Cimetidine B. Metoclopramide C. Alosetron D. Loperamide E. Bismuth subsalicylate Which drug when taken together with Azathioprine has resulted in severe myelosupression by inhibiting the enzyme responsible for its metabolism ? A. Theophylline B. Celecoxib C. Allopurinol D. Digoxin E. Rifampicin

Metoclopramide is DOC for diabetic gastropathy

ERIC ROYD TALAVERA, MD (TOP 1 - AUG 2014 MED BOARDS; TOPNOTCH MD)

Azathioprine is converted to mercaptopurine which is responsible for its immunosuppressant and hematotoxicity. Allopurinol inhibits xanthine oxidase, the enzyme that metabolizes mercaptopurine.

ERIC ROYD TALAVERA, MD (TOP 1 - AUG 2014 MED BOARDS; TOPNOTCH MD)

FINAL EXAM - FEB 2015

301

During anaphylaxis, administration of IM epinephrine is essential because of its bronchodilatory effect in the lungs. How does epinephrine acts to produce such an effect in this particular condition? A. Chemical antagonist B. physiologic antagonist C. partial agonist D. noncompetitive antagonist E. none of the above

LEAN ANGELO SILVERIO, MD (TOP 4 - AUG 2014 MED BOARDS; TOPNOTCH MD), MD

BACK-UP MIDTERM EXAM - FEB 2015

302

which of the following drugs exhibit zero order kinetics ? A. Ethanol B. tolbutamide C. warfarin D. heparin E. All of the above

physiologic antagonist binds to a different receptor molecules producing an effect opposite to that produced by the drug it antagoniszes. It differs from a pharmacologic antagonist, which interacts with the same receptors as the drug it is inhibiting. the antagonism of bronchoconstrictor effect of histamine ( through histamine receptors) by epinephrine bronchodilator action ( beta receptors). chemical antagonist - reacts directly with the drug being antagonized to remove it or to prevent it from reaching its target. Katzung review 8th ed p 13. zero order kinetics refers to a contant rate of elimination of a drug regardless of its concentration. The following drugs exhibit zero order kinetics: aspirin, ethanol, warfarin,heparin, penytoin, tolbutamide, theophyline phenylbutazone, and salicylates.

LEAN ANGELO SILVERIO, MD (TOP 4 - AUG 2014 MED BOARDS; TOPNOTCH MD), MD

BACK-UP MIDTERM EXAM - FEB 2015

303

The following drugs promote induction of liver CYP enzymes except ? A. Rifampin B. Carbamazepine C. Phenylbutazone D. Disulfiram E. Glutethimide

CYP inducers enhances drug metabolism. Ex. Benzopyrene, carbamazepine, chlorcyclizine, glutethimide, griseofulvin, phenobarbital, phenylbutazone, phenytoim, rifampin, ritonavir ( chronic administration, acute -acts as inhibitor), st johns wort. Katzung 11th ed p 64-65.

LEAN ANGELO SILVERIO, MD (TOP 4 - AUG 2014 MED BOARDS; TOPNOTCH MD), MD

BACK-UP MIDTERM EXAM - FEB 2015

304

which of the following effects in parathion poisoning will not be antagonized by timely administration of pralidoxime? A. cyclospasm B. Urinary incontinence C. convulsion D. Negative dromotrophy E. None of the above

pralidoxime is a chemical antagonist acting as cholinesterase regenerator. It cannot enter the BBB because of its positively charged group and poor lipid solubility. Katzung review pp 69.

LEAN ANGELO SILVERIO, MD (TOP 4 - AUG 2014 MED BOARDS; TOPNOTCH MD), MD

BACK-UP MIDTERM EXAM - FEB 2015

305

Among the beta adrenergic blockers, which of the following will less likely to cause bradycardia and increased VLDL concentration? A. metoprolol B. nebivolol C. carteolol D. labetalol E. None of the above

pindolol, acebutolol, carteolol, bopindolol, penbutolol,celiprolol are the beta blockers having also partial b agonist activity ( intrinsic sympathomimetic activity). The advantage of this property is that it is less likely to cause elevation of plasma lipids without affecting its antihypertensive effect. Katzung 11th ed p 159.

LEAN ANGELO SILVERIO, MD (TOP 4 - AUG 2014 MED BOARDS; TOPNOTCH MD), MD

BACK-UP MIDTERM EXAM - FEB 2015

306

which of the following changes will promote digitalis induced cardiototoxicity? A. Acute tubular necrosis B. hypermagnesemia C. hyperparathyroidism D. Triamterene administration E. All of the above

any condition that can predisposed to hyperkalemia, hypermagnesemia and hypocalcemia will inhibit the binding of digitalis to Na K ATPase resulting to decrease toxicity. Triamterene and ATN can induce hyperkalemia. Katzung 11th ed p 216.

LEAN ANGELO SILVERIO, MD (TOP 4 - AUG 2014 MED BOARDS; TOPNOTCH MD), MD

BACK-UP MIDTERM EXAM - FEB 2015

307

which of the following drugs is highly selective to ischemic or prolonged depolarized purkinje fibers? A. Mexiletine B. Procainamide C. Propafenone D. ibutilide E. None of the above

phenytoin, lidocaine and mexiletine are classified as class IB antiarrhythmics. This group affects primarily ischemic or depolarized ventricular tissue and does not have any use in atrial arrhytmia. B- procainamide is a class IA drug, C- propafenone is a Class IB agent, D- ibutilide is class III drugs.

LEAN ANGELO SILVERIO, MD (TOP 4 - AUG 2014 MED BOARDS; TOPNOTCH MD), MD

BACK-UP MIDTERM EXAM - FEB 2015

300

TOPNOTCH MEDICAL BOARD PREP PHARMACOLOGY SUPEREXAM Page 36 of 83 For inquiries visit www.topnotchboardprep.com.ph or email us at [email protected]

TOPNOTCH MEDICAL BOARD PREP PHARMACOLOGY SUPEREXAM For inquiries visit www.topnotchboardprep.com.ph or email us at [email protected] Item # 308

309

310

QUESTION

EXPLANATION

AUTHOR

which of the following effects of acetazolamide is self limiting? A. Sodium reabsorption B. Tubular bicarbonate excretion C. CSF acidosis D. Aqueous humor production E. None of the above

the major effect of acetazolamide is bicarbonate diuresis resulting to metabolic acidosis. As increased sodium is presented to the cortical collecting tubule, some of the excess sodium is reabsorbed and potassium is secreted resulting to significant potassium wasting. as a result of bicarbonate depletion, sodium bicarbonate excretion slows even with continued administration. Katzung review 8th ed p 135 dermanent discoloration of the iris and eyelashes is a well known side effect of latanoprost. Its mechanism of action is to increase aqueous outflow. A- brinzolamide, dorsolamide, B- epinephrine. D- timolol. Katzung review p 87.

LEAN ANGELO SILVERIO, MD (TOP 4 - AUG 2014 MED BOARDS; TOPNOTCH MD), MD LEAN ANGELO SILVERIO, MD (TOP 4 - AUG 2014 MED BOARDS; TOPNOTCH MD), MD

BACK-UP MIDTERM EXAM - FEB 2015

Olanzapine -weight gain, clozapine - agranulocytosis, haloperidol- extrapyramidal symptoms. Thioridazine - retinal deposits. Among the atypical antipsychotics, ziprasidone has the greatest risk of inducing QT prolongation leading to ventricular arrhythmias. Kazung review p 242.

LEAN ANGELO SILVERIO, MD (TOP 4 - AUG 2014 MED BOARDS; TOPNOTCH MD), MD

BACK-UP MIDTERM EXAM - FEB 2015

Buspirone is a selective anxiolytic medication that does not cause, sedative, hypnotic, euphoric, and acute withdrawal or rebound effect. It has no effect in psychomotor skills (driving skills). The mechanism of anxiolysis is secondary to partial antagonism to 5HT1A receptors in the brain. it has no direct GABAergic activity. weeks are needed before buspirone takes effect, therefore, it is not effect in treating acute panic attacks. Katzung 11ed p374. single dose administration of spectinomycin is effective in treating Gonnorhea in a patient who has severe beta lactam hypersensitivity. It has lesser risk of teratogenicity compared to conventional aminglycosides. Clindamycin is active only on gram positive and anaerobic organisms. ceftriaxone is a 3rd generation cephalosporin effective against Gonorrhea infection however it should be avoided since the patient has beta lactam hypersensitivity. Doxycycline should never be given for pregnant patients. erythromycin is not effective treatment for gonorrhea infection. Katzung 11th ed p 813. (SIMILAR TO PREVIOUS BOARD EXAM CONCEPT/PRINCIPLE) Katzung review 8th ed p 211

LEAN ANGELO SILVERIO, MD (TOP 4 - AUG 2014 MED BOARDS; TOPNOTCH MD), MD

BACK-UP MIDTERM EXAM - FEB 2015

LEAN ANGELO SILVERIO, MD (TOP 4 - AUG 2014 MED BOARDS; TOPNOTCH MD), MD

BACK-UP MIDTERM EXAM - FEB 2015

LEAN ANGELO SILVERIO, MD (TOP 4 - AUG 2014 MED BOARDS; TOPNOTCH MD), MD

BACK-UP MIDTERM EXAM - FEB 2015

A 39 y/o male went for follow up consult secondary to open angle glaucoma. He was compliant with all the topical medications given to him. Indirect ophthalmoscopy revealed brownish discoloration in his iris. What is the mechanism of action of the drug that causes such change?. A. decrease aqueous secretion due to lack of HCO3 B. increase outflow via uveoscleral veins C. increase aqueous outflow D. decrease aqueous secretion from the ciliary epithelium E. none of the above which in the following antipsychotic medications has the greatest risk of inducing Torsades de pointes arryhthmia? A. Fluphenazine B. Clozapine C. Thioridazine D. Ziprasidone E. Haloperidol

TOPNOTCH EXAM BACK-UP MIDTERM EXAM - FEB 2015

311

Which of the following is true about Buspirone? A. Anxiolytic effect is secondary to direct stimulation of GABAergic neurons B. It is highly effect in acute panic attacks C. Less psychomotor retardation D. Abrupt discontinuation can lead acute withdrawal syndrome E. all of the above

312

what is the recommended treatment for a 29 y/o G1PO female diagnosed with Gonnorrhea infection who has severe hypersensitivity to pennicilin? A. erythromycin B. Spectinomycin C. Cetriaxone D. Doxycycline E. Clindamycin

313

the pungency of this inhaled anesthetics leading breath-holding limits it use in anesthesia induction? A. desflurane B. Isoflurane C. Sevoflurane D. enflurane E. Halothane

314

what is the drug of choice for restless leg syndrome? A. haloperidol B. aripriprazole C. Citalopram D. Ropinirole E. Propanolol

the pathologic basis for restless leg syndrome is dopamine blockade. Agonist to dopamine receptors such as pramipexole or ropinirole is very effective in treating this condition. Katzung review 8th ed p 235

LEAN ANGELO SILVERIO, MD (TOP 4 - AUG 2014 MED BOARDS; TOPNOTCH MD), MD

BACK-UP MIDTERM EXAM - FEB 2015

315

Prolonged treatment with a PPAR alpha ligand in the treatment of hyperlipidemia will increase the risk for the following condition? A. Cutaneous flushing B. Hyperuricemia leading to Gout C. Intestinal malabsorption D. Gallstones E. hepatotoxicity

PPAR alpha ligands is the MOA of fibric acid derivatives ( Gemfibrozil and fenofibrate). It increases the supersaturation of bile leading cholelithiasis. Risk for hepatotoxicity only significantly increase if gemfibrozil is combined with HMG CoA inhibitor. Cutaneous flushing and hyperuricemia are toxicity profile of Niacin. Intestinal malabsorption is a primary side effect of resins ( cholestyramine, colestipol, colsevelam)

LEAN ANGELO SILVERIO, MD (TOP 4 - AUG 2014 MED BOARDS; TOPNOTCH MD), MD

BACK-UP MIDTERM EXAM - FEB 2015

TOPNOTCH MEDICAL BOARD PREP PHARMACOLOGY SUPEREXAM Page 37 of 83 For inquiries visit www.topnotchboardprep.com.ph or email us at [email protected]

TOPNOTCH MEDICAL BOARD PREP PHARMACOLOGY SUPEREXAM For inquiries visit www.topnotchboardprep.com.ph or email us at [email protected] Item # 316

QUESTION

EXPLANATION

AUTHOR

oral administration of this antibiotic will yield greater blood levels compared when it is administered intravenously? A. chlorampenicol B. Quinipristin -dalfopristin C. Azithromycin D. Telithromycin E. None of the above

the pharmacokinetics of chlorampenicol is unusual since blood levels are higher when the drug is given at oral form rather than intravenous form. Katzung 11th ed page 802

LEAN ANGELO SILVERIO, MD (TOP 4 - AUG 2014 MED BOARDS; TOPNOTCH MD), MD

317

what is the mechanism of action of nystatin? A. Formation of artificial pore in the fungal membrane B. Inhibit ergosterol synthesis C. Inhibit microtubular formation D. Inhibits squalene epoxidase E. Inhibit the synthesis of B(1-2) glycan

nystatin and ampothericin B are both polyene antifungal antibiotics. It binds to ergosterol and creates an artificial pores in the fungal membrane leading to increase membrane permeability. Katzung review 8th ed p 401

LEAN ANGELO SILVERIO, MD (TOP 4 - AUG 2014 MED BOARDS; TOPNOTCH MD), MD

BACK-UP MIDTERM EXAM - FEB 2015

318

the drug of choice for Chagas disease acts through what mechanism of action? A. Inhibit mitochondrial electron transport B. Trypanothione reductase inhibition C. Suicide substrate of ornithine decarboxylase D. Inhibits enzyyme sulfhydryl groups E. none of the above

LEAN ANGELO SILVERIO, MD (TOP 4 - AUG 2014 MED BOARDS; TOPNOTCH MD), MD

BACK-UP MIDTERM EXAM - FEB 2015

319

what is the primary drug of treatment for ascariasis, pinworm, whipworm and hydatid disease? A. albendazole B. Mebendazole C. Pyrantel pamoate D. Diethylcarbamazine E. All of the above

the drug of choice for Chagas disease or american trypanosomiasis is nifurtimox. Its anitprotozoan action is due to inhibition of trypanothiane reductase. A- atovaquone MOA ( use for PCP and chloroquine resistant malaria) C- eflornithine ( use in cerebral stage of african trypanosomiasis). D- melarsoprol ( african sleeping sickness). SIMILAR TO PREVIOUS BOARD EXAM CONCEPT/PRINCIPLE Katzung review p 438-439. B,C,D has no clinical effectivity in hydatid disease.

LEAN ANGELO SILVERIO, MD (TOP 4 - AUG 2014 MED BOARDS; TOPNOTCH MD), MD

BACK-UP MIDTERM EXAM - FEB 2015

320

what is the dose limiting toxicity of vincristine ? A. areflexia B. Bone marrow suppression C. Acute tubular necrosis D. Hepatocyte degeneration E. None of the above

vincristine does not cause serious myelosuppresive effect however it induces neurotoxicity such as areflexia, ileus and peripheral neuritis. Katzung review 8th ed p 455.

LEAN ANGELO SILVERIO, MD (TOP 4 - AUG 2014 MED BOARDS; TOPNOTCH MD), MD

BACK-UP MIDTERM EXAM - FEB 2015

321

Among insulin secretagogues, the risk of hypoglycemia is least in? A. Meglitinides B. 1st generation sulfonylureas C. 2nd generation sulfonylureas D. Biguanides

Biguanides are not insulin secretagogues.

DEBBIE ROSE TANENGSY, MD (TOP 5 - AUG 2014 MED BOARDS; TOPNOTCH MD)

BACK-UP MIDTERM EXAM - FEB 2015

322

What is the most important drug in pulmonary tuberculosis? A. INH B. Pyrazinamide C. Rifampin D. Ethambutol



DEBBIE ROSE TANENGSY, MD (TOP 5 - AUG 2014 MED BOARDS; TOPNOTCH MD)

BACK-UP MIDTERM EXAM - FEB 2015

323

Pupillary construction is a characteristic effet of all opioids, except: A. Oxycodone B. Meperidine C. Morphine D. Fentanyl



DEBBIE ROSE TANENGSY, MD (TOP 5 - AUG 2014 MED BOARDS; TOPNOTCH MD)

BACK-UP MIDTERM EXAM - FEB 2015

324

This antiplatelet inhibits aggregation by interfering with GpIIb/IIIa binding to fibrinogen and other ligands: A. Aspirin B. Clopidogrel C. Dipyridamole D. Tirofiban

Aspirin is a nonselective irreversible COX inhibitor. Clopidogrel irreversibly inhibits platelet ADP receptor. Dipyridamole inhibits adenosine uptake and inhibits phosphodiesterase enzymes that degrade cyclic nucleotides.

DEBBIE ROSE TANENGSY, MD (TOP 5 - AUG 2014 MED BOARDS; TOPNOTCH MD)

BACK-UP MIDTERM EXAM - FEB 2015

325

Which tetracycline has the broadest spectrum of microbial coverage? A. Tetracyline B. Doxycyline C. Minocycline D. Tigecycline



DEBBIE ROSE TANENGSY, MD (TOP 5 - AUG 2014 MED BOARDS; TOPNOTCH MD)

BACK-UP MIDTERM EXAM - FEB 2015

326

The following drugs' efficacy is directly related to time above the minimal inhibitory concentration and becomes independent of concentration once the MIC has been reached, except? A. Penicillins B. Cephalosporins C. A & B D. none of the above

What is described is the time-dependent killing property for which penicillins and cephalosporins are known. Aminoglycosides are known for their concentration-dependent killing.

DEBBIE ROSE TANENGSY, MD (TOP 5 - AUG 2014 MED BOARDS; TOPNOTCH MD)

BACK-UP MIDTERM EXAM - FEB 2015

TOPNOTCH MEDICAL BOARD PREP PHARMACOLOGY SUPEREXAM Page 38 of 83 For inquiries visit www.topnotchboardprep.com.ph or email us at [email protected]

TOPNOTCH EXAM BACK-UP MIDTERM EXAM - FEB 2015

TOPNOTCH MEDICAL BOARD PREP PHARMACOLOGY SUPEREXAM For inquiries visit www.topnotchboardprep.com.ph or email us at [email protected] Item # 327

QUESTION

EXPLANATION

AUTHOR

TOPNOTCH EXAM BACK-UP MIDTERM EXAM - FEB 2015

The following are bactericidal, except: A. TMP-SXZ B. Vancomycin C. Metronidazole D. Cefalexin E. none of the above

All drugs stated are bactericidal. TMP and SXZ alone are bacteriostatic but are bactericidal when in combination.

DEBBIE ROSE TANENGSY, MD (TOP 5 - AUG 2014 MED BOARDS; TOPNOTCH MD)

The following anti-tuberculosis drugs are correctly paired with their mechanisms of action, except: A. INH - inhibits mycolic acid synthesis B. Rifampin - inhibits RNA dependent DNA polymerase C. Ethambutol - inhibits arabinosyl transferase D. none of the above This drug requires phosphorylation by host cell kinases to be active: A. Efavirenz B. Delavirdine C. Nevirapine D. Tenofovir

Rifampicin inhibits DNA dependent RNA polymerase

DEBBIE ROSE TANENGSY, MD (TOP 5 - AUG 2014 MED BOARDS; TOPNOTCH MD)

BACK-UP MIDTERM EXAM - FEB 2015

Efavirenz, delaverdine, and nevirapine are non nucleoside reverse transcriptase inhibitors which do not require host cell kinase for activation, unlike tenofovir, a nucleoside reverse transcriptase inhibitor which does.

DEBBIE ROSE TANENGSY, MD (TOP 5 - AUG 2014 MED BOARDS; TOPNOTCH MD)

BACK-UP MIDTERM EXAM - FEB 2015

330

This antihistamine causes depolarization-induced paralysis in nematodes: A. Mebendazole B. Ivermectin C. Pyrantel pamoate D. Piperazine

Mebendazole selectively inhibits microtubule synthesis and glucose uptake. Ivermectin interferes with GABA mediated neurotransmission. Piperazine is an agonist at GABA receptors.

DEBBIE ROSE TANENGSY, MD (TOP 5 - AUG 2014 MED BOARDS; TOPNOTCH MD)

BACK-UP MIDTERM EXAM - FEB 2015

331

Which of the following is uricosuric? A. Allopurinol B. Colchicine C. Sulfinpyrazone D. Celecoxib

Allopurinol is a xanthine oxidase inhibitor. Colchicine inhibits microtubule assembly. Celecoxib is a COX-2 inhibitor.

DEBBIE ROSE TANENGSY, MD (TOP 5 - AUG 2014 MED BOARDS; TOPNOTCH MD)

BACK-UP MIDTERM EXAM - FEB 2015

332

Which drug is most effective in raising HDL? A. Niacin B. Fibrates C. Statins D. Bile acid sequestrants



DEBBIE ROSE TANENGSY, MD (TOP 5 - AUG 2014 MED BOARDS; TOPNOTCH MD)

BACK-UP MIDTERM EXAM - FEB 2015

333

The metabolism of which drug is inhibited by allopurinol and febuxostat? A. Cytarabine B. Methotrexate C. Fluorouracil D. Mercaptopurine

Allopurinol inhibits the enzymatic inactivation of 6MP and its derivative, Azathioprine, by xanthine oxidase. Thus, when allopurinol is used concurrently with oral 6-MP/azathioprine, dosage of the antineoplastic agent must be reduced to 1/2 to 1/3 of usual dose.

DEBBIE ROSE TANENGSY, MD (TOP 5 - AUG 2014 MED BOARDS; TOPNOTCH MD)

BACK-UP MIDTERM EXAM - FEB 2015

334

This regimen is utilized in the treatment of Hodgkin lyphoma: A. PEB regimen B. ABVD regimen C. CHOP regimen D. GnRH agonist and androgen receptor antagonist

DEBBIE ROSE TANENGSY, MD (TOP 5 - AUG 2014 MED BOARDS; TOPNOTCH MD)

BACK-UP MIDTERM EXAM - FEB 2015

335

This drug is a monoclonal antibody that binds to VEGF and prevents it from interacting with VEGF receptors: A. Bevacuzimab B. Rituximab C. Trastuzumab D. Imatinib

ABVD - adriamycin, bleomycin, vincristine, dacarbazine, prednisone. PEB (cisplatin, etoposide, bleomycin) regimen is used in testicular CA. CHOP (cyclophosphamide, doxorubicin, vincristine, prednisone) regimen in non-Hodgkin lymphoma, and GnRH agonist and androgen receptor antagonist in prostate CA treatment. Rituximab is a monoclonal antibody that binds to surface protein in NHL inducing lysis, cytotoxicity and apoptosis. Trastuzumab is a monoclonal antibody that recognizes protein in breast CA cells overexpressing HER-2neu receptor for EGF. Imatinib inhibits tyrosine kinase activity of the product of bcr-abl oncogene expressed in CML.

DEBBIE ROSE TANENGSY, MD (TOP 5 - AUG 2014 MED BOARDS; TOPNOTCH MD)

BACK-UP MIDTERM EXAM - FEB 2015

336

This chemotherapeutic agent is known for its association with heart toxicity in cumulative doses: A. 5-FU B. 6-MP C. Methotrexate D. Doxorubicin



DEBBIE ROSE TANENGSY, MD (TOP 5 - AUG 2014 MED BOARDS; TOPNOTCH MD)

BACK-UP MIDTERM EXAM - FEB 2015

337

The following laxative mechanisms are correctly paired with its representative laxative drug, except: A. bulk forming - psyllium B. stool softening - docusate C. osmotic - lactulose D. stimulant - polyethylene glycol

Polyethylene glycol is an osmotic laxative. Examples of stimulant laxatives include senna, bisacodyl, castor oil.

DEBBIE ROSE TANENGSY, MD (TOP 5 - AUG 2014 MED BOARDS; TOPNOTCH MD)

BACK-UP MIDTERM EXAM - FEB 2015

338

Ondansetron, highly effective in preventing chemotherapy-induced nausea and vomiting, is an antagonist at which receptor? A. muscarinic B. B2 C. 5-HT3 D. D2



DEBBIE ROSE TANENGSY, MD (TOP 5 - AUG 2014 MED BOARDS; TOPNOTCH MD)

BACK-UP MIDTERM EXAM - FEB 2015

328

329

TOPNOTCH MEDICAL BOARD PREP PHARMACOLOGY SUPEREXAM Page 39 of 83 For inquiries visit www.topnotchboardprep.com.ph or email us at [email protected]

TOPNOTCH MEDICAL BOARD PREP PHARMACOLOGY SUPEREXAM For inquiries visit www.topnotchboardprep.com.ph or email us at [email protected] Item # 339

QUESTION

EXPLANATION

This drug stimulates platelet production and decreases the number of platelet transfusions required by patients undergoing bone marrow suppression in treatment for CA: A. Oprevelkin B. Filgrastim C. Erythropoietin D. Folic acid Serum concentration of lithium in mEq/L favored for acutely manic patients is: A. 0.9-1.1 B. 0.6-0.75 C. 0.5-0.85 D. 0.1-0.5

Filgrastim stimulates the function and production of neutrophils.

DEBBIE ROSE TANENGSY, MD (TOP 5 - AUG 2014 MED BOARDS; TOPNOTCH MD)

The serum concentration favored for long-term use in the prevention of reccurent manic-depressive illness, on the other hand, is 0.6-0.75 mEq/L.

DEBBIE ROSE TANENGSY, MD (TOP 5 - AUG 2014 MED BOARDS; TOPNOTCH MD)

BACK-UP MIDTERM EXAM - FEB 2015

This direct acting cholinomimetic is used in the treatment of postoperative ileum and neurogenic bladder: A. Neostigmine B. Bethanechol C. Pilocarpine D. Physostigmine Use of aspirin as an anti-platelet inhibits primarily which prostanoid? A. Prostacyclin B. Thromboxane C. Prostaglandin D. Epoprosterenol



JESSICA MAE SANCHEZ, MD (TOP 4 - AUG 2014 MED BOARDS; TOPNOTCH MD)

BACK-UP MIDTERM EXAM - FEB 2015



JESSICA MAE SANCHEZ, MD (TOP 4 - AUG 2014 MED BOARDS; TOPNOTCH MD)

BACK-UP MIDTERM EXAM - FEB 2015

343

This is not a characteristic of phase I clinical trials in drug development: A. Pharmacokinetic measurements are done B. Determination of drug’s efficacy C. Determination of maximum tolerated dose D. Subjects are healthy volunteers

Determination of drug efficacy is done during phase II of clinical trials.

JESSICA MAE SANCHEZ, MD (TOP 4 - AUG 2014 MED BOARDS; TOPNOTCH MD)

BACK-UP MIDTERM EXAM - FEB 2015

344

The prime target of addictive drugs in the brain: A. Thalamocortical system B. Nigrostriatal dopamine system C. Mesolimbic dopamine system D. Mesocortical dopamine system

Reference: Katzung. Basic and Clinical Pharmacology, 11th ed. p. 554

JESSICA MAE SANCHEZ, MD (TOP 4 - AUG 2014 MED BOARDS; TOPNOTCH MD)

BACK-UP MIDTERM EXAM - FEB 2015

345

A 30 year old female with bipolar disorder delivered a baby with Ebstein anomaly. The drug that the mother was taking during her pregnancy that caused this congenital anomaly is: A. Olanzapine B. Lithium C. Carbamazepine D. Risperidone Ketamine exerts its anaesthetic effect by its inhibitory action on these receptors: A. NMDA B. GABA-A C. Glycine D. Neuronal nictonic acetylcholine receptors



JESSICA MAE SANCHEZ, MD (TOP 4 - AUG 2014 MED BOARDS; TOPNOTCH MD)

BACK-UP MIDTERM EXAM - FEB 2015



JESSICA MAE SANCHEZ, MD (TOP 4 - AUG 2014 MED BOARDS; TOPNOTCH MD)

BACK-UP MIDTERM EXAM - FEB 2015

A 20 year old male develops tachycardia, hypotension, muscle rigidity, and spiking fever immediately after start of surgery. Which of the following are commonly associated with this anaesthetic complication? A. IV anesthetic and succinylcholine B. IV anesthetic and midazolam C. Inhalational anesthetic and succinylcholine D. Inhalational anesthetic and midazolam Nitrates relieve angina by: A. Decreasing venous return to the heart B. Increasing intracardiac volume C. Increasing end diastolic left ventricular volume D. All of the above

This is a case of malignant hyperthermia, which is associated with the combination of inhaled anaesthetics and succinylcholine.

JESSICA MAE SANCHEZ, MD (TOP 4 - AUG 2014 MED BOARDS; TOPNOTCH MD)

BACK-UP MIDTERM EXAM - FEB 2015



JESSICA MAE SANCHEZ, MD (TOP 4 - AUG 2014 MED BOARDS; TOPNOTCH MD)

BACK-UP MIDTERM EXAM - FEB 2015

349

The main indication of this agent is hypertension in pregnancy: A. Reserpine B. Prazosin C. Methyldopa D. Clonidine



JESSICA MAE SANCHEZ, MD (TOP 4 - AUG 2014 MED BOARDS; TOPNOTCH MD)

BACK-UP MIDTERM EXAM - FEB 2015

350

This antihypertensive drug is absolutely contraindicated in pregnancy because it can cause renal damage in the fetus: A. Reserpine B. Hydralazine C. Propranolol D. Captopril



JESSICA MAE SANCHEZ, MD (TOP 4 - AUG 2014 MED BOARDS; TOPNOTCH MD)

BACK-UP MIDTERM EXAM - FEB 2015

340

341

342

346

347

348

AUTHOR

TOPNOTCH MEDICAL BOARD PREP PHARMACOLOGY SUPEREXAM Page 40 of 83 For inquiries visit www.topnotchboardprep.com.ph or email us at [email protected]

TOPNOTCH EXAM BACK-UP MIDTERM EXAM - FEB 2015

TOPNOTCH MEDICAL BOARD PREP PHARMACOLOGY SUPEREXAM For inquiries visit www.topnotchboardprep.com.ph or email us at [email protected] Item # 351

QUESTION

EXPLANATION

AUTHOR

TOPNOTCH EXAM BACK-UP MIDTERM EXAM - FEB 2015

Which class of antibiotics can be safely given during pregnancy? A. Tetracycline B. Aminoglycoside C. Fluoroquinolones D. Beta-lactams



JESSICA MAE SANCHEZ, MD (TOP 4 - AUG 2014 MED BOARDS; TOPNOTCH MD)

352

Vancomycin is administered orally for this condition: A. Sepsis B. Endocarditis C. Antibiotic-associated enterocolitis D. Meningitis



JESSICA MAE SANCHEZ, MD (TOP 4 - AUG 2014 MED BOARDS; TOPNOTCH MD)

BACK-UP MIDTERM EXAM - FEB 2015

353

These anti-neoplastic agents are cell-cycle specific, except: A. 5-fluorouracil B. Cyclophosphamide C. Bleomycin D. Vincristine



JESSICA MAE SANCHEZ, MD (TOP 4 - AUG 2014 MED BOARDS; TOPNOTCH MD)

BACK-UP MIDTERM EXAM - FEB 2015

354

The preferred anti-thyroid drug for thyroid storm: A. Methimazole B. Carbimazole C. Propylthiouracil D. Propranolol



JESSICA MAE SANCHEZ, MD (TOP 4 - AUG 2014 MED BOARDS; TOPNOTCH MD)

BACK-UP MIDTERM EXAM - FEB 2015

355

Which of the following is a contraindication to the use of thiazolidinediones? A. Alcohol ingestion B. Heart failure C. Cholelithiasis D. Renal insufficiency

Reference: Katzung. Basic and Clinical Pharmacology, 11th ed. p. 743

JESSICA MAE SANCHEZ, MD (TOP 4 - AUG 2014 MED BOARDS; TOPNOTCH MD)

BACK-UP MIDTERM EXAM - FEB 2015

356

Clomiphene acts to induce ovulation by: A. Diminishing estrogen mediated negative feedback at the pituitary B. Increasing the action of estrogen in the hypothalamus C. Increasing the action of estrogen in the ovary D. Increasing the amount of estrogen receptors Organophosphate toxicity is based on its ability to: A. Compete with brain pyridoxal phosphate B. Inhibit acetylcholinesterase C. Interrupt heme synthesis D. React with sulfhydryl groups binding to protein and other enzymes



JESSICA MAE SANCHEZ, MD (TOP 4 - AUG 2014 MED BOARDS; TOPNOTCH MD)

BACK-UP MIDTERM EXAM - FEB 2015



JESSICA MAE SANCHEZ, MD (TOP 4 - AUG 2014 MED BOARDS; TOPNOTCH MD)

BACK-UP MIDTERM EXAM - FEB 2015

A drug used for treatment of advanced prostatic carcinoma by producing continuous gonadal suppression: A. Ketoconazole B. Testosterone undecanoate C. Leuprolide D. Cyproterone acetate The following drug/s is/are used to augment labor: A. Oxytocin B. Misoprostol C. Methyl ergonovine D. All of the above

Leuprolide combined with an androgen receptor antagonist such as flutamide is the primary medical therapy for advanced prostate cancer and is as effective as surgical castration. Reference: Katzung. Basic and Clinical Pharmacology, 11th ed. p. 654

JESSICA MAE SANCHEZ, MD (TOP 4 - AUG 2014 MED BOARDS; TOPNOTCH MD)

BACK-UP MIDTERM EXAM - FEB 2015

JESSICA MAE SANCHEZ, MD (TOP 4 - AUG 2014 MED BOARDS; TOPNOTCH MD)

BACK-UP MIDTERM EXAM - FEB 2015

360

Type II DM with insulin resistance will be best treated with which of the following: A. Sulfonylureas B. Gliptins C. Insulin glargine D. TZDs



JESSICA MAE SANCHEZ, MD (TOP 4 - AUG 2014 MED BOARDS; TOPNOTCH MD)

BACK-UP MIDTERM EXAM - FEB 2015

361

A 26/M presented with RUQ pain and fever. Imaging showed an abscess and aspiration biopsy was done. The aspirate resembled anchovy paste in color and in consistency. What is/are the possible drug/s that you can give for this patient? A. Metronidazole B. Tinidazole C. Chloroquine D. A and B E. All of the above

MAIRRE JAMES GADDI, MD (TOP 4 - AUG 2013 MED BOARDS; TOPNOTCH MD)

BACK-UP MIDTERM EXAM - FEB 2015

362

Which antifungal drug inhibits β-glucan synthase causing a disruption in cell wall synthesis? A. Terbinafine B. Ketoconazole C. Amphotericin B D. Fluconazole E. Caspofungin

For cases of amebic hepatic abscess you can give either Metronidazole, 750 mg 3 times daily (or 500 mg IV every 6 hours) for 10 days or Tinidazole, 2 g daily for 5 days PLUS a luminal agent. Alternatively you can give Dehydroemetine2 or emetine,2 1 mg/kg SC or IM for 8–10 days, followed by (liver abscess only) chloroquine, 500 mg twice daily for 2 days, then 500 mg daily for 21 days PLUS a luminal agent Luminal agent (Diloxanide, iodoquinol, paramomycin) Katzung 12th ed pg 928 Terbinafine - inhibits squalene oxidase; Ketoconazole - inhibits synthesis of ergosterol; Fluconazole - inhibits thymidylate synthase; Ampho B - polyene causes disruption of fungal cell walls Katzung 12th ed pg 857

MAIRRE JAMES GADDI, MD (TOP 4 - AUG 2013 MED BOARDS; TOPNOTCH MD)

BACK-UP MIDTERM EXAM - FEB 2015

357

358

359

TOPNOTCH MEDICAL BOARD PREP PHARMACOLOGY SUPEREXAM Page 41 of 83 For inquiries visit www.topnotchboardprep.com.ph or email us at [email protected]

TOPNOTCH MEDICAL BOARD PREP PHARMACOLOGY SUPEREXAM For inquiries visit www.topnotchboardprep.com.ph or email us at [email protected] Item # 363

364

365

366

367

368

369

370

QUESTION

EXPLANATION

AUTHOR

TOPNOTCH EXAM BACK-UP MIDTERM EXAM - FEB 2015

A 65/M hypertensive, diabetic, smoker presents to you with fever, productive cough with an infected non healing wound in the foot. Labs showed anemia and very elevated creatinine.You schedule the patient for dialysis. As the prescribing physician, which of the following drug/s dose/s would you have to modify? A. Vancomycin B. Meropenem C. Metoprolol D. A and B E. All of the above

Vancomycin and meropenem need to be adjusted Katzung 12th ed pg 798 Metoprolol is metabolized in the liver and requires no adjustment pg 179

MAIRRE JAMES GADDI, MD (TOP 4 - AUG 2013 MED BOARDS; TOPNOTCH MD)

A 34/F was diagnosed to have lepromatous leprosy. She was given the appropriate drugs which she took religiously. One day she came back to you complaining that her skin changed to a red-brownish color. Which of the following drug/s most likely have caused her predicament? A. Dapsone B. Rifampicin C. Clofazimine D. Rifabutin E. Cycloserine Which of the following drug/s can be used for eliminating meningococcal carriage states? A. Minocycline B. Pen G C. Rifampin D. Demeclocycline E. A and C

Clofazimine is a phenazine dye which causes redbrown to nearly black skin discoloration. SE of dapsone: methemoglobinemia; Rifampicin: red orange discoloration of body fluid Katzung 12th ed pg 846

MAIRRE JAMES GADDI, MD (TOP 4 - AUG 2013 MED BOARDS; TOPNOTCH MD)

BACK-UP MIDTERM EXAM - FEB 2015

Minocycline, 200 mg orally daily for 5 days, can eradicate the meningococcal carrier state, but because of side effects and resis- tance of many meningococcal strains, rifampin is preferred. Katzung 12th ed pg 812

MAIRRE JAMES GADDI, MD (TOP 4 - AUG 2013 MED BOARDS; TOPNOTCH MD)

BACK-UP MIDTERM EXAM - FEB 2015

A 3/M was brought to the hospital for decreased sensorium. On further history, the child accidentally ingested a mouthful of antifreeze. You then proceed to give the child which antidote? A. Disulfiram B. Fomepizole C. Famotidine D. Methanol E. There is no antidote Which antihelminthic drugs rapidly kills adult worms presumably through inhibition of oxidative phosphorylation or stimulation of ATPase activity? A. Mebendazole B. Niclosamide C. Praziquantel D. Pyrantel pamoate E. Ivermectin

Antidotes for ethylene glycol or methanol poisoning include ethanol and fomepizole. Although ethanol is effective, it is difficult to achieve safe and effective blood levels that is why fomepizole is preferred. Katzung 12th ed pg 1037

MAIRRE JAMES GADDI, MD (TOP 4 - AUG 2013 MED BOARDS; TOPNOTCH MD)

BACK-UP MIDTERM EXAM - FEB 2015

MOA of mebendazole: Mebendazole probably acts by inhibiting microtubule synthesis; Praziquantel: increase the permeability of trematode and cestode cell membranes to calcium, resulting in paralysis, dislodgement, and death; Pyrantel pamoate: neuromuscular blocking agent that causes release of acetylcholine and inhibition of cholinesterase which results in paralysis of worms followed by expulsion; Ivermectin: paralyze nematodes and arthropods by intensifying γ-aminobutyric acid (GABA)–mediated transmission of signals in peripheral nerves. Katzung 12th ed pg 941-945 Cetuximab targets EGFR; Bevacizumab targets VEGF; Omalizumab is an anti-IgE useful in asthma therapy; Natalizumab binds to α4-subunit of α4β1 and α4β7 integrins expressed on the surfaces of all leukocytes except neutrophils. Used in MS and Crohn's disease. Katzung 12th ed pg 991-993

MAIRRE JAMES GADDI, MD (TOP 4 - AUG 2013 MED BOARDS; TOPNOTCH MD)

BACK-UP MIDTERM EXAM - FEB 2015

MAIRRE JAMES GADDI, MD (TOP 4 - AUG 2013 MED BOARDS; TOPNOTCH MD)

BACK-UP MIDTERM EXAM - FEB 2015

The patient is most likely experiencing bronchospasm and although all volatile anesthetics possess varying degrees of bronchodilating properties airway irritation induced by the pungency of isoflurane and desflurane may induce bronchospasm. These rarely occur with halothane, sevoflurane and nitrous oxide since they are non pungent. Katzung 12th ed pg 436

MAIRRE JAMES GADDI, MD (TOP 4 - AUG 2013 MED BOARDS; TOPNOTCH MD)

BACK-UP MIDTERM EXAM - FEB 2015

Fibrates stimulate PPAR alpha, most effective for decreasing elevated triglyceride levels. Katzung 12th ed pg 626, 628, 631

MAIRRE JAMES GADDI, MD (TOP 4 - AUG 2013 MED BOARDS; TOPNOTCH MD)

BACK-UP MIDTERM EXAM - FEB 2015

A 60/M was diagnosed to have refractory follicular B-cell lymphoma. He was prescribed with an agent which binds CD20 on both normal and malignant B lymphocytes. Which of the following is this drug? A. Rituximab B. Cetuximab C. Bevacizumab D. Omalizumab E. Natalizumab A 34/F asthmatic, scheduled to undergo cholelithiasis, was being induced by the anesthesiologist. After starting the volatile anesthetic, the anesthesiologist noted increased peak air pressure, prolonged expiration with the classic "shark fin" capnograph. Which of the following most likely have caused the above findings? A. Halothane B. Sevoflurane C. Desflurane D. Nitrous oxide E. None of the above Which of the following regarding anticholesterol drugs is/are TRUE? A. Simvastatin, a reversible inhibitor of HMG-CoA reductase, is most effective in reducing LDL levels B. Niacin stimulates PPAR-α causing an increase in HDL and a decrese in triglycerides C. There is increased risk of gallstone formation when fibrates are combined with bile acid binding resins D. A and C E. All of the above

TOPNOTCH MEDICAL BOARD PREP PHARMACOLOGY SUPEREXAM Page 42 of 83 For inquiries visit www.topnotchboardprep.com.ph or email us at [email protected]

TOPNOTCH MEDICAL BOARD PREP PHARMACOLOGY SUPEREXAM For inquiries visit www.topnotchboardprep.com.ph or email us at [email protected] Item # 371

372

373

374

375

376

377

378

QUESTION

EXPLANATION

AUTHOR

A 50/M is about to be given 6-mercaptopurine for his ulcerative colitis as an off-label use. The patient has multiple comorbidities which include hypertension, diabetes, asthma and gout and takes losartan, metformin, salmeterol, allopurinol and diclofenac for these conditions. Which of the above drugs would prompt you to rethink the dose of 6-MP that you are about to administer? A. Losartan B. Metformin C. Salmeterol D. Allopurinol E. Diclofenac A 18/M previously diagnosed with epilepsy and maintained on phenobarbital seizure free for the past 2 years, presented to you with a 3 month history of cough, recurrent fever and weight loss. AFB was positive on two occasions and CXR show cavitations on the upper lung fields. You conclude that he has PTB and you start him of HRZE. After a few days, you were informed that you patient was in the ER and their working impression is status epilepticus. Which of the following drugs could have caused the breakthrough seizures? A. Isoniazid B. Rifampicin C. Pyrazinamide D. Ethambutol E. None of the above A 50/M previously diagnosed with BPH develops mild fever, cough and rhinorrhea. Thinking that he just has the common cold, he self medicates with over the counter medication. After 12 hours he rushes to the ER and complains of inability to void. Which of the following OTCs may have caused his condition? A. Chlorphenamine B. Phenylephrine C. Acetaminophen D. Ibuprofen E. None of the above Which of the following have been proven to cause pulmonary fibrosis? A. Methothrexate B. Bleomycin C. Busulfan D. Amiodarone E. All of the above

Because allopurinol inhibits xanthine oxidase, simultaneous therapy with allopurinol and 6-MP would result in increased levels of 6-MP, thereby leading to excessive toxicity. In this setting, the dose of mercaptopurine must be reduced by 50–75%. Katzung 12th ed pg 961-962

MAIRRE JAMES GADDI, MD (TOP 4 - AUG 2013 MED BOARDS; TOPNOTCH MD)

Memorize inducers & inhibitors VERY HIGH YIELD

MAIRRE JAMES GADDI, MD (TOP 4 - AUG 2013 MED BOARDS; TOPNOTCH MD)

BACK-UP MIDTERM EXAM - FEB 2015

Sympathomimetic agents may cause or worsen urinary difficulty in patients with prostate enlargement due to smooth muscle contraction in the bladder neck via stimulation of alpha-1 adrenergic receptors. Katzung 12th ed pg 139

MAIRRE JAMES GADDI, MD (TOP 4 - AUG 2013 MED BOARDS; TOPNOTCH MD)

BACK-UP MIDTERM EXAM - FEB 2015

Topnotch

MAIRRE JAMES GADDI, MD (TOP 4 - AUG 2013 MED BOARDS; TOPNOTCH MD)

BACK-UP MIDTERM EXAM - FEB 2015

A 21/M presented with recurrent fever, night sweats, enlarged abdomen and generalized lymphadenopathy. Biopsy of a lymph node showed Reed-Sternberg cells. Which of the following chemotherapeutic regimen/s can be given to the patient? A. ABVD B. CHOP C. MOPP D. Rituximab E. A and C Drug X is currently undergoing clinical trials with it being tested on a small number of volunteers with the goal being to find the maximum tolerated dose. It is now on what phase of clinical testing? A. Phase 1 B. Phase 2 C. Phase 3 D. Phase 4 E. Phase 5 A 65/M was diagnosed to have CAP-HR. You plan to start Piperacillin-Tazobactam with a loading dose of 4.5g IV. Which among the following variables is not involved in the calculation of the loading dose? A. Volume of distribution B. Clearance C. Bioavailability D. Desired concentration E. All are needed to calculate the LD Which of the following inhibit/s cortisol synthesis? A. Aminoglutethimide B. Ketoconazole C. Mifepristone D. A and B E. All of the above

ABVD (doxorubicin, bleomycin, vinblastine, and dacarbazine) and MOPP (mechlorethamine, vincristine, procarbazine, and prednisone) have been used to treat Hodgkin's lymphoma. ABVD has supplanted MOPP as the regimen of choice since it is more effective and less toxic. CHOP and Rituximab are used for NHL Katzung 12th ed pg 970

MAIRRE JAMES GADDI, MD (TOP 4 - AUG 2013 MED BOARDS; TOPNOTCH MD)

BACK-UP MIDTERM EXAM - FEB 2015

Phase 1- tested on small number of healthy volunteers in order to find the maximum tolerated dose and prevent severe toxicity; Phase 2- studied on a modest number of patients with the target disease to test efficacy; Phase 3- evaluated on a large number of patients to further establish efficacy and safety, usually a RCT with blinding and crossover; Phase 4- post marketing surveillance Katzung 12th ed pg 75 Clearance is involved in determining the maintenance dose Topnotch

MAIRRE JAMES GADDI, MD (TOP 4 - AUG 2013 MED BOARDS; TOPNOTCH MD)

BACK-UP MIDTERM EXAM - FEB 2015

MAIRRE JAMES GADDI, MD (TOP 4 - AUG 2013 MED BOARDS; TOPNOTCH MD)

BACK-UP MIDTERM EXAM - FEB 2015

Aminoglutethimide blocks the conversion of cholesterol to pregnenolone and causes a reduction in the synthesis of all hormonally active steroids. Ketoconazole is a potent nonselective inhibitor of adrenal and gonadal steroid synthesis through inhibiton of P450 enzymes. Mifepristone is a glucocorticoid receptor antagonist with strong antiprogesterone activity. Katzung 12th ed pg 709711

MAIRRE JAMES GADDI, MD (TOP 4 - AUG 2013 MED BOARDS; TOPNOTCH MD)

BACK-UP MIDTERM EXAM - FEB 2015

TOPNOTCH MEDICAL BOARD PREP PHARMACOLOGY SUPEREXAM Page 43 of 83 For inquiries visit www.topnotchboardprep.com.ph or email us at [email protected]

TOPNOTCH EXAM BACK-UP MIDTERM EXAM - FEB 2015

TOPNOTCH MEDICAL BOARD PREP PHARMACOLOGY SUPEREXAM For inquiries visit www.topnotchboardprep.com.ph or email us at [email protected] Item # 379

380

381

382

383

QUESTION

EXPLANATION

AUTHOR

TOPNOTCH EXAM BACK-UP MIDTERM EXAM - FEB 2015

A 34/F came in to the ER complaining of severe eye pain. On PE you notice that the cornea is steamy with accompanying conjunctival injection. You proceed to administer a drug that will cause an increase in the outflow of aqueous humor through the canal of Schlemm resulting in a decrease in IOP. Which drug was administered? A. Timolol B. Apraclonidine C. Latanoprost D. Pilocarpine E. Epinephrine Minoxidil causes vasodilation through: A. Hyperpolarization of smooth muscle membrane through opening of potassium channels B. Blockade of voltage gated calcium channels causing a reduction in calcium influx C. Release of nitric oxide from the drug itself D. Activation of dopamine receptors E. Blockade of beta receptors Which of the following drugs is classified as Category X during pregnancy? A. Valproate B. Statins C. Aspirin D. Captopril E. Phenytoin

All of the choices can decrease IOP and are used in the treatment of glaucoma. Beta blockers, osmotic agents, alpha-2 agonists and carbonic anhydrase inhibitors decrease secretion of aqueous humor from the ciliary epithelium. Cholinomimetics cause contraction of the ciliary muscle causing the trabecular meshwork to open, increasing outflow. Prostaglandins increase outflow through the canal of Schlemm. Nonselective alpha agonists increase outflow via the uveoscleral veins. Topnotch

MAIRRE JAMES GADDI, MD (TOP 4 - AUG 2013 MED BOARDS; TOPNOTCH MD)

Choice B - calcium channel blockers; Choice C - Nitroprusside, nitrates; Choice D - fenoldepam

MAIRRE JAMES GADDI, MD (TOP 4 - AUG 2013 MED BOARDS; TOPNOTCH MD)

BACK-UP MIDTERM EXAM - FEB 2015

The rest of the choices are category D drugs.

SCOTT RILEY ONG, MD (TOP 5 - AUG 2014 MED BOARDS; TOPNOTCH MD)

BACK-UP MIDTERM EXAM - FEB 2015

Which of the following is a balanced vasodilator-meaning, both an arterial and a venous vasodilator-used to treat both congestive heart failure and coronary artery vasospasm? A. Verapamil B. Captopril C. Nifedipine D. Propranolol E. Hydralazine Which of the following diuretics do not act on the luminal side of the renal tubules? A. Mannitol B. Furosemide C. Spironolactone D. Acetazolamide E. Hydrochlorothiazide

Balanced vasodilators include ACEIs, ARBs, alphaadrenergic blockers, alpha-central agonists, nitroprusside and nesiritide.

SCOTT RILEY ONG, MD (TOP 5 - AUG 2014 MED BOARDS; TOPNOTCH MD)

BACK-UP MIDTERM EXAM - FEB 2015

SIMILAR TO PREVIOUS BOARD EXAM CONCEPT/PRINCIPLE. Spironolactone is an aldosterone antagonist that blocks its binding to mineralocorticoid receptors in the basolateral membrane of the cortical collecting tubules. The rest of the choices act on the luminal side. Mannitol is an osmotic diuretic. Furosemide inhibits Na/K/Cl transporters in the luminal side of the loop of Henle. Acetazolamide inhibits carbonic anhydrase activity at the luminal side of the PCT. Hydrochlorothiazide inhibits Na/Cl transporters in the luminal side of the DCT.

SCOTT RILEY ONG, MD (TOP 5 - AUG 2014 MED BOARDS; TOPNOTCH MD)

BACK-UP MIDTERM EXAM - FEB 2015

SCOTT RILEY ONG, MD (TOP 5 - AUG 2014 MED BOARDS; TOPNOTCH MD)

BACK-UP MIDTERM EXAM - FEB 2015

384

Which of the following vasodilators has also been used for the treatment of male pattern baldness due to its side effect of inducing hair growth A. Verapamil B. Fenoldopam C. Nitroprusside D. Minoxidil E. Hydralazine

385

Which of the following is the drug of choice for tic disorders? A. Carbamazepine B. Lamotrigine C. Valproic acid D. Topiramate E. Phenytoin

Carbamazepine is also the drug of choice for trigeminal neuralgia.

SCOTT RILEY ONG, MD (TOP 5 - AUG 2014 MED BOARDS; TOPNOTCH MD)

BACK-UP MIDTERM EXAM - FEB 2015

386

Which of the following drugs used in the treatment of peptic ulcer disease works by forming a protective coating over ulcer beds and has known adverse effects of melena and darkening of the tongue? A. Sucralfate B. Bismuth salicylate C. Psyllium D. Senna E. Kaolin Which of the following analgesics would you avoid in a patient with history of epilepsy as it is known to lower the seizure threshold? A. Ketorolac B. Aspirin C. Tramadol D. Celecoxib E. Indomethacin



SCOTT RILEY ONG, MD (TOP 5 - AUG 2014 MED BOARDS; TOPNOTCH MD)

BACK-UP MIDTERM EXAM - FEB 2015



SCOTT RILEY ONG, MD (TOP 5 - AUG 2014 MED BOARDS; TOPNOTCH MD)

BACK-UP MIDTERM EXAM - FEB 2015

387

TOPNOTCH MEDICAL BOARD PREP PHARMACOLOGY SUPEREXAM Page 44 of 83 For inquiries visit www.topnotchboardprep.com.ph or email us at [email protected]

TOPNOTCH MEDICAL BOARD PREP PHARMACOLOGY SUPEREXAM For inquiries visit www.topnotchboardprep.com.ph or email us at [email protected] Item # 388

QUESTION

EXPLANATION

AUTHOR

TOPNOTCH EXAM BACK-UP MIDTERM EXAM - FEB 2015

In which of the following drugs used for the treatment of gout is cataract an important side effect? A. Colchicine B. Probenicid C. Allopurinol D. Febuxostate E. Indomethacin

SIMILAR TO PREVIOUS BOARD EXAM CONCEPT/PRINCIPLE.

SCOTT RILEY ONG, MD (TOP 5 - AUG 2014 MED BOARDS; TOPNOTCH MD)

389

Which of the following adrenergic receptors in the ciliary body of the eye causes a decrease in aqueous humor production when activated? A. alpha-1 B. alpha-2 C. beta-1 D. beta-2 E. beta-3

Activation of alpha-2 receptors decreases aqueous humor production, while activation of either beta-1 or beta-2 receptors increases aqeous humor production. Alpha-1 and beta-3 receptors have no known function in relation to aqueous humor production.

SCOTT RILEY ONG, MD (TOP 5 - AUG 2014 MED BOARDS; TOPNOTCH MD)

BACK-UP MIDTERM EXAM - FEB 2015

390

Tocainide is an antiarrhythmic agent classified under which group? A. Class IA B. Class IB C. Class IC D. Class II E. Class III



SCOTT RILEY ONG, MD (TOP 5 - AUG 2014 MED BOARDS; TOPNOTCH MD)

BACK-UP MIDTERM EXAM - FEB 2015

391

Ondansetron is an antagonist of which serotonin receptor? A. 5HT1D B. 5HT1B C. 5HT2 D. 5HT3 E. 5HT4

5HT3 receptor is also the only ligand-gated ion channel among the serotonin receptors.

SCOTT RILEY ONG, MD (TOP 5 - AUG 2014 MED BOARDS; TOPNOTCH MD)

BACK-UP MIDTERM EXAM - FEB 2015

392

Clomiphene is a selective estrogen-receptor modulator commonly used to induce ovulation in patients undergoing assisted reproduction therapy. Which of the following best describes its mechanism of action? A. It potentiates the action of FSH and LH on the ovaries. B. It acts as a partial agonist in the pituitary to block negative feedback from estradiol. C. It acts as a GnRH analogue that increases the release of FSH and LH from the pituitary. D. It inhibits the release of inhibin from ovarian follicles, thus facilitating FSH and LH secretion. E. It is an LH analogue and mimics the physiologic LH surge when given just before ovulation. Which of the following anti-lipid drugs would you avoid giving to a diabetic patient due to its adverse effect of inducing hyperglycemia? A. Cholestyramine B. Egetimibe C. Niacin D. Gemfibrozil E. Colestipol

C refers to leuprolide when administered in pulsatile fashion.

SCOTT RILEY ONG, MD (TOP 5 - AUG 2014 MED BOARDS; TOPNOTCH MD)

BACK-UP MIDTERM EXAM - FEB 2015



SCOTT RILEY ONG, MD (TOP 5 - AUG 2014 MED BOARDS; TOPNOTCH MD)

BACK-UP MIDTERM EXAM - FEB 2015

394

Which of the following traditional antipsychotic agents has now been more widely used for antiemetic treatment of nausea and vertigo? A. Prochlorperazine B. Promethazine C. Haloperidol D. Chlorpromazine E. Metoclopramide

Promethazine is histamine H1-antagonist. Haloperidol and chlorpromazine are typical antipsychotics mainly used for psychotic disorders, not so much as antiemesis. Metoclopramide is a D2antagonist used as GI prokinetic agent.

SCOTT RILEY ONG, MD (TOP 5 - AUG 2014 MED BOARDS; TOPNOTCH MD)

BACK-UP MIDTERM EXAM - FEB 2015

395

Which of the following is not used for the treatment of alcohol dependence? A. Chlordiazepoxide B. Disulfiram C. Naltrexone D. Acamprosate E. None of the above

Chlordiazepoxide is used for the treatment of alcohol withdrawal, not dependence. Diazepam may also be used for alcohol withdrawal.

SCOTT RILEY ONG, MD (TOP 5 - AUG 2014 MED BOARDS; TOPNOTCH MD)

BACK-UP MIDTERM EXAM - FEB 2015

396

Which of the following antitumor antibiotics is cellcycle specific? A. Mitomycin B. Doxorubicin C. Bleomycin D. Actinomycin E. None of the above

Bleomycin is specific for G2 phase of the cell cycle. The rest of the choices are non-cell-cycle specific.

SCOTT RILEY ONG, MD (TOP 5 - AUG 2014 MED BOARDS; TOPNOTCH MD)

BACK-UP MIDTERM EXAM - FEB 2015

393

TOPNOTCH MEDICAL BOARD PREP PHARMACOLOGY SUPEREXAM Page 45 of 83 For inquiries visit www.topnotchboardprep.com.ph or email us at [email protected]

TOPNOTCH MEDICAL BOARD PREP PHARMACOLOGY SUPEREXAM For inquiries visit www.topnotchboardprep.com.ph or email us at [email protected] Item # 397

QUESTION

EXPLANATION

AUTHOR

TOPNOTCH EXAM BACK-UP MIDTERM EXAM - FEB 2015

Which of the following antimicrobial agent acts as a dihydrofolate reductase inhibitor? A. Trimethoprim B. Sulfamethoxazole C. Metronidazole D. Nitrofurantoin E. Rifampicin



SCOTT RILEY ONG, MD (TOP 5 - AUG 2014 MED BOARDS; TOPNOTCH MD)

398

In which of the following penicillins is interstitial nephritis a known adverse effect, partly leading to a decline in its popularity? A. Nafcillin B. Methicillin C. Carbenicillin D. Bacitracin E. Metampicillin

SIMILAR TO PREVIOUS BOARD EXAM CONCEPT/PRINCIPLE.

SCOTT RILEY ONG, MD (TOP 5 - AUG 2014 MED BOARDS; TOPNOTCH MD)

BACK-UP MIDTERM EXAM - FEB 2015

399

Which of the following antibiotics work by binding to the D-Ala-D-Ala terminus of bacterial cell wall precursors, thus preventing peptidoglycan polymerization? A. Imipenem B. Cycloserine C. Bacitracin D. Vancomycin E. Piperacillin Which of the following ophthalmic agents decreases intraocular pressure by increasing outflow of aqueous humor through the uveoscleral or unconventional pathway? A. Homatropine B. Bimatoprost C. Apraclonidine D. Mannitol E. Timolol Which of the following chemotherapeutic drugs may be used for Lentigo Maligna? A. Actinomycin B. Bleomycin C. Doxorubicin D. Vinblastine E. Cisplatin

SIMILAR TO PREVIOUS BOARD EXAM CONCEPT/PRINCIPLE. Piperacillin: inhibits transpeptidase and binds to penicillin-binding proteins in the cell membrane. Cycloserine: blocks incorporation of D-Ala into the pentapeptide side chain of peptidoglycan. Bacitracin: interferes with a late stage in cell wall synthesis.

SCOTT RILEY ONG, MD (TOP 5 - AUG 2014 MED BOARDS; TOPNOTCH MD)

BACK-UP MIDTERM EXAM - FEB 2015

Bimatoprost is a prostaglandin analogue that increase outflow via the uveoscleral pathway. Apraclonidine is a alpha-2 receptor agonist that decreases aqueous humor production. Mannitol works via osmosis. Timolol is a beta-receptor antagonist that also decreases aqueous humor production. Homatropine has no effect on IOP.

SCOTT RILEY ONG, MD (TOP 5 - AUG 2014 MED BOARDS; TOPNOTCH MD)

BACK-UP MIDTERM EXAM - FEB 2015

SIMILAR TO PREVIOUS BOARD EXAM CONCEPT/PRINCIPLE

JOSE CARLO MASANGKAY III, MD (TOP 8 - FEB 2014 MED BOARDS; TOPNOTCH MD)

DIAGNOSTIC EXAM - AUG 2014

402

Which of the following chemotherapeutic drugs may be used for Lung Cancer? A. Procarbazine B. Cytarabine C. Topotecan D. Cisplatin E. Cyclophosphamide

SIMILAR TO PREVIOUS BOARD EXAM CONCEPT/PRINCIPLE

JOSE CARLO MASANGKAY III, MD (TOP 8 - FEB 2014 MED BOARDS; TOPNOTCH MD)

DIAGNOSTIC EXAM - AUG 2014

403

Which of the following drugs is an effective prophylactic drug against migraine headache attacks? A. Ondansetron B. Sumatriptan C. Ergotamine D. Methylergonovine E. Propranolol

SIMILAR TO PREVIOUS BOARD EXAM CONCEPT/PRINCIPLE

JOSE CARLO MASANGKAY III, MD (TOP 8 - FEB 2014 MED BOARDS; TOPNOTCH MD)

DIAGNOSTIC EXAM - AUG 2014

404

A newly formulated drug with a half life of 6 hours is eliminated via 1st order kinetics, how many hours will the drug be 100% eliminated from the body of a normal person? A. 6 hours B. 12 hours C. 18 hours D. 24 hours E. 36 hours In calculating the loading dose of a drug, which of the following parameters are not necessary? A. Volume of distribution B. Desired Plasma Concentration C. Bioavailability D. Clearance E. None of the Above

100% elimination will be reached in 4 half lives. (6 x 4=24)

JOSE CARLO MASANGKAY III, MD (TOP 8 - FEB 2014 MED BOARDS; TOPNOTCH MD)

DIAGNOSTIC EXAM - AUG 2014

LD = (Vd x DPC) / Bioavailability

JOSE CARLO MASANGKAY III, MD (TOP 8 - FEB 2014 MED BOARDS; TOPNOTCH MD)

DIAGNOSTIC EXAM - AUG 2014

Which of the following is a reversible inhibitor of Xanthine Oxidase used in Chronic Gout? A. Allopurinol B. Colchicine C. Febuxostat D. Mycophenolate Mofetil E. Probenecid

Allopurinol is an IRREVERSIBLE Xanthine Oxidase inhibitor, Febuxostat is a REVERSIBLE Xanthine Oxidase inhibitor,

JOSE CARLO MASANGKAY III, MD (TOP 8 - FEB 2014 MED BOARDS; TOPNOTCH MD)

DIAGNOSTIC EXAM - AUG 2014

400

401

405

406

TOPNOTCH MEDICAL BOARD PREP PHARMACOLOGY SUPEREXAM Page 46 of 83 For inquiries visit www.topnotchboardprep.com.ph or email us at [email protected]

TOPNOTCH MEDICAL BOARD PREP PHARMACOLOGY SUPEREXAM For inquiries visit www.topnotchboardprep.com.ph or email us at [email protected] Item # 407

QUESTION

EXPLANATION

AUTHOR

TOPNOTCH EXAM DIAGNOSTIC EXAM - AUG 2014

A patient will be undergoing neurosurgery, the neurosurgeon would like to maintain the patient's consciousness while the procedure is on going. What would be the preferred anesthetic of choice? A. Ketamine B. Etomidate C. Midazolam D. Fentanyl E. Propofol A 14 year old child was diagnosed with Schizophrenia, which of the following is the only antipsychotic approved for pediatric patients? A. Clozapine B. Risperidone C. Quetiapine D. Olanzapine E. Thioridazine

Ketamine is capable of Dissociative Anesthesia ( Analgesia, Amnesia and Catatonia with retained consciousness)

JOSE CARLO MASANGKAY III, MD (TOP 8 - FEB 2014 MED BOARDS; TOPNOTCH MD)



JOSE CARLO MASANGKAY III, MD (TOP 8 - FEB 2014 MED BOARDS; TOPNOTCH MD)

DIAGNOSTIC EXAM - AUG 2014

The following statements are true of Aminoglycosides, EXCEPT: A. Act by Time-Dependent Killing Action B. As the plasma level is increased above the MIC an increasing proportion of bacteria are killed and at a more rapid rate C. Has a post-antibiotic effect D. Requires Oxygen for uptake E. Amikacin has the widest coverage Among the following Anti-arrhythmics, which of the following has a highest potential for Torsade de pointes? A. Lidocaine B. Propranolol C. Sotalol D. Procainamide E. Verapamil



JOSE CARLO MASANGKAY III, MD (TOP 8 - FEB 2014 MED BOARDS; TOPNOTCH MD)

DIAGNOSTIC EXAM - AUG 2014

K channel blockers like Sotalol can increase the QT interval thereby increasing the propensity to develop Torsade de Pointes.

JOSE CARLO MASANGKAY III, MD (TOP 8 - FEB 2014 MED BOARDS; TOPNOTCH MD)

DIAGNOSTIC EXAM - AUG 2014

A patient with CHF was rushed to the ED where you are on-duty, ABG was done revealing Metabolic Acidosis. His wife told you that he had a history of severe allergic reaction to Sulfa drugs, but you need to give the patient a powerful diuretic, Unfortunately the following drugs are the only ones available in your Pharmacy, Among which of the following will you give? A. Furosemide B. Spironolactone C. Torsemide D. Ethacrynic Acid E. Acetazolamide To improve the quality of life of patients with ongoing treatment for Prostate Adenocarcinoma with Leuprolide, which of the following medications should be added? A. Anastrazole B. Examestane C. Flutamide D. Prednisone E. Morphine A 9-year old Filipino child was brought to you in the clinics by her mother due to an observed blank-stare episodes noted by her mother which lasts from a few seconds to a few minutes. Which of the following medications is the most suitable for this patient? A. Valproate B. Phenytoin C. Phenobarbital D. Carbamazepine E. Lorazepam

Spironolactone and Acetazolamide can aggravate the ongoing acidosis of the patient, the only ones left are loop diuretics, but then Furosemide and Torsemide are both Sulfa containing hence cannot be given to the patient. Ethacrynic acid is most suitable to this patient.

JOSE CARLO MASANGKAY III, MD (TOP 8 - FEB 2014 MED BOARDS; TOPNOTCH MD)

DIAGNOSTIC EXAM - AUG 2014

Flutamide an Androgen Antagonist is given in adjunct to Leuprolide to lessen the side effects of Leuprolide like Gynecomastia, decreased libido, Apoplexy and hot flushes.(SIMILAR TO PREVIOUS BOARD EXAM CONCEPT/PRINCIPLE)

JOSE CARLO MASANGKAY III, MD (TOP 8 - FEB 2014 MED BOARDS; TOPNOTCH MD)

DIAGNOSTIC EXAM - AUG 2014

Valproate is the DOC for Absence Seizures, most especially in our setting here in the Philippines, Ethosuximide is NOT commercially available.

JOSE CARLO MASANGKAY III, MD (TOP 8 - FEB 2014 MED BOARDS; TOPNOTCH MD)

DIAGNOSTIC EXAM - AUG 2014

414

Which of the following Beta Blockers has an Intrinsic Sympathomimetic Activity? A. Carvedilol B. Pindolol C. Nadolol D. Metoprolol E. Timolol

Pindolol has ISA, which makes it a B blocker with a partial Alpha agonist action.

JOSE CARLO MASANGKAY III, MD (TOP 8 - FEB 2014 MED BOARDS; TOPNOTCH MD)

DIAGNOSTIC EXAM - AUG 2014

415

Early morning hypoglycemia is a challenge among physicians in treating diabetic patients. When a diabetic patient monitors his Glucose at 3 AM revealing Hypoglycemia with a Hyperglycemic 7 AM CBG, This effect is termed as: A. Somogyi B. Waning C. Dawn D. Dawn and Waning E. Somogyi and Waning



JOSE CARLO MASANGKAY III, MD (TOP 8 - FEB 2014 MED BOARDS; TOPNOTCH MD)

DIAGNOSTIC EXAM - AUG 2014

408

409

410

411

412

413

TOPNOTCH MEDICAL BOARD PREP PHARMACOLOGY SUPEREXAM Page 47 of 83 For inquiries visit www.topnotchboardprep.com.ph or email us at [email protected]

TOPNOTCH MEDICAL BOARD PREP PHARMACOLOGY SUPEREXAM For inquiries visit www.topnotchboardprep.com.ph or email us at [email protected] Item # 416

QUESTION

EXPLANATION

AUTHOR

TOPNOTCH EXAM DIAGNOSTIC EXAM - AUG 2014

In a patient with Grave's Disease this drug is given to decrease peripheral conversion of T4 to T3: A. Propranolol B. I 131 C. Methimazole D. PTU E. Lugol's Iodine



JOSE CARLO MASANGKAY III, MD (TOP 8 - FEB 2014 MED BOARDS; TOPNOTCH MD)

417

In a Diabetic patient with impaired renal function which of the following drugs is contraindicated? A. Nateglinide B. Glipizide C. Metformin D. Pioglitazone E. Acarbose

Before prescribing 2nd generation Sulfonylureases liver and hepatic function must be checked first because they are contraindicated.

JOSE CARLO MASANGKAY III, MD (TOP 8 - FEB 2014 MED BOARDS; TOPNOTCH MD)

DIAGNOSTIC EXAM - AUG 2014

418

Which of the following Cephalosporins is synergistic with Aminoglycosides? A. Cefazolin B. Cefaclor C. Cefixime D. Cefipime E. Loracarbef

3rd gen Cephalosporins have syndergistic effects with Aminoglycosides, 1st and 2nd gen cephalosporines can increase nephrotoxic effects of aminoflycosides.

JOSE CARLO MASANGKAY III, MD (TOP 8 - FEB 2014 MED BOARDS; TOPNOTCH MD)

DIAGNOSTIC EXAM - AUG 2014

419

What class of anti-hypertensive drugs are also used in BPH? A. Alpha 1A Agonists B. Alpha 1B Agonists C. Alpha 1A Antagonists D. Alpha 1B Antagonists E. Alpha 2 Agonists

Alpha 1-A receptor is specifically seen in the Urinary tract most especially the Prostate.Alpha 1A antagonists/Quinazolines include Prazosin Terazosin, and Tamsulosin

JOSE CARLO MASANGKAY III, MD (TOP 8 - FEB 2014 MED BOARDS; TOPNOTCH MD)

DIAGNOSTIC EXAM - AUG 2014

420

Cocaine has which of the following mechanism of action? A. Promotes Adrenergic Synthesis B. Inhibits Adrenergic Storage C. Promotes Adrenergice Release D. Inhibits Adrenergic Metabolism/Degradation E. Inhibits Adrenergic Reuptake



JOSE CARLO MASANGKAY III, MD (TOP 8 - FEB 2014 MED BOARDS; TOPNOTCH MD)

DIAGNOSTIC EXAM - AUG 2014

421

A condition where there is depletion of needed substrates as a result of continuous activation by a drug and hence decrease in responsiveness, such as in chronic nitroglycerin use, is known as: A. Tachyphylaxis B. Idiosyncratic response C. Tolerance D. Resistance

Idiosyncratic means unusual drug reponse, infrequently observed in most patients; tolerance on the other hand, refers to decrease in the intensity of response to drug as a consequence of continued administration.

WEBSTER ALINDOG, MD (TOP 3 - FEB 2014 MED BOARDS; TOPNOTCH MD)

MIDTERM 1 EXAM - AUG 2014

422

Teratogenesis is one of the dreaded complications of improper use of drugs during pregnancy. Of note, lithium, a drug primarily used to treat bipolar disorders, can cause: A. Ebstein's anomaly B. Vaginal clear cell adenocarcinoma C. Phocomelia D. Mobius sequence



WEBSTER ALINDOG, MD (TOP 3 - FEB 2014 MED BOARDS; TOPNOTCH MD)

MIDTERM 1 EXAM - AUG 2014

423

Pralidoxime is an antidote to organophosphate poisoning. Which of the following is not true about this drug? A. It is a cholinesterase regenerator. B. It requires to be given 6-8 hours before organophosphate-cholinesterase binding occurs. C. It binds with the phosphorus of organophosphates. D. All of the above E. None of the above Methyldopa is a drug of choice in treating preeclampsia. Its ability to lower blood pressure is based on its action of: A. Activating alpha-1 adrenergic receptors B. Inhibiting alpha-1 adrenergic receptors C. Activating alpha-2 adrenergic receptors D. Inhibiting alpha-2 adrenergic receptors



WEBSTER ALINDOG, MD (TOP 3 - FEB 2014 MED BOARDS; TOPNOTCH MD)

MIDTERM 1 EXAM - AUG 2014

Activation of alpha-2 adrenergic receptors by methyldopa metabolites in the brainstem appears to inhibit sympathetic nervous system output and lower blood pressure. This is also the mechanism of action of clonidine. Activation of alpha-1 adrenergic receptors meanwhile, stimulates vasoconstriction and hence increase in BP.

WEBSTER ALINDOG, MD (TOP 3 - FEB 2014 MED BOARDS; TOPNOTCH MD)

MIDTERM 1 EXAM - AUG 2014

Hydralazine lowers BP by causing arteriolar smooth muscle relaxation. This effect is achieved by its ability to alter metabolism of which ion? A. Sodium B. Potassium C. Magnesium D. Calcium



WEBSTER ALINDOG, MD (TOP 3 - FEB 2014 MED BOARDS; TOPNOTCH MD)

MIDTERM 1 EXAM - AUG 2014

424

425

TOPNOTCH MEDICAL BOARD PREP PHARMACOLOGY SUPEREXAM Page 48 of 83 For inquiries visit www.topnotchboardprep.com.ph or email us at [email protected]

TOPNOTCH MEDICAL BOARD PREP PHARMACOLOGY SUPEREXAM For inquiries visit www.topnotchboardprep.com.ph or email us at [email protected] Item # 426

QUESTION

EXPLANATION

AUTHOR

TOPNOTCH EXAM MIDTERM 1 EXAM - AUG 2014

Throbbing headaches in patients taking nitrates are mainly attributed to: A. Chemical toxicity B. Meningeal artery vasodilation C. Reflex tachycardia D. Abrupt lowering of BP



WEBSTER ALINDOG, MD (TOP 3 - FEB 2014 MED BOARDS; TOPNOTCH MD)

427

Which diuretic is found to have long-term benefits and can reduce mortality in chronic heart failure? A. Thiazide B. Furosemide C. Spironolactone D. Ethacrynic acid



WEBSTER ALINDOG, MD (TOP 3 - FEB 2014 MED BOARDS; TOPNOTCH MD)

MIDTERM 1 EXAM - AUG 2014

428

Aside from its lipid-lowering action, statins are used in coronary artery disease because it can also: A. Vasodilate stenotic segments of the coronary B. Stabilize atherosclerotic plaques C. Inhibits platelet aggregation D. Decrease myocardial oxygen demand



WEBSTER ALINDOG, MD (TOP 3 - FEB 2014 MED BOARDS; TOPNOTCH MD)

MIDTERM 1 EXAM - AUG 2014

429

Which of the following is considered as a reliever of asthma rather than a controller? A. Montelukast B. Cromolyn C. Omalizumab D. Terbutaline

Terbutaline is a beta-2 receptor agonist used as a fast-acting bronchodilator for asthma and is also used in preventing premature labor (tocolysis).

WEBSTER ALINDOG, MD (TOP 3 - FEB 2014 MED BOARDS; TOPNOTCH MD)

MIDTERM 1 EXAM - AUG 2014

430

Oral chelation treatment in hemochromatosis can be done by giving: A. Deferoxamine B. Deferasirox C. EDTA D. Penicillamine

Therapeutic phlebotomy is the main line of treatment of hemochromatosis, especially the primary type. Should chelation becomes warranted, it can be done via IV (deferoxamine) or oral (Deferasirox).

WEBSTER ALINDOG, MD (TOP 3 - FEB 2014 MED BOARDS; TOPNOTCH MD)

MIDTERM 1 EXAM - AUG 2014

431

Aspirin toxicity results in increased respiratory drive that leads to hyperventilation and respiratory alkalosis. Which of these statements is not true about aspirin intoxication? A. It is achieved when given at a dose of 150 mg/kg. B. It can manifest as a pure high anion gap metabolic acidosis in children. C. It is a known inhibitor of the electron transport chain. D. All of the abbove E. None of the above Tranexamic acid is used to prevent and treat acute bleeding episodes in patients who are at high risk, especially those who have hemophilia and intracranial aneurysms. The mechanism of action of this drug involves: A. Inhibition of plasminogen activators B. Inhibition of plasmin-degrading factors C. Potentiation of factor VIII activity D. Delivery of Vitamin K to the clotting factorsynthesizing cells Treatment for inflammatory bowel disease: A. Sulfadiazine B. Sulfasalazine C. Sulfamethoxazole D. Sulfatide

Aspirin is not an inhibitor of electron-transport chain (ETC), but an uncoupler. Uncouplers of oxidative phosphorylation in mitochondria inhibit the coupling between the electron transport and phosphorylation reactions and thus inhibit ATP synthesis without affecting the respiratory chain and ATP synthase (H(+)-ATPase), thus heat is just produced without producing the needed ATPs.

WEBSTER ALINDOG, MD (TOP 3 - FEB 2014 MED BOARDS; TOPNOTCH MD)

MIDTERM 1 EXAM - AUG 2014

Plasminogen are converted to plasmin whose function is to lyse the formed fibrin. When it is inhibited, hemostasis is promoted.

WEBSTER ALINDOG, MD (TOP 3 - FEB 2014 MED BOARDS; TOPNOTCH MD)

MIDTERM 1 EXAM - AUG 2014

Sulfasalazine is poorly absorbed in the GIT and therefore it is believed that it has its main action in lumen. It is used to treat IBDs by reducing the synthesis of inflammatory mediators known as eicosanoids and inflammatory cytokines. However, unlike glucocorticoids, sulfasalazine is also a mild immunosuppressant.

WEBSTER ALINDOG, MD (TOP 3 - FEB 2014 MED BOARDS; TOPNOTCH MD)

MIDTERM 1 EXAM - AUG 2014

434

Remarkable to these drugs is their ability to inhibit cytochrome P450 causing significant interactions with other drugs, except: A. Isoniazid B. Cimetidine C. Erythromycin D. Barbiturates



WEBSTER ALINDOG, MD (TOP 3 - FEB 2014 MED BOARDS; TOPNOTCH MD)

MIDTERM 1 EXAM - AUG 2014

435

Which aminoglycoside has the narrowest therapeutic window but with the widest coverage? A. Gentamicin B. Streptomycin C. Amikacin D. Kanamycin



WEBSTER ALINDOG, MD (TOP 3 - FEB 2014 MED BOARDS; TOPNOTCH MD)

MIDTERM 1 EXAM - AUG 2014

432

433

TOPNOTCH MEDICAL BOARD PREP PHARMACOLOGY SUPEREXAM Page 49 of 83 For inquiries visit www.topnotchboardprep.com.ph or email us at [email protected]

TOPNOTCH MEDICAL BOARD PREP PHARMACOLOGY SUPEREXAM For inquiries visit www.topnotchboardprep.com.ph or email us at [email protected] Item # 436

QUESTION

EXPLANATION

AUTHOR

TOPNOTCH EXAM MIDTERM 1 EXAM - AUG 2014

Antifolates sulfamethoxazole and trimethoprim are bactericidal by virtue of their synergistic actions. Nonetheless, resistence can develop via plasmidmediated: A. Decrease in sensitivity of dihydropteroate synthase to sulfonamides B. Increase in PABA synthesis by the organism C. Poor intracellular accumulation of the drugs D. All of the above E. None of the above Isoniazid is the most important drug used in TB. It inhibits mycolic acid synthesis and is therefore bactericidal. Possible adverse effects include neuritis, insomnia and seizures. To prevent neurotoxicities, patient should be supplemented with: A. Folic acid B. Pyridoxine C. Vitamin A D. Riboflavin



WEBSTER ALINDOG, MD (TOP 3 - FEB 2014 MED BOARDS; TOPNOTCH MD)



WEBSTER ALINDOG, MD (TOP 3 - FEB 2014 MED BOARDS; TOPNOTCH MD)

MIDTERM 1 EXAM - AUG 2014

438

What is the drug of choice for the cyst carrier state of Entamoeba histolytica? A. Diloxanide furoate B. Metronidazole C. Tinidazole D. Paramomycin

Diloxanide furoate is a luminal amoebicide which is hydrolysed in the gut, thus releasing the free diloxanide which acts as an amoebicide. It is given alone in asymptomatic cyst passers. For patients with active amoebic infections, it can be administered along with metronidazole.

WEBSTER ALINDOG, MD (TOP 3 - FEB 2014 MED BOARDS; TOPNOTCH MD)

MIDTERM 1 EXAM - AUG 2014

439

A vinca alkaloid acting primarily in the M phase of cancer cell cycle that is useful in the treatment of acute leukemias, lymphomas and neuroblastoma: A. Paclitaxel B. Vincristine C. Doxorubicin D. Cytarabine



WEBSTER ALINDOG, MD (TOP 3 - FEB 2014 MED BOARDS; TOPNOTCH MD)

MIDTERM 1 EXAM - AUG 2014

440

This is a mucosal protective drug used as an adjunct treatment of PUD. It binds to injured tissues and forms a protective covering over ulcer beds, resulting in accelerated healing: A. Misoprostol B. Bismuth salicylate C. Calcium carbonate D. Sucralfate



WEBSTER ALINDOG, MD (TOP 3 - FEB 2014 MED BOARDS; TOPNOTCH MD)

MIDTERM 1 EXAM - AUG 2014

441

Factor/s that influence/s passage of drugs through cell membranes: A. Presence of pores in the membrane B. Amount of protein binding C. Lipid solubility D. pH E. All of the above

All are factors that influence passage of drugs through cell membranes. Lipid solubility is the most important limiting factor for permeation of drugs because cells are covered by lipid membranes.

JULIET KRISTINE EVANGELISTA, MD (TOP 9 - FEB 2014 MED BOARDS; TOPNOTCH MD)

MIDTERM 2 EXAM - AUG 2014

442

In this clinical phase of drug development, the drug is studied in patients with the target disease to determine its efficacy: A. Phase 0 B. Phase 1 C. Phase 2 D. Phase 3 E. Phase 4

JULIET KRISTINE EVANGELISTA, MD (TOP 9 - FEB 2014 MED BOARDS; TOPNOTCH MD)

MIDTERM 2 EXAM - AUG 2014

443

The most important pharmacologic action of digitalis in the treatment of heart failure is its ability to: A. Increase heart rate B. Reduce venous pressure C. Reduce arterial pressure D. Produce diuresis in edematous patients E. Increase myocardial contractile force

Phase 1 trial involves evaluation of dose-response relationship and pharmacokinetics in normal volunteer patients with target disease. Phase 2 trial involves determination whether the agent has the desired efficacy at doses that are tolerated by sick patients. Phase 3 trial is a large design including placebo and positive controls in a double-blind crossover design. Phase 4 trial is a postmarketing surveillance. The most important pharmacologic action of digitalis in the treatment of heart failure is its ability to increase myocardial contractile force by inhibiting Na/K ATPase and increaseing intracellular calcium.

JULIET KRISTINE EVANGELISTA, MD (TOP 9 - FEB 2014 MED BOARDS; TOPNOTCH MD)

MIDTERM 2 EXAM - AUG 2014

444

Reserpine blocks this step of adrenergic transmission: A. Synthesis of the neurotransmitter B. Transport of neurotransmitter to vesicles C. Release of the neurotransmitter D. Reuptake into the presynaptic neuron E. Diffusion in the synaptic cleft

Reserpine blocks vesicular transport in adrenergic transmission.

JULIET KRISTINE EVANGELISTA, MD (TOP 9 - FEB 2014 MED BOARDS; TOPNOTCH MD)

MIDTERM 2 EXAM - AUG 2014

437

TOPNOTCH MEDICAL BOARD PREP PHARMACOLOGY SUPEREXAM Page 50 of 83 For inquiries visit www.topnotchboardprep.com.ph or email us at [email protected]

TOPNOTCH MEDICAL BOARD PREP PHARMACOLOGY SUPEREXAM For inquiries visit www.topnotchboardprep.com.ph or email us at [email protected] Item # 445

EXPLANATION

AUTHOR

A 25 year-old farmer was brought to the emergency room after having allegedly drank a poison. PE revealed that he had constricted pupils, excessive salivation and sweating, wheezing and had several bouts of vomiting and diarrhea. He was given antidotes Atropine and Pralidoxime. Pralidoxime exert its effects by: A. Destruction of the enzyme B. Competitive antagonism C. Noncompetitive antagonism D. Physiologic antagonism E. Chemical antagonism It is the major second messenger of beta receptor activation that participates in signal transduction: A. Inositol triphosphate B. cAMP C. cGMP D. Calcium E. Adenylyl cyclase

Pralidoxime is a cholinesterase regenerator, an antidote for organophosphate poisoning. Pralidoxime is an example of chemical antagonism. Chemical antagonism is when an antagonist directly interacts with the drug being antagonized to remove it or prevent it from reaching its target.

JULIET KRISTINE EVANGELISTA, MD (TOP 9 - FEB 2014 MED BOARDS; TOPNOTCH MD)

cAMP is the major second messenger of beta receptor activation that participates in signal transduction.

JULIET KRISTINE EVANGELISTA, MD (TOP 9 - FEB 2014 MED BOARDS; TOPNOTCH MD)

MIDTERM 2 EXAM - AUG 2014

A neonate was given IV antibiotics for 3 days. However, the baby developed hypothermia associated with diarrhea and grayish color of the skin. What is the mechanism of action of the drug that was given to the baby? A. Inhibits transpeptidation in bacterial cell walls B. Inhibits protein synthesis by binding to 30s subunit C. Inhibits transpeptidation at 50s subunit D. Inhibits dihydropteroate synthase E. Inhibits DNA replication by binding to DNA gyrase It is a drug that is used almost solely as an alternative treatment for gonorrhea in patients whose gonococci are resistant to other drugs is: A. Docycycline B. Spectinomycin C. Ofloxacin D. Azithromycin E. Tetracycline

Chloramphenicol is the drug that was given to neonate causing gray baby syndrome. It inhibits transpeptidation at 50s subunit and it is bacteriostatic. Penicillin inhibits transpeptidation in bacterial cell walls. Aminoglycoside inhibits protein synthesis by binding to 30s subunit. Co-trimoxazole inhibits dihydropteroate synthase. Fluoroquinolone inhibits DNA replication by binding to DNA gyrase.

JULIET KRISTINE EVANGELISTA, MD (TOP 9 - FEB 2014 MED BOARDS; TOPNOTCH MD)

MIDTERM 2 EXAM - AUG 2014

Spectinomycin is an aminoglycoside that inhibits protein synthesis by binding to 30s subunit. It is the drug of choice of drug-resistant gonorrhea and for gonorrhea in penicillin-allergic patients.

JULIET KRISTINE EVANGELISTA, MD (TOP 9 - FEB 2014 MED BOARDS; TOPNOTCH MD)

MIDTERM 2 EXAM - AUG 2014

449

Drugs like quinidine, procainamide and disopyramide are very useful in treatment of: A. Congestive heart failure B. Hypertension C. Thyrotoxicosis D. Status asthmaticus E. Arrhythmia

Drugs like quinidine, procainamide and disopyramide are very useful in treatment of arrhythmia.

JULIET KRISTINE EVANGELISTA, MD (TOP 9 - FEB 2014 MED BOARDS; TOPNOTCH MD)

MIDTERM 2 EXAM - AUG 2014

450

It is an anticonvulsant that is effective for neuropathic pain: A. Valproic acid B. Gabapentin C. Phenytoin D. Duloxetine E. Levetiracetam

Gabapentin is an anticonvulsant drug, a GABA derivative which blocks calcium channels, increases GABA release, very useful in neuropathic pain also in postherpetic neuralgia.

JULIET KRISTINE EVANGELISTA, MD (TOP 9 - FEB 2014 MED BOARDS; TOPNOTCH MD)

MIDTERM 2 EXAM - AUG 2014

451

A 43 year-old male was diagnosed with Pulmonary Tuberculosis. He was given Anti-TB medications. However, he developed hyperuricemia. Which among these drugs inhibits the excretion of urate causing the above findings: A. Ethionamide B. Streptomycin C. Ethambutol D. Pyrazinamide E. Rifampicin

Pyrazinamide provoke avute gouty arthritis by inhibiting urate excretion.

JULIET KRISTINE EVANGELISTA, MD (TOP 9 - FEB 2014 MED BOARDS; TOPNOTCH MD)

MIDTERM 2 EXAM - AUG 2014

452

A 28 year-old woman wanted to get pregnant. A substance that enhances probability of ovulation by blocking the inhibitory effect of estrogen and thus stimulating the release of gonadotrophin from the pituitary is: A. Tamoxifen B. Danazol C. Clomiphene D. Progesterone E. Oxytocin

Clomiphene citrate enhances probability of ovulation by blocking the inhibitory effect of estrogen and thus stimulating the release of gonadotrophin from the pituitary.

JULIET KRISTINE EVANGELISTA, MD (TOP 9 - FEB 2014 MED BOARDS; TOPNOTCH MD)

MIDTERM 2 EXAM - AUG 2014

446

447

448

QUESTION

TOPNOTCH MEDICAL BOARD PREP PHARMACOLOGY SUPEREXAM Page 51 of 83 For inquiries visit www.topnotchboardprep.com.ph or email us at [email protected]

TOPNOTCH EXAM MIDTERM 2 EXAM - AUG 2014

TOPNOTCH MEDICAL BOARD PREP PHARMACOLOGY SUPEREXAM For inquiries visit www.topnotchboardprep.com.ph or email us at [email protected] Item # 453

QUESTION

EXPLANATION

AUTHOR

TOPNOTCH EXAM MIDTERM 2 EXAM - AUG 2014

Upon taking Penicillin, a 20 year-old male suddenly experienced respiratory distress. A drug would be most helpful to treat the respiratory symptoms was given in this patient. In comparison to norepinephrine, epinephrine has more effects on: A. alpha 1 receptors B. alpha 2 receptors C. beta 1 receptors D. beta 2 receptors E. dopamine receptors

Epinehrine is a sympathomimetic, nonselective and direct-acting. It is helpful in patients with anaphylactic shock. Respiratory distress is treated by Epinephrine's action on beta 2 receptors resulting to bronchodilation.

JULIET KRISTINE EVANGELISTA, MD (TOP 9 - FEB 2014 MED BOARDS; TOPNOTCH MD)

454

Warfarin is prescribed to a 40 year-old male. The excessive anticoagulant effect results to bleeding in this patient can be reversed by: A. Administration of vitamin K B. Administration of fresh frozen plasma C. Discontinuance of the drug D. A and C E. All of the above

Bleeding with warfarin can be reversed by administration of Vitamin K, FFP and by discontinuance of the drug.

JULIET KRISTINE EVANGELISTA, MD (TOP 9 - FEB 2014 MED BOARDS; TOPNOTCH MD)

MIDTERM 2 EXAM - AUG 2014

455

An anticancer agent that is prescribed for melanoma: A. Carmustine B. Cytarabine C. Vincristine D. Vinblastine E. Cisplatin

Carmustine is an alkylating agent, cell cycle nonspecific that is given for melanoma.

JULIET KRISTINE EVANGELISTA, MD (TOP 9 - FEB 2014 MED BOARDS; TOPNOTCH MD)

MIDTERM 2 EXAM - AUG 2014

456

An antihelminthic agent for strongyloidiasis: A. Nifurtimox B. Praziquantel C. Ivermectin D. Diethylcarbamazine E. Niclosamide

Ivermectin is the drug of choice for strongyloidiasis. It intensifies GABA-mediated neurotransmission in nematodes and immobilizes parasites.

JULIET KRISTINE EVANGELISTA, MD (TOP 9 - FEB 2014 MED BOARDS; TOPNOTCH MD)

MIDTERM 2 EXAM - AUG 2014

457

A 60 year-old female patient was diagnosed with breast cancer. She was given an alkylating agent, Cyclophosphamide. Later in the treatment, she developed hemorrhagic cystitis due to this toxic metabolite: A. Amifostine B. Acrolein C. Mercaptopurine D. Lomustine E. Mesna

Acrolein is the metabolite of Cyclophosphamide that causes hemorrhagic cystitis.

JULIET KRISTINE EVANGELISTA, MD (TOP 9 - FEB 2014 MED BOARDS; TOPNOTCH MD)

MIDTERM 2 EXAM - AUG 2014

458

Chlorpromazine may be used not only in treating schzophrenia but is also effective in: A. Reducing nausea and vomiting B. Allergies C. Hypertension D. Treating bipolar disorders E. Sleep disorders

Chlorpromazine may be used not only in treating schzophrenia but is also effective in reducing nausea and vomiting.

JULIET KRISTINE EVANGELISTA, MD (TOP 9 - FEB 2014 MED BOARDS; TOPNOTCH MD)

MIDTERM 2 EXAM - AUG 2014

459

The best treatment option for an obese patient with Type 2 Diabetes Mellitus is: A. Glibenclamide B. Metformin C. Repaglinide D. Acarbose E. Insulin

Metformin is a biguanide which is the first-line treatment for Type 2 DM, also a drug of choice for obese diabetics.

JULIET KRISTINE EVANGELISTA, MD (TOP 9 - FEB 2014 MED BOARDS; TOPNOTCH MD)

MIDTERM 2 EXAM - AUG 2014

460

A 10 year-old child with asthma was prescribed with a drug which inhibits binding of leukotriene D4 to its receptor on target tissues. The drug prescribed is: A. Zileuton B. Ipratropium C. Montelukast D. Omalizumab E. Theophylline

Montelukast blocks leukotriene receptor for leukotrienes C4, D4, E4 preventing airway inflammation and bronchoconstriction in asthmatic patients.

JULIET KRISTINE EVANGELISTA, MD (TOP 9 - FEB 2014 MED BOARDS; TOPNOTCH MD)

MIDTERM 2 EXAM - AUG 2014

461

The most dangerous hallucinogenic agent. A. LSD B. phencyclidine C. MDMA D. cocaine E. Marijuana



LUISA SARANILLO, MD (TOP 6 - FEB 2014 MED BOARDS; TOPNOTCH MD)

BACK-UP MIDTERM EXAM AUG 2014 - FOR INCLUSION IN THE SAMPLEX

462

A 50 year old smoker, hypertensive patient complained of 2 month cough and dyspnea. He was on Carvedilol for his hypertension. What is the side effect of this drug that aggravates the condition. A. bronchospasm B. bronchoconstriction C. tachycardia D. tremor E. all of the choices



LUISA SARANILLO, MD (TOP 6 - FEB 2014 MED BOARDS; TOPNOTCH MD)

BACK-UP MIDTERM EXAM AUG 2014 - FOR INCLUSION IN THE SAMPLEX

TOPNOTCH MEDICAL BOARD PREP PHARMACOLOGY SUPEREXAM Page 52 of 83 For inquiries visit www.topnotchboardprep.com.ph or email us at [email protected]

TOPNOTCH MEDICAL BOARD PREP PHARMACOLOGY SUPEREXAM For inquiries visit www.topnotchboardprep.com.ph or email us at [email protected] Item # 463

Mr. X was maintaining an unrecalled drug for the past few months. He is now complaining of visual changes or "yellow halo vision". What is the drug that is known to cause such effect? A. amiodarone B. ethambutol C. digoxin D. furosemide E. Procainamide A 45 year old male complained of chest pain upon exertion but relieved by rest. The doctor prescribed him a beta blocker. What is the effect of beta blocker? A. Increase oxygen supply B. Decrease oxygen demand C. Increase oxygen demand D. decrease oxygen supply E. A and B



LUISA SARANILLO, MD (TOP 6 - FEB 2014 MED BOARDS; TOPNOTCH MD)

TOPNOTCH EXAM BACK-UP MIDTERM EXAM AUG 2014 - FOR INCLUSION IN THE SAMPLEX

drugs used in angina pectoris are the following: those that increase the oxygen supply are: nitrates and calcium blockers; those that decrease the oxygen demand are: beta blockers and still calcium blockers

LUISA SARANILLO, MD (TOP 6 - FEB 2014 MED BOARDS; TOPNOTCH MD)

BACK-UP MIDTERM EXAM AUG 2014 - FOR INCLUSION IN THE SAMPLEX

465

Monday disease is caused by occupational exposure to: A. lead B. dust C. arsenic D. nitrates E. Gun powder



LUISA SARANILLO, MD (TOP 6 - FEB 2014 MED BOARDS; TOPNOTCH MD)

BACK-UP MIDTERM EXAM AUG 2014 - FOR INCLUSION IN THE SAMPLEX

466

What is the predominant form of a weakly acidic drug with pKa of 5 at physiologic pH? A. Unprotonated charged B. Protonated charged C. Unprotonated uncharged D. Protonated uncharged E. None of the choices

The physiologic pH is at 7.35-7.45 which is above the pKa of this drug 5. At pH above pKa, weakly acidic drug is predominantly unprotonated charged, if it is below pKa, the predominant form is protonated uncharged.

LUISA SARANILLO, MD (TOP 6 - FEB 2014 MED BOARDS; TOPNOTCH MD)

BACK-UP MIDTERM EXAM AUG 2014 - FOR INCLUSION IN THE SAMPLEX

467

It is the flow or transfer of a drug to the bloodstream. A. distribution B. bioequivalence C. absorption D. bioavailability E. Elimination

Absorption is the transfer of a drug to the bloodstream. Distribution is the entry of a drug to the target organ. Elimination is the termination of drug action. Bioavailability is the fraction of a drug that reaches the systemic circulation. Bioequivalence is the comparability of 2 related drugsnto achieve peak blood concetration.

LUISA SARANILLO, MD (TOP 6 - FEB 2014 MED BOARDS; TOPNOTCH MD)

BACK-UP MIDTERM EXAM AUG 2014 - FOR INCLUSION IN THE SAMPLEX

468

What type of muscarinic receptors is used by the heart? A. M1 B. M2 C. M3 D. N1 E. N2

M1 - nerve endings, M2-heart, M3 - smooth muscle, glands, endothelium

LUISA SARANILLO, MD (TOP 6 - FEB 2014 MED BOARDS; TOPNOTCH MD)

BACK-UP MIDTERM EXAM AUG 2014 - FOR INCLUSION IN THE SAMPLEX

469

Rivastigmine is used for: A. Motion sickness B. Myasthenia gravis C. Schizophrenia D. Parkinson's disease E. Alzheimer's disease

Rivastigmine is a cholinomimetic drug used for Alzheimer's disease.

LUISA SARANILLO, MD (TOP 6 - FEB 2014 MED BOARDS; TOPNOTCH MD)

BACK-UP MIDTERM EXAM AUG 2014 - FOR INCLUSION IN THE SAMPLEX

470

The drug that is used for cardiogenic shock as a last resort. A. epinephrine B. dopamine C. dobutamine D. norepinephrine E. Vasopressin



LUISA SARANILLO, MD (TOP 6 - FEB 2014 MED BOARDS; TOPNOTCH MD)

BACK-UP MIDTERM EXAM AUG 2014 - FOR INCLUSION IN THE SAMPLEX

471

A 58 year old male hypertensive was diagnosed with benign prostatic hyperplasia. What is the antihypertensive drug that is the most selective for prostatic smooth muscle? A. prazosin B. tamsulosin C. losartan D. metoprolol E. Terazosin A 75 year old male diabetic, hypertensive and a diagnosed case of CHF came in due to edema and dyspnea. If the patient had hypokalemia, a potassium sparing diuretic is indicated. This potassium sparing diuretic inhibits ENaC - epithelial sodium channel in collecting duct. A. Amiloride B. triamterene C. spironolactone D. all of the above E. A and B only

Alpha1 blockers are antihypertensive that is also used for BPH. Prazosin, tamsulosin, and terazosin are alpha1 blockers, but the most selective for prostatic smooth muscle is tamsulosin.

LUISA SARANILLO, MD (TOP 6 - FEB 2014 MED BOARDS; TOPNOTCH MD)

BACK-UP MIDTERM EXAM AUG 2014 - FOR INCLUSION IN THE SAMPLEX

Spironolactone, triamterene, and amiloride are all potassium sparing diuretic. Triamterene and amiloride inhibit the ENaC of collecting duct, while spironolactone inhibits aldosterone receptor in collecting ducts.

LUISA SARANILLO, MD (TOP 6 - FEB 2014 MED BOARDS; TOPNOTCH MD)

BACK-UP MIDTERM EXAM AUG 2014 - FOR INCLUSION IN THE SAMPLEX

464

472

QUESTION

EXPLANATION

AUTHOR

TOPNOTCH MEDICAL BOARD PREP PHARMACOLOGY SUPEREXAM Page 53 of 83 For inquiries visit www.topnotchboardprep.com.ph or email us at [email protected]

TOPNOTCH MEDICAL BOARD PREP PHARMACOLOGY SUPEREXAM For inquiries visit www.topnotchboardprep.com.ph or email us at [email protected] Item # 473

A 100kg male patient had elevated LDL and VLDL. What vitamin is also used as antihyperlipidemic drug? A. Vitamin A B. pyridoxine C. niacin D. resin E. Cholestyramine



LUISA SARANILLO, MD (TOP 6 - FEB 2014 MED BOARDS; TOPNOTCH MD)

TOPNOTCH EXAM BACK-UP MIDTERM EXAM AUG 2014 - FOR INCLUSION IN THE SAMPLEX

474

What is the drug of choice for paroxysmal supraventricular tachycardia? A. Esmolol B. adenosine C. verapamil D. Sotalol E. Any of the choices

Esmolol, sotalol, verapamil are all used for the treatment of supraventricular tachycardia. For paroxysmal supraventricular tachycardia, the drug of choice is adenosine.

LUISA SARANILLO, MD (TOP 6 - FEB 2014 MED BOARDS; TOPNOTCH MD)

BACK-UP MIDTERM EXAM AUG 2014 - FOR INCLUSION IN THE SAMPLEX

475

A 10 year old child, known asthmatic had episodes of nocturnal asthma attacks approximately 2-3x/week. In order to prevent subsequent attack, what drug is used for prophylaxis against nocturnal attack? A. Salbutamol + Ipratropium B. Salbutamol C. salmeterol D. cromolyn E. Aminophylline

Salbutamol is the druig of choice for acute asthma attack, while aminophylline is used for prophylaxis against nocturnal attacks.

LUISA SARANILLO, MD (TOP 6 - FEB 2014 MED BOARDS; TOPNOTCH MD)

BACK-UP MIDTERM EXAM AUG 2014 - FOR INCLUSION IN THE SAMPLEX

476

A 4 year old child, febrile at 38deg. C was given Paracetamol at 15mkd. What is the mechanism of action of this drug? A. COX-2 selective inhibitor B. IL-1 and IL-6 inhibitors C. COX-3 selective inhibitor D. Irreversible COX-1 and COX-2 inhibitor E. TNF - α inhibitor

Paracetamol or acetaminophen selectively inhibits COX-3. COX-2 selective inhibitors are coxib such as celecoxib. Irreversible COX-1 and 2 inhibitor is aspirin. TNF - α inhibitors are infliximab, adalimumab and etanercept.

LUISA SARANILLO, MD (TOP 6 - FEB 2014 MED BOARDS; TOPNOTCH MD)

BACK-UP MIDTERM EXAM AUG 2014 - FOR INCLUSION IN THE SAMPLEX

477

One of the choices does not belong to the third generation cephalosporins. A. cefotaxime B. ceftazidime C. ceftriaxone D. cefoxitin E. Ceftizoxime

All of the choices except cefoxitin are all third generation cephalosporin. Cefoxitin is a second generation.

LUISA SARANILLO, MD (TOP 6 - FEB 2014 MED BOARDS; TOPNOTCH MD)

BACK-UP MIDTERM EXAM AUG 2014 - FOR INCLUSION IN THE SAMPLEX

478

A 32 year old G2P1 mother delivered a live term baby Boy with an upturned nose, mild midfacial hypoplasia, long upper lip, and lower distal digital hypoplasia. Upon review of maternal history, the mother was taking an anti-epileptic drug. The drug that is most likely used was: A. phenytoin B. valproic acid C. diazepam D. carbamazepine E. lamotrigine A 50 year old obese patient was diagnosed to have DM type 2. There were no other comorbidities. What hypoglycemic drug should you prescribed with this patient? A. Insulin B. Gliclazide C. Metformin D. Pioglitazone E. Sitagliptin Penicillin is the drug of choice for syphilis. It acts through: A. Inhibition of transpeptidase B. Inhibition of topoisomerase II C. Inhibition of glucan synthase D. Inhibition of D-ala-D-ala terminus E. None of the choices

The physical description of the neonate is consistent with fetal hydantoin syndrome which is the side effect of phenytoin

LUISA SARANILLO, MD (TOP 6 - FEB 2014 MED BOARDS; TOPNOTCH MD)

BACK-UP MIDTERM EXAM AUG 2014 - FOR INCLUSION IN THE SAMPLEX

Metformin is the first line treatment for type 2 DM and is the drug of choice for obese patients. It is contraindicated in patients with renal disease, hepatic disease, and alcoholism

LUISA SARANILLO, MD (TOP 6 - FEB 2014 MED BOARDS; TOPNOTCH MD)

BACK-UP MIDTERM EXAM AUG 2014 - FOR INCLUSION IN THE SAMPLEX

Penicillin inhibits transpeptidase enzyme needed for the bacterial cell wall synthesis. Vancomycin inhibits D-ala D ala synthase. Caspofungin inhibits glucan synthase. Flouroquinolone inhibits topoisomerase II and IV.

LUISA SARANILLO, MD (TOP 6 - FEB 2014 MED BOARDS; TOPNOTCH MD)

BACK-UP MIDTERM EXAM AUG 2014 - FOR INCLUSION IN THE SAMPLEX

Marc, an asthmatic, is on theophylline. He has a number of other medical conditions. Which of the following medications may be safely administered with theophylline? A. Penicillin B. Erythromycin C. Phenytoin D. Rifampicin What is the maximum dose of Vincristine? A. 100 micrograms B. 200 micrograms C. 1 milligram D. 2 milligrams

Theophylline has a narrow therapeutic window and drug interactions can lead to severe morbidity. Erythromycin decreases the clearance of theophylline and increases its serum levels. Phenytoin and rifampicin have the oppositve effect. Advice: Memorize the inducers and inhibitors of the CYP450 system!

ANGELIS ANDREA COCOS, MD (TOP 1 - FEB 2014 MED BOARDS; TOPNOTCH MD)

FINAL EXAM - AUG 2014

SIMILAR TO PREVIOUS BOARD EXAM CONCEPT/PRINCIPLE

ANGELIS ANDREA COCOS, MD (TOP 1 - FEB 2014 MED BOARDS; TOPNOTCH MD)

FINAL EXAM - AUG 2014

479

480

481

482

QUESTION

EXPLANATION

AUTHOR

TOPNOTCH MEDICAL BOARD PREP PHARMACOLOGY SUPEREXAM Page 54 of 83 For inquiries visit www.topnotchboardprep.com.ph or email us at [email protected]

TOPNOTCH MEDICAL BOARD PREP PHARMACOLOGY SUPEREXAM For inquiries visit www.topnotchboardprep.com.ph or email us at [email protected] Item # 483

QUESTION

EXPLANATION

AUTHOR

TOPNOTCH EXAM FINAL EXAM - AUG 2014

A pregnant patient complains of multiple external genital warts. Which of the following agents can be used in this patient? A. Podofilox solution B. Imiquimod cream C. Trichloroacetic acid D. Sinecatechins ointment

The most common treatments for genital warts in pregnancy are cryotherapy, laser removal, and trichloroacetic acid. The rest of the choices are under Pregnancy Category C.

ANGELIS ANDREA COCOS, MD (TOP 1 - FEB 2014 MED BOARDS; TOPNOTCH MD)

484

The drug of choice for leptospirosis is: A. Doxycycline B. Penicillin C. Ceftriaxone D. Erythromycin

SIMILAR TO PREVIOUS BOARD EXAM CONCEPT/PRINCIPLE The rest of the choices are alternate drug choices for leptospirosis.

ANGELIS ANDREA COCOS, MD (TOP 1 - FEB 2014 MED BOARDS; TOPNOTCH MD)

FINAL EXAM - AUG 2014

485

A 23-year-old presents with abdominal pain, tenesmus, and mucoid diarrhea with frank blood. Which is the best empiric treatment? A. Ciprofloxacin B. Metronidazole C. Amoxicillin D. Cotrimoxazole

Ciprofloxacin is the recommended empiric treatment for patients with acute dysenteriae. SIMILAR TO PREVIOUS BOARD EXAM CONCEPT/PRINCIPLE

ANGELIS ANDREA COCOS, MD (TOP 1 - FEB 2014 MED BOARDS; TOPNOTCH MD)

FINAL EXAM - AUG 2014

486

The most powerful hallucinogen known to man is: A. LSD B. marijuana C. methamphetamine D. phencyclidine

SIMILAR TO PREVIOUS BOARD EXAM CONCEPT/PRINCIPLE

ANGELIS ANDREA COCOS, MD (TOP 1 - FEB 2014 MED BOARDS; TOPNOTCH MD)

FINAL EXAM - AUG 2014

487

It is the only licensed chemotherapy drug for advanced melanoma: A. adriamycin B. bleomycin C. cisplatin D. Dacarbazine

SIMILAR TO PREVIOUS BOARD EXAM CONCEPT/PRINCIPLE There was quite a number of questions on oncology drugs.

ANGELIS ANDREA COCOS, MD (TOP 1 - FEB 2014 MED BOARDS; TOPNOTCH MD)

FINAL EXAM - AUG 2014

488

Plasma magnesium levels should be maintained at this therapeutic range to prevent eclamptic convulsions: A. 2-4 meq/L B. 4-7 meq/L C. 8-10 meq/L D. 10-12 meq/L



ANGELIS ANDREA COCOS, MD (TOP 1 - FEB 2014 MED BOARDS; TOPNOTCH MD)

FINAL EXAM - AUG 2014

489

This is the only ligand-gated serotonin receptor: A. 5HT1 B. 5HT2 C. 5HT3 D. 5HT4



ANGELIS ANDREA COCOS, MD (TOP 1 - FEB 2014 MED BOARDS; TOPNOTCH MD)

FINAL EXAM - AUG 2014

490

This autonomic drug should be given with caution in patients with depressive disorders: A. guanethidine B. metyrosine C. vesamicol D. Reserpine

Reserpine which crosses the blood brain barrier is notorious for causing depression and increasing suicidal tendencies.

ANGELIS ANDREA COCOS, MD (TOP 1 - FEB 2014 MED BOARDS; TOPNOTCH MD)

FINAL EXAM - AUG 2014

491

If a patient has COPD, which of the following betablockers would be more preferrable to use, theoretically? A. propranolol B. carvedilol C. labetalol D. atenolol

Atenolol, metoprolol, esmolol, betaxolol, and acebutolol (A-BEAM) are more beta1-selective hence are advantageous in patients with comorbid pulmonary disease.

ANGELIS ANDREA COCOS, MD (TOP 1 - FEB 2014 MED BOARDS; TOPNOTCH MD)

FINAL EXAM - AUG 2014

492

An 8-year-old being treated with a combination of chemotherapeutic agents had difficulty eating and drinking due to red, inflamed sores in her mouth and esophagus. Which of the following antineoplastic agents is the most likely etiology? A. Methotrexate B. Vinblastine C. Doxorubicin D. Prednisone A young boy with G6PD deficiency would note teacolored urine after the intake of which drug/s: A. primaquine B. cotrimoxazole C. nitrofurantoin D. All of the above

Major toxicities of methotrexate include gastrointestinal mucositis, bone marrow suppression, skin erythema and hepatic dysfunction. SIMILAR TO PREVIOUS BOARD EXAM CONCEPT/PRINCIPLE

ANGELIS ANDREA COCOS, MD (TOP 1 - FEB 2014 MED BOARDS; TOPNOTCH MD)

FINAL EXAM - AUG 2014



ANGELIS ANDREA COCOS, MD (TOP 1 - FEB 2014 MED BOARDS; TOPNOTCH MD)

FINAL EXAM - AUG 2014

This drug activates peroxisome proliferator-activated receptors, a group of nuclear receptors, with greatest specificity to PPAR-γ receptors: A. pioglitazone B. gemfibrozil C. sitagliptin D. Exenatide

Do not confuse with the mechanism of action of fibrates which is activation of the nuclear transcription factor PPAR-α.

ANGELIS ANDREA COCOS, MD (TOP 1 - FEB 2014 MED BOARDS; TOPNOTCH MD)

FINAL EXAM - AUG 2014

493

494

TOPNOTCH MEDICAL BOARD PREP PHARMACOLOGY SUPEREXAM Page 55 of 83 For inquiries visit www.topnotchboardprep.com.ph or email us at [email protected]

TOPNOTCH MEDICAL BOARD PREP PHARMACOLOGY SUPEREXAM For inquiries visit www.topnotchboardprep.com.ph or email us at [email protected] Item # 495

QUESTION

EXPLANATION

AUTHOR

TOPNOTCH EXAM FINAL EXAM - AUG 2014

Constipation is one of the major side effects of this drug: A. Magnesium hydroxide B. Aluminum hydroxide C. Omeprazole D. Colchicine

Aluminum hydroxide causes constipation (ALang tae). Magnesium hydroxide causes diarrhea (MaGtatae). Colchicine also causes diarrhea if taken in excess.

ANGELIS ANDREA COCOS, MD (TOP 1 - FEB 2014 MED BOARDS; TOPNOTCH MD)

496

A patient presents with petit mal seizures. The drug of choice for this type of seizure is: A. phenytoin B. topiramate C. ethosuximide D. lamotrigine

Petit mal seizures, also known as absence seizures, involves a brief, sudden lapse of consciousness. The DOC is ethosuximide.

ANGELIS ANDREA COCOS, MD (TOP 1 - FEB 2014 MED BOARDS; TOPNOTCH MD)

FINAL EXAM - AUG 2014

497

A psychotic patient on haloperidol presents with rigidity, hyperpyrexia and autonomic instability. This condition is known as: A. Serotonin syndrome B. Neuroleptic malignant syndrome C. Tardive dyskinesia D. Malignant hyperthermia

Tardive dyskinesia is also one of the toxicities of antipsychotics aside from NMS. It is a condition where the patient presents with stereotypic oralfacial movements.

ANGELIS ANDREA COCOS, MD (TOP 1 - FEB 2014 MED BOARDS; TOPNOTCH MD)

FINAL EXAM - AUG 2014

498

This diuretic can also be used in the treatment of glaucoma: A. acetazolamide B. hydrochlorothiazide C. mannitol D. amiloride

Technically, mannitol can also be used but acetazolamide is a better answer. SIMILAR TO PREVIOUS BOARD EXAM CONCEPT/PRINCIPLE

ANGELIS ANDREA COCOS, MD (TOP 1 - FEB 2014 MED BOARDS; TOPNOTCH MD)

FINAL EXAM - AUG 2014

499

It is a monoclonal anti-IgE antibody used in the treatment of asthma resistant to inhaled steroids and long-acting B2-agonists: A. trastuzumab B. rituximab C. infliximab D. omalizumab

Trastuzumab aka herceptin is a monoclonal antibody that interferes with the HER2/neu receptor, rituximab is against the protein CD20 on B cells, and infliximab is against TNF-alpha.

ANGELIS ANDREA COCOS, MD (TOP 1 - FEB 2014 MED BOARDS; TOPNOTCH MD)

FINAL EXAM - AUG 2014

500

This drug inhibits platelet aggregation by irreversibly blocking ADP receptors: A. cilostazol B. tirofiban C. alteplase D. Ticagrelor

Cilostazol is a phosphodiesterase III inhibitor, tirofiban binds to GP IIb/IIIa receptor, and alteplase is a thrombolytic (not an anti-platelet) which converts plasminogen to plasmin.

ANGELIS ANDREA COCOS, MD (TOP 1 - FEB 2014 MED BOARDS; TOPNOTCH MD)

FINAL EXAM - AUG 2014

501

These drugs act by preventing the binding of the agonist molecule to the receptor but do not activate generation of a signal: A. Pharmacologic antagonists B. Inverse agonists C. Competitive agonists D. Partial agonists E. Orphan drug

Katzung 10th ed, p. 11 - Pharmacologic anatgonists bind to the receptor preventing agonist binding thereby blocking the agonist's biologic actions.

JAN CHARMAINE PALOMAR, MD (TOP 9 - FEB 2014 MED BOARDS; TOPNOTCH MD)

BACK-UP MIDTERM EXAM AUG 2014

502

This route of drug administration is the most convenient, however bioavailability may be less than 100% because of incomplete absorption and firstpass elimination. A. Intramuscular B. Subcutaneous C. Oral D. Transdermal E. All of the above Loca is a 23 year old G4P4 (4004) who has been on oral contraceptives for 8 months now. What is the expected drug-drug interaction If she was to start on an anti-tuberculosis drug that acts by inhibitting RNA synthesis and which imparts orange color to urine and sweat? A. Enhanced efficacy of the oral contraceptive she is taking B. Enhanced potency of the oral contraceptive she is taking C. reduced activity of the cytochrome P450 isoforms D. Increased elimination of the anti-tuberculosis drug E. Significantly lower serum level of the oral contraceptive It is during this phase of drug development and testing that the effects of the drug as a function of dosage are established in a small number of healthy volunteers. A. Phase 1 B. Phase 2 C. Phase 3 D. Phase 4 E. Phase 5

Katzung 10th ed, p. 41

JAN CHARMAINE PALOMAR, MD (TOP 9 - FEB 2014 MED BOARDS; TOPNOTCH MD)

BACK-UP MIDTERM EXAM AUG 2014

Rifampin revs up the cytochrome P450 enzyme system thereby enhancing the metabolism and hence the elimination of the oral contraceptive the patient is taking resulting to its significantly lower serum level. Katzung 10th ed, p.774.

JAN CHARMAINE PALOMAR, MD (TOP 9 - FEB 2014 MED BOARDS; TOPNOTCH MD)

BACK-UP MIDTERM EXAM AUG 2014

Phase 1 trials determine the probable limits of the safe clinical dosage range in a small number of healthy volunteers. In Phase 2, the drug is studied in patients with the target disease to determine its efficacy. In phase 3, the drug is evaluated in larger numbers of patients with the target disease to further establish safety and efficacy. Phase 4 constitutes monitoring the safety of the new drug under actual conditions of use in large number of patients. Katzung 10th ed., 70-71.

JAN CHARMAINE PALOMAR, MD (TOP 9 - FEB 2014 MED BOARDS; TOPNOTCH MD)

BACK-UP MIDTERM EXAM AUG 2014

503

504

TOPNOTCH MEDICAL BOARD PREP PHARMACOLOGY SUPEREXAM Page 56 of 83 For inquiries visit www.topnotchboardprep.com.ph or email us at [email protected]

TOPNOTCH MEDICAL BOARD PREP PHARMACOLOGY SUPEREXAM For inquiries visit www.topnotchboardprep.com.ph or email us at [email protected] Item # 505

506

507

508

509

QUESTION

EXPLANATION

AUTHOR

Which of the following drugs lower blood pressure by Metyrosine inhibits tyrosine hydroxylase which is JAN CHARMAINE preventing normal physiologic release of responsible for the conversion of Tyrosine to Dopa, PALOMAR, MD norepinephrine from postganglionic sympathetic the rate-limiting step in catecholamine synthesis. (TOP 9 - FEB neurons? Cocaine blocks re-uptake of NE from the synaptic 2014 MED A. Metyrosine cleft thereby prolonging its effect. Reserpine blocks BOARDS; B. Cocaine dopamine transport into the vesicle. Botulinum TOPNOTCH MD) C. Reserpine blocks Acetylcholine release from the nerve terminal D. Botulinum toxin into the junctional cleft. Study figures 6-3 and 6-4 of E. Guanethidine Katzung 10th ed, p. 79 and 81. A tricyclic antidepresssant drug long used to reduce Katzung 115: Ipratropium-is an anti-muscarinic JAN CHARMAINE incontinence in institutionalized elderly patients and drug used as an inhalational drug for asthma. PALOMAR, MD in children with bedwetting problems because of its Scopolamine- is an antimuscarinic drug used to (TOP 9 - FEB strong anti-muscarinic actions: remedy seasickness. Oxybutinin- is an 2014 MED A. Ipratropium antimuscarinic selective for M3 receptors used to BOARDS; B. Imipramine relieve bladder spasm after urologic surgery and is TOPNOTCH MD) C. Scopolamine also valuable in reducing involuntary voiding in D. Oxybutinin patients with neurologic disease. Fluoxetine is an E. Fluoxetine SSRI. Which of the following statements regarding diuretics The collecting tubule and not the distal tubule is the JAN CHARMAINE is not true? final site NaCl reabsorption. It is responsible for PALOMAR, MD A. Diuretics lower blood pressure primarily by tight regulation of body fluid volume and for (TOP 9 - FEB depleting body sodium stores. determining the final sodium concentration of the 2014 MED B. The most common adverse effect of thiazide urine. Katzung 10th ed., 239. BOARDS; diuretics is potassium depletion. TOPNOTCH MD) C. The intercalated cells in the collecting tubule are the primary sites of H+ secretion. D. The distal convoluted tubule is the final site of NaCl reabsorption and is responsible for the final sodium concentration of the urine. E. The thick ascending limb of the loop of Henle is the "diluting segment". Which of the following drug acts by selectively Katzung 10th ed., 169. Metoprolol, Esmolol, JAN CHARMAINE inhibitting stimulation of B1 adrenoceptor and may Atenolol, Acebutolol, Betaxolol, Bisoprolol are PALOMAR, MD be advantageous in treating hypertensive patients selective B1-blockers. Propranolol and Nadolol are (TOP 9 - FEB who also suffer from asthma, diabetes, or peripheral non-selective B-blockers. Labetalol and Carvedilol 2014 MED vascular disease? have combined alpha and b-blocking activity. BOARDS; A. Nadolol TOPNOTCH MD) B. Metoprolol C. Propranolol D. Labetalol E. Carvedilol

TOPNOTCH EXAM BACK-UP MIDTERM EXAM AUG 2014

BACK-UP MIDTERM EXAM AUG 2014

BACK-UP MIDTERM EXAM AUG 2014

BACK-UP MIDTERM EXAM AUG 2014

It is a very efficacious orally active arteriolar vasodilator that is also available in topical preparations used to stimulate hair growth for correction of baldness. A. Minoxidil B. Sodium Nitroprusside C. Hydralazine D. Diazoxide E. Fenoldopam A Class I-B anti-arrhythmic drug which has a high degree of effectiveness in arrhythmias associated with acute myocardial infarction. A. Quinidine B. Flecainide C. Propranolol D. Lidocaine E. Amiodarone

Katzung 10th ed, 173.

JAN CHARMAINE PALOMAR, MD (TOP 9 - FEB 2014 MED BOARDS; TOPNOTCH MD)

BACK-UP MIDTERM EXAM AUG 2014

Katzung 10th ed., 225 Quinidine- Class IA, associated with cinchonism; Flecainide- Class IC, very effective in suppressing premature vebtricular contractions; Propranolol-Class 2, Amiodarone-broad spectrum of actions

JAN CHARMAINE PALOMAR, MD (TOP 9 - FEB 2014 MED BOARDS; TOPNOTCH MD)

BACK-UP MIDTERM EXAM AUG 2014

511

A potent antiandrogen that is usually coadministered with leuprolide that has been used in the treatment of prostatic carcinoma that frequently causes gynecomastia and occassionally mild reversible hepatic toxicity: A. Goserelin B. Spironolactone C. Cyproterone D. Ketoconazole E. Flutamide

Katzung 10th ed., 679. this is SIMILAR TO PREVIOUS BOARD EXAM CONCEPT/PRINCIPLE

JAN CHARMAINE PALOMAR, MD (TOP 9 - FEB 2014 MED BOARDS; TOPNOTCH MD)

BACK-UP MIDTERM EXAM AUG 2014

512

Disulfiram causes extreme discomfort in patients who drink alcoholic beverages by inhibitting which enzyme resulting to accumulation of acetaldehyde causing flushing, throbbing headache, nausea, vomiting, sweating, hypotension and confusion within few hours of drinking alcohol: A. alcohol dehydrogenase B. aldehyde dehydrogenase C. alcohol decarboxylase D. aldehyde decarboxylase E. none of the above

Katzung 10th ed., 371. this was SIMILAR TO PREVIOUS BOARD EXAM CONCEPT/PRINCIPLE

JAN CHARMAINE PALOMAR, MD (TOP 9 - FEB 2014 MED BOARDS; TOPNOTCH MD)

BACK-UP MIDTERM EXAM AUG 2014

510

TOPNOTCH MEDICAL BOARD PREP PHARMACOLOGY SUPEREXAM Page 57 of 83 For inquiries visit www.topnotchboardprep.com.ph or email us at [email protected]

TOPNOTCH MEDICAL BOARD PREP PHARMACOLOGY SUPEREXAM For inquiries visit www.topnotchboardprep.com.ph or email us at [email protected] Item # 513

QUESTION The following helminths are matched with their respective drug of choice except: A. Fasciola hepatica - Bithionol B. Echinococcus granulosus - Albendazole C. Stongyloides stercoralis - Thiabendazole D. Schistosoma mansoni - Praziquantel E. Dracunculus medinensis - Metronidazole

514

What is the drug of choice for the eradication of dormant liver forms of Plasmodium vivax and Plasmodium ovale? A. Quinidine B. Primaquine C. Mefloqione D. Chloroquine E. Lumefantrine

515

This halogen in a 1:20,000 solution is bactericidal in 1 minute and kills spores in 15 minutes. It is the most active antiseptic for intact skin: A. Phenol B. Hypochlorus acid C. Sodium hypochlorite D. Iodine E. Hexachlorophene

516

This antiviral agent against Influenza A acts by blocking the M2 proton ion channel of the virus particle and by inhibitting uncoating of the viral RNA within infected host cells, thus preventing its replication. A. Amantadine B. Ribavirin C. Oseltamivir D. Entecavir E. None of the abpve This drug is a derivative of rifamycin which is indicated in place of rifampin for treatment of tuberculosis in HIV-infected patients who are receiving concurrent protease inhibitors: A. Rifapentine B. Rifabutin C. Ritonavir D. Rifadin E. Rimactane

517

518

A 35 year old woman came to your clinic because of dysuria. She has been experiencing burning on urination with increased frequency and urgency to urinate for 5 days now. Urinalysis revealed WBC of 20-30/hpf, bacteria=moderate, nitrite=positive. You may prescribe the following drugs to her except for: A. Ciprofloxacin B. Levofloxacin C. Ofloxacin D. Moxifloxacin E. None of the above

519

The following are true of sulfonamides except: A. Sulfonamides are structural analogs of paminobenzoic acid and act by inhibitting dihydropteroate synthase and folate production. B. Sulfonamides are very effective against rickettsiae. C. Combination of a sulfonamide with an inhibitor of dihydrofolate reductase provides synergistic activity because of sequential inhibition of folate synthesis. D. trimethoprim-sulfamethoxazole is the drug of choice for infections such as Pneumocystis jiroveci pneumonia. E. Sulfadiazine with pyrimethamine is first-line therapy for treatment of acute toxoplasmosis. A 45 year old soldier has been maintaining on fluoxetine for his major depressive disorder. Which of the following drugs can precipitate a potentially fatal syndrome of hypertension, hyperreflexia, tremor, clonus, hyperhtermia, diarrhea, mydriasis and agitation within hours when taken consurrently with fluoxetine? A. Amytriptyline B. Imipramine C. Sertraline D. Citalopram E. Phenelzine

520

EXPLANATION

AUTHOR

The drug of choice in strongylodiasis and onchocerciasis that acts by intensifying GABAmediated transmission of signals in peripheral nerves resulting to paralysis of the worms is Ivermectin. Thiabendazole is an alternative drug only. Katzung 10th ed., 870; See also p.868 for the table of drug of choice for other helminthic infections Primaquine is the only available agent active against the dormant hypnozoite stages of p. vivax and p. ovale. (radical cure) Katzung 10th ed., 852

JAN CHARMAINE PALOMAR, MD (TOP 9 - FEB 2014 MED BOARDS; TOPNOTCH MD) JAN CHARMAINE PALOMAR, MD (TOP 9 - FEB 2014 MED BOARDS; TOPNOTCH MD)

BACK-UP MIDTERM EXAM AUG 2014

Katzung 10th ed., 822. Phenolic disinfectants are the oldest of the surgical antiseptics. They are used for hard surface decontamination in hospitals and labs. They are no longer used as a disinfectant because of its corrosive effect on tissues, its toxicity when absorbed (hyperbilirubinemia in newborns) and its carcinogenic effect. Sodium hypochlorite (household bleach) in 1:10 dilution which provides 5000 ppm of available chlorine is recommended by CDC for disinfection of blood spills and to kill spores. katzung 10th ed., 815.

JAN CHARMAINE PALOMAR, MD (TOP 9 - FEB 2014 MED BOARDS; TOPNOTCH MD)

BACK-UP MIDTERM EXAM AUG 2014

JAN CHARMAINE PALOMAR, MD (TOP 9 - FEB 2014 MED BOARDS; TOPNOTCH MD)

BACK-UP MIDTERM EXAM AUG 2014

Katzung 10th ed., 777. Rifabutin is a less potent inducer of the cytochrome P450 enzyme (compared to Rifampin) resulting to slower elimination and longer halflife of protease inhibitors used in treating HIV patients is cases of HIV with concurrent tuberculosis. Rifapentine like Rifampin is a potent inducer of cytochrome p450 enzymes and should not be used to treat HIV-infected patients because of an unaccepptably high relapse rate with rifampinresistant organisms. Rtonavir is a protease inhibitor. Rifadin and Rimactane are brand names of Rifampin. Quinolones act by blocking bacterial DNA synthesis by inhibitting bacterial topoisomerase II and IV. Moxifloxacin is the only non-renally cleared fluoroqionolone and thus achieves low urinary level making it ineffective for cases of UTI. Katzung 10th ed., 768.

JAN CHARMAINE PALOMAR, MD (TOP 9 - FEB 2014 MED BOARDS; TOPNOTCH MD)

BACK-UP MIDTERM EXAM AUG 2014

JAN CHARMAINE PALOMAR, MD (TOP 9 - FEB 2014 MED BOARDS; TOPNOTCH MD)

BACK-UP MIDTERM EXAM AUG 2014

It is interesting that rickettsiae are not inhibitted by sulfonamides but are actually stimulated in their growth. Katzung 10th ed., 763.

JAN CHARMAINE PALOMAR, MD (TOP 9 - FEB 2014 MED BOARDS; TOPNOTCH MD)

BACK-UP MIDTERM EXAM AUG 2014

This is a case of serotonin syndrome which is a condition precipitated when MAO inhibitors are given with serotonin agaonists, especially antidepressants of the SSRI class. Amytriptyline and Imipramine are TCAs. Fluoxetine, Sertraline and Citalopram are SSRIs. Phenelzine is a MAO inhibitor. Katzung 10th ed., 267.

JAN CHARMAINE PALOMAR, MD (TOP 9 - FEB 2014 MED BOARDS; TOPNOTCH MD)

BACK-UP MIDTERM EXAM AUG 2014

TOPNOTCH MEDICAL BOARD PREP PHARMACOLOGY SUPEREXAM Page 58 of 83 For inquiries visit www.topnotchboardprep.com.ph or email us at [email protected]

TOPNOTCH EXAM BACK-UP MIDTERM EXAM AUG 2014

TOPNOTCH MEDICAL BOARD PREP PHARMACOLOGY SUPEREXAM For inquiries visit www.topnotchboardprep.com.ph or email us at [email protected] Item # 521

522

523

524

525

QUESTION A 37 year old man comes to the physician because of a 1 week history of pain with swallowing. He received the diagnosis of AIDS 4 years ago, but has not been able to tolerate highly active antiretroviral therapy. He currently takes TMP-SMX. Vital signs are within normal limits. Examination shows a few white plaques over the pharynx. CD4+ count is 50/mm3 and plasma HIV viral load is 50000 copies/mL. Which of the following is the most appropriate pharmacotherapy? A) Acyclovir B) Amphotericin B C) Fluconazole D) Foscarnet A 72 year old man is brought to the physician by his daughter because of a 7 month history of difficulty with memory. He frequently loses his keys and misses appointments and regular family dinners. He lives alone, but recently family members had to drive him on errands and help him with daily tasks at home. He has not had difficulty sleeping and does not use illicit drugs. Mental status examination shows an irritable mood and labile affect. There is no evidence of depressed mood, anxiety, or hallucinations. Which of the following is the most appropriate pharmacotherapy for this patient? A) Alprazolam B) Donepezil C) Dextroamphetamine D) imipramine A 47 year old woman comes to the physician for a routine health maintenance examination. She has a 2 year history of venous insufficiency and noted daily swelling of her ankles that worsens in the evening. She takes no medications. Her pulse is 80 bpm, and blood pressure is 160/100 mmHg. Cardiopulmonary examination shows no abnormalities. Examination of the lower extremities shows 2+ edema and increased pigmentation .Which of the following antihypertensive medications would most likely exacerbate this patient’s swelling? A) Atenolol B) Lisinopril C) Losartan D) Nifedipine A 55-year-old man has had crushing substernal chest pain on exertion over the past 6 weeks. He had a myocardial infarction 2 months ago. He takes nitroglycerin as needed and one aspirin daily. He has smoked two packs of cigarettes daily for 30 years. Examination shows normal heart sounds and no carotid or femoral bruits. Treatment with a βadrenergic blocking agent is most likely to improve his symptoms due to which of the following mechanisms? A) Decreasing diastolic relaxation B) Decreasing myocardial contractility C) Dilating the coronary arteries D) Peripheral vasodilation A 72-year-old man comes for a routine follow-up examination. He has chronic obstructive pulmonary disease treated with β-adrenergic agonists and ipratropium by metered-dose inhaler and mild arterial insufficiency of the lower extremities treated with aspirin. His blood pressure is 160/60 mm Hg, pulse is 70/min, and respirations are 12/min. Funduscopic examination shows arteriovenous nicking. Pedal pulses are decreased bilaterally. Which of the following antihypertensive drugs is most likely to cause adverse effects in this patient? A) α2-Adrenergic agonist B) α-Adrenergic blocking agent C) β-Adrenergic blocking agent D) Angiotensin-converting enzyme (ACE) inhibitor

EXPLANATION

AUTHOR

TOPNOTCH EXAM MIDTERM 1 EXAM - FEB 2013



MIGUEL RAFAEL RAMOS, MD (TOP 3 - FEB 2012 MED BOARDS; TOPNOTCH MD)



MIGUEL RAFAEL RAMOS, MD (TOP 3 - FEB 2012 MED BOARDS; TOPNOTCH MD)

MIDTERM 1 EXAM - FEB 2013



MIGUEL RAFAEL RAMOS, MD (TOP 3 - FEB 2012 MED BOARDS; TOPNOTCH MD)

MIDTERM 1 EXAM - FEB 2013



MIGUEL RAFAEL RAMOS, MD (TOP 3 - FEB 2012 MED BOARDS; TOPNOTCH MD)

MIDTERM 1 EXAM - FEB 2013

COPD and Peripheral arterial disease >> contraindications for Beta-Blockers

MIGUEL RAFAEL RAMOS, MD (TOP 3 - FEB 2012 MED BOARDS; TOPNOTCH MD)

MIDTERM 1 EXAM - FEB 2013

TOPNOTCH MEDICAL BOARD PREP PHARMACOLOGY SUPEREXAM Page 59 of 83 For inquiries visit www.topnotchboardprep.com.ph or email us at [email protected]

TOPNOTCH MEDICAL BOARD PREP PHARMACOLOGY SUPEREXAM For inquiries visit www.topnotchboardprep.com.ph or email us at [email protected] Item # 526

527

528

529

530

QUESTION A 17-year-old boy is brought to the emergency department by his parents because of bizarre behavior for 6 hours. Last night he was out with friends, and since returning, he has been confused and has "trashed" his room. His blood pressure is 165/95 mm Hg. He is hypervigilant, has little spontaneous speech, and is disoriented to place and time. He appears catatonic but abruptly becomes assaultive two times and needs to be restrained. Which of the following is the most likely substance taken? A) Cocaine B) Ecstasy C) LSD D) PCP A 72-year-old man comes to the physician because of a 2-month history of urination twice nightly and occasional urinary frequency and urgency. He has a 15-year history of type 2 diabetes mellitus now moderately well controlled with glyburide. His father was diagnosed with prostate cancer at the age of 70 years, and his sister died of complications from systemic lupus erythematosus. His blood pressure is 135/86 mm Hg. Cardiopulmonary examination shows no abnormalities. Abdominal examination shows no suprapubic fullness or tenderness. There is mild enlargement of the prostate with no palpable nodules. His postvoid residual volume is 10 mL. Serum studies show a urea nitrogen (BUN) level of 45 mg/dL and creatinine level of 3.8 mg/dL. Urine dipstick shows 3+ protein. Which of the following is most likely to have prevented progression of this patient's renal disease? A) Oral finasteride therapy B) Oral terazosin therapy C) Oral cyclophosphamide and prednisone therapy D) Oral enalapril therapy A 47-year-old man is admitted to the hospital after threatening to harm a radio announcer he believed was broadcasting his thoughts. Over the past 20 years, he has had multiple psychiatric hospitalizations for threatening people who he believed were plotting against him, trying to control his mind, or causing him to hear voices by implanting devices in his head. Past symptoms improved with neuroleptic therapy; after discharge, he discontinued the medication and his symptoms worsened. Which of the following is the most appropriate pharmacotherapy to decrease this patient's risk for future hospitalization? A) Clozapine B) Fluphenazine hydrochloride C) Haloperidol decanoate D) Risperidone A 47-year-old man is admitted to the hospital after threatening to harm a radio announcer he believed was broadcasting his thoughts. Over the past 20 years, he has had multiple psychiatric hospitalizations for threatening people who he believed were plotting against him, trying to control his mind, or causing him to hear voices by implanting devices in his head. Past symptoms improved with neuroleptic therapy; after discharge, he discontinued the medication and his symptoms worsened. Which of the following is the most appropriate pharmacotherapy to decrease this patient's risk for future hospitalization? A) Clozapine B) Fluphenazine hydrochloride C) Haloperidol decanoate D) Risperidone A 32-year-old woman comes to the physician because of a 3-month history of increasing pain and stiffness in her wrists, hands, and ankles. During this period, she also has had progressive fatigue and morning stiffness lasting 2 hours. She has a 1-year history of rheumatoid arthritis treated with naproxen. Examination shows redness, swelling, and warmth over the wrist, hand, and ankle joints bilaterally. There are nontender subcutaneous nodules over the extensor surfaces of both elbows. X-ray films of the hands show diffuse osteopenia and erosions over several of the distal metacarpal bones. Which of the following is the most appropriate pharmacotherapy? A) Add oral cyclophosphamide B) Add oral gold

EXPLANATION

AUTHOR

TOPNOTCH EXAM MIDTERM 1 EXAM - FEB 2013



MIGUEL RAFAEL RAMOS, MD (TOP 3 - FEB 2012 MED BOARDS; TOPNOTCH MD)



MIGUEL RAFAEL RAMOS, MD (TOP 3 - FEB 2012 MED BOARDS; TOPNOTCH MD)

MIDTERM 1 EXAM - FEB 2013



MIGUEL RAFAEL RAMOS, MD (TOP 3 - FEB 2012 MED BOARDS; TOPNOTCH MD)

MIDTERM 1 EXAM - FEB 2013



MIGUEL RAFAEL RAMOS, MD (TOP 3 - FEB 2012 MED BOARDS; TOPNOTCH MD)

MIDTERM 1 EXAM - FEB 2013



MIGUEL RAFAEL RAMOS, MD (TOP 3 - FEB 2012 MED BOARDS; TOPNOTCH MD)

MIDTERM 1 EXAM - FEB 2013

TOPNOTCH MEDICAL BOARD PREP PHARMACOLOGY SUPEREXAM Page 60 of 83 For inquiries visit www.topnotchboardprep.com.ph or email us at [email protected]

TOPNOTCH MEDICAL BOARD PREP PHARMACOLOGY SUPEREXAM For inquiries visit www.topnotchboardprep.com.ph or email us at [email protected] Item #

QUESTION

EXPLANATION

AUTHOR

TOPNOTCH EXAM

C) Add oral methotrexate D) Add oral penicillamine

531

532

533

534

535

An otherwise healthy 19-year-old woman comes to the physician because of a 3-year history of intermittent facial blemishes. She drinks wine occasionally on weekends. She takes no medications. Examination shows multiple 1- to 2-mm red and white papules and on the forehead and cheeks. Which of the following is the most appropriate initial pharmacotherapy? A) Topical benzoyl peroxide B) Systemic corticosteroids C) Oral isotretinoin D) Topical corticosteroids A 57-year-old man has been hospitalized for 2 days for treatment of unstable angina pectoris. He is currently receiving intravenous heparin and undergoing evaluation for coronary artery bypass grafting. His blood pressure is 160/90 mm Hg, pulse is 88/min, and respirations are 16/min. Laboratory studies show: Platelet count 90,000/mm3 Prothrombin time 12 sec (INR=1.1) Partial thromboplastin time 35 sec Which of the following is the most likely cause of these findings? A) Excessive platelet destruction B) Factor VIII deficiency C) Inadequate platelet production D) Uncontrolled activation of coagulation and fibrinolytic cascades A 42-year-old woman, gravida 2, para 2, comes to the physician because of a 3-month history of swelling of her legs and mild abdominal pain and bloating. Abdominal examination shows no abnormalities. Rectovaginal examination shows fullness in the right adnexa. Transvaginal ultrasonography shows an irregular mass in the right ovary with some solid components to a predominantly cystic lesion. Her serum CA 125 level is 120 U/mL (N<35). Treatment with which of the following is most likely to have prevented this patient's symptoms? A) Antiestrogens B) Antiprogestationals C) Medroxyprogesterone D) Oral contraceptives A 67-year-old man is hospitalized for treatment of renal insufficiency. Three days after admission, his pulse is 40/min. An ECG shows tall, tented T waves. Serum studies show a sodium level of 134 mEq/L, potassium level of 6.9 mEq/L, and glucose level of 85 mg/dL. The most appropriate next step in management is intravenous administration of which of the following? A) Calcium, furosemide, and 3% saline B) Calcium, insulin, and digitalis C) Calcium, insulin, and glucose D) Glucose, furosemide, and phosphate A 37 year old man comes to the physician for a follow-up examination. He was diagnosed with HIV infection 3 weeks ago. He is asymptomatic and takes no edications. His temperature is 37.2 C, pulse is 100 bpm, and blood presure is 100/60 mmHg. Examination shows no abnormalities except for erythematous scaling at the hairline and nasolabial folds. His CD4+ T-lymphocyte count is 160/mm3 and plasma HIV viral load is 25,000 copies/mL. Which of the following is the most appropriate pharmacotherapy? A) Pneumocystis jiroveci prophylaxis only B) Three-drug antiretroviral therapy only C) Three-drug antiretroviral therapy and P. jiroveci

Oral isotretinoin is never the first option for acne, only used when topical treatments have failed

MIGUEL RAFAEL RAMOS, MD (TOP 3 - FEB 2012 MED BOARDS; TOPNOTCH MD)

MIDTERM 1 EXAM - FEB 2013

Heparin-induced thrombocytopenia >>> the Heparin-Platelet4 complex are attacked by IgG in the human bodies that essentially activate the platelet and thus form clots. These clots are the reason free platelets are low.

MIGUEL RAFAEL RAMOS, MD (TOP 3 - FEB 2012 MED BOARDS; TOPNOTCH MD)

MIDTERM 1 EXAM - FEB 2013



MIGUEL RAFAEL RAMOS, MD (TOP 3 - FEB 2012 MED BOARDS; TOPNOTCH MD)

MIDTERM 1 EXAM - FEB 2013

insulin and glucose >> fastest way to lower down serum K

MIGUEL RAFAEL RAMOS, MD (TOP 3 - FEB 2012 MED BOARDS; TOPNOTCH MD)

MIDTERM 1 EXAM - FEB 2013

Start HAART with CD4 below 350 and Pneumocystis prophylaxis with CD4 below 200

MIGUEL RAFAEL RAMOS, MD (TOP 3 - FEB 2012 MED BOARDS; TOPNOTCH MD)

MIDTERM 1 EXAM - FEB 2013

TOPNOTCH MEDICAL BOARD PREP PHARMACOLOGY SUPEREXAM Page 61 of 83 For inquiries visit www.topnotchboardprep.com.ph or email us at [email protected]

TOPNOTCH MEDICAL BOARD PREP PHARMACOLOGY SUPEREXAM For inquiries visit www.topnotchboardprep.com.ph or email us at [email protected] Item #

QUESTION

EXPLANATION

AUTHOR

TOPNOTCH EXAM

Phenoxybenzamine >> nonselective, IRREVERSIBLE alpha blocker..Phentolamine>> reversible alpha blocker.

MIGUEL RAFAEL RAMOS, MD (TOP 3 - FEB 2012 MED BOARDS; TOPNOTCH MD)

MIDTERM 1 EXAM - FEB 2013

Pilocarpine >> ciliary muscle contraction >> opening trabecular meshwork and increasing the outflow

MIGUEL RAFAEL RAMOS, MD (TOP 3 - FEB 2012 MED BOARDS; TOPNOTCH MD)

MIDTERM 1 EXAM - FEB 2013

tolbutamide >> disulfuram reaction

MIGUEL RAFAEL RAMOS, MD (TOP 3 - FEB 2012 MED BOARDS; TOPNOTCH MD)

MIDTERM 1 EXAM - FEB 2013



MIGUEL RAFAEL RAMOS, MD (TOP 3 - FEB 2012 MED BOARDS; TOPNOTCH MD)

MIDTERM 1 EXAM - FEB 2013



MIGUEL RAFAEL RAMOS, MD (TOP 3 - FEB 2012 MED BOARDS; TOPNOTCH MD)

MIDTERM 1 EXAM - FEB 2013

prophylaxis D) Two-drug antiretroviral therapy and P. jiroveci prophylaxis

536

537

538

539

540

A 55-year-old woman was recently diagnosed with an adrenal mass due to symptoms of sweating, elevated blood pressure, and severe headaches. Which of the following nonselective, irreversible alpha blocker is the drug of choice as a preoperative agent used in this disease? A) Phentolamine B) Phenoxybenzamine C) Prazosin D) Yohimbine A 72-year-old man was diagnosed to have glaucoma. Which of the following drugs induces ciliary muscle contraction thereby opening the trabecular meshwork and increasing the outflow? A) Timolol B) Mannitol C) Epinephrine D) Pilocarpine A 58-year-old woman with newly diagnosed type 2 diabetes mellitus presents to the emergency department complaining of vomiting, severe headache, dizziness, blurry vision, and dyspnea. She says that she had been at a party when the symptoms began. Her skin is notably flushed on physical examination. Which of the following medications is responsible for this reaction? A) Tolbutamide B) Glipizide C) Glyburide D) Metformin A 25-year-old woman with AIDS comes to the physician because of a 10-day history of fever, shortness of breath, night sweats, fatigue, and a nonproductive cough. She had been treated with antiretroviral agents for 2 years but stopped taking her medications 6 weeks ago. She has no known allergies. Her temperature is 38.6 C, pulse is 110/min, respiration are 20/min, and blood pressure is 110/60 mm Hg. Examination shows no cyanosis. Diffuse fine crackles are heard bilaterally. An x-ray of the chest shows bilateral interstitial infiltrates. A silver stain of sputum is positive for cysts and organisms. Which of the following is the most appropriate pharmacotherapy? A) Ceftriaxone B) Erythromycin C) Cotrimazole D) Co-trimoxazole A 27-year-old nulligravid woman comes to the physician for preconceptional counseling. She has a mechanical mitral heart valve and chronic rheumatoid arthritis. Her cardiac status is New York Heart Association Class II. She feels well. Current daily medications include warfarin, prednisone, and acetaminophen with codeine. Examination shows no abnormalities except for audible clicking from the heart valve. Which of the following is the most appropriate advice for this patient? A) Chemical dependency counseling before pregnancy B) Discontinuation of anticoagulant therapy during pregnancy C) Discontinuation of prednisone during pregnancy D) Switching from warfarin to heparin before pregnancy

TOPNOTCH MEDICAL BOARD PREP PHARMACOLOGY SUPEREXAM Page 62 of 83 For inquiries visit www.topnotchboardprep.com.ph or email us at [email protected]

TOPNOTCH MEDICAL BOARD PREP PHARMACOLOGY SUPEREXAM For inquiries visit www.topnotchboardprep.com.ph or email us at [email protected] Item # 541

EXPLANATION

AUTHOR

What is the half-life of alcohol? A. 2 hours B. 4 hours C. 8 hours D. 12 hours E. None of the above

Ethanol, over most of its plasma concentration range, exhibits zero-order elimination. Thus, there is no constant half life of elimination as in drugs exhibiting first order elimination. (Katzung Board Review 9e, p.6)

A 48/M farmer with a history of alcohol abuse, is rushed to the emergency room after the patient has deliberately ingested 300 mL of an unknown pesticide while drunk. On examination, you note that the patient has pinpoint pupils and frothy secretions at the mouth. After you determine that the patient has ingested malathion; you then immediately start treatment with pralidoxime. Pralidoxime and malathion are examples of: A. Competitive pharmacologic antagonists B. Irreversible pharmacologic antagonists C. Physiologic antagonists D. Structural antagonists E. Chemical antagonists A 48-year old Caucasian male is referred to you for chronic cough, night sweats and weight loss. Sputum AFB was positive on three occasions; findings of chest X ray done was also consistent with pulmonary tuberculosis. Before you start drug therapy, you remember that a much larger percentage of Caucasians are slow acetylators of one of the anti-TB drugs. The dosage of which of the following may need to be modified. A. Isoniazid B. Rifampicin C. Streptomycin D. Pyrazinamide E. Ethambutol A recently-diagnosed hypertensive was started on a BP lowering agent. Due to financial constraints, the patient abruptly stopped taking the prescribed medications after stocks were consumed. A day later, the patient was brought to the clinic for lightheadedness; on admission, patient's BP was 180/100. Which of the following anti-hypertensives, known for rebound hypertension, was most likely taken by the patient? A. Enalapril B. Furosemide C. Methyldopa D. Clonidine E. Metoprolol A 24/G3P2(2002) who came in with a BP of 200/140 was given Hydralazine. What is the mechanism of action of hydralazine? A. Reduction of Ca influx via L-type channels B. Antagonism of alpha-adrenergic receptors in blood vessels C. Release of NO D. Hyperpolarization of smooth muscles due to opening of K channels E. Activation of dopamine D1 receptors

A chemical antagonist interacts directly with the drug being antagonized to remove it or prevent it from binding to its target. Pralidoxime which combines avidly with the phosphorus in organosphosphate cholinesterase inhibitors is an example. (KBR9e, p. 14)

ABDELSIMAR OMAR II, MD (TOP 2 - AUG 2013 MED BOARDS; TOPNOTCH MD - 200 QUESTIONS) AND MARC DENVER TIONGSON, MD (40 QUESTIONS) ABDELSIMAR OMAR II, MD (TOP 2 - AUG 2013 MED BOARDS; TOPNOTCH MD - 200 QUESTIONS) AND MARC DENVER TIONGSON, MD (40 QUESTIONS)

546

Which of the following diuretics is incorrectly paired with its associated toxicity? A. Thiazide - renal stone B. Acetazolamide - hepatic encephalopathy C. Bumetanide - hypokalemic metabolic alkalosis D. Eplerenone - hyperkalemic metabolic acidosis E. Mannitol - pulmonary edema

Thiazides increase reabsorption of Ca and prevent Ca stone formation. Hepatic encephalopathy can ensue with acetazolamide because it causes alkalinization of urine due to decreased bicarbonate reabsorption. This prevents ammonia from turning to ammonium. Ammonia is more easily reabsorbed.

547

Leukotrienes are among the most potent bronchoconstrictors implicated in the pathogenesis of bronchial asthma. Which of the following is a leukotriene receptor inhibitor that has found clinical application in asthma prophylaxis? A. Zileuton B. Zafirlukast C. Ipratropium D. Budesonide E. Bosentan

Zileuton is a lipooxygenase inhibitor.

542

543

544

545

QUESTION

TOPNOTCH EXAM FINAL EXAM - FEB 2014

FINAL EXAM - FEB 2014

50% of white and African american persons are slow acetylators of INH, hydralazine and procainamide. KBR9e, p33

ABDELSIMAR OMAR II, MD (TOP 2 - AUG 2013 MED BOARDS; TOPNOTCH MD - 200 QUESTIONS) AND MARC DENVER TIONGSON, MD (40 QUESTIONS)

FINAL EXAM - FEB 2014



ABDELSIMAR OMAR II, MD (TOP 2 - AUG 2013 MED BOARDS; TOPNOTCH MD - 200 QUESTIONS) AND MARC DENVER TIONGSON, MD (40 QUESTIONS)

FINAL EXAM - FEB 2014

A describes the MOA of verapamil; D describes minoxidil; E describes fenoldopam. All are direct vasodilators.

ABDELSIMAR OMAR II, MD (TOP 2 - AUG 2013 MED BOARDS; TOPNOTCH MD - 200 QUESTIONS) AND MARC DENVER TIONGSON, MD (40 QUESTIONS) ABDELSIMAR OMAR II, MD (TOP 2 - AUG 2013 MED BOARDS; TOPNOTCH MD - 200 QUESTIONS) AND MARC DENVER TIONGSON, MD (40 QUESTIONS) ABDELSIMAR OMAR II, MD (TOP 2 - AUG 2013 MED BOARDS; TOPNOTCH MD - 200 QUESTIONS) AND MARC DENVER TIONGSON, MD (40 QUESTIONS)

FINAL EXAM - FEB 2014

TOPNOTCH MEDICAL BOARD PREP PHARMACOLOGY SUPEREXAM Page 63 of 83 For inquiries visit www.topnotchboardprep.com.ph or email us at [email protected]

FINAL EXAM - FEB 2014

FINAL EXAM - FEB 2014

TOPNOTCH MEDICAL BOARD PREP PHARMACOLOGY SUPEREXAM For inquiries visit www.topnotchboardprep.com.ph or email us at [email protected] Item # 548

QUESTION

EXPLANATION

A 28/M car mechanic was brought to the emergency room after ingesting automobile antifreeze. ABG reveals high anion gap metabolic acidosis; while urinalysis reveals calcium oxalate crystals. You suspect ethylene glycol poisoning. Which of the following drugs is recommended? A. Naltrexone B. Diazepam C. Disulfiram D. Acamprosate E. Fomepizole A patient with neuropathic pain was prescribed gabapentin. What is the primary mechanism of action of gabapentin? A. Direct activation of GABA reeptors B. Inhibition of T type Ca2+ channels C. Blockade of voltage-gated Na channels D. Inhibition of GABA transporter (GAT-1) prolonging the action of the neurotransmitter E. Enhancement of K+ channel permeability causing neuronal hyperpolarization

Ethylene glycol and methanol poisoning may be treated with fomepizole, an alcohol dehydrogenase inhibitor, which prvents conversion of methanol/ehtylene glycol to toxic metabolites.

A 6/F is brought to the clinic due to frequent episodes of sudden activity arrest with staring and minimal eyelid flutter occurring for 10 - 20 seconds, 5 - 10 times per day. During such episodes, patient is unresponsive to voice or tactile stimulation. She is diagnosed with absence seizure. Which of the following antiseizure drugs is most appropriate? A. Carbamazepine B. Valproic acid C. Phenytoin D. Phenobarbital E. Levetiracetam Administration of most inhaled anesthetics cause a decrease in the following except: A. Arterial blood pressure B. Brain metabolic rate C. Myocardial function D. Tidal volume E. None of the above

Ethosuzimide and valproic acid are the preferred drugs for absence seizures because they cause minimal sedation.

552

A 53/M, admitted at the Medical ICU, after having a myocardial infarction, develops ventricular tachycardia. Which of the following anti-arrhythmics is most appropriate? A. Quinidine B. Mexiletine C. Adenosine D. Lidocaine E. Flecainide

Lidocaine and other Class IB drugs act primarily in ischemic tissues and are best for arrhythmias following MI.

553

Flumazenil may be effective in patients who overdosed on the following sedative-hypnotics except: A. Alprazolam B. Zolpidem C. Thiopental D. Midazolam E. Lorazepam

Most S-H drugs facilitate the actions of GABA, a major inhibitory transmitter. Benzodiazepines potentiate GABA by increasing frequency of Cl ion channel opening; its action is blocked by flumazenil, a BZ receptor antagonist. Flumazenil, on the other hand, does NOT block the effects of barbiturates.

554

Which of the following IV anesthetics is the only one that causes CV stimulation? Its use in neurosurgical operations is limited due to its propensity to increase ICP. A. Midazolam B. Propofol C. Fentanyl D. Ketamine E. Thiopental



549

550

551

Although gabapentin is a structural analog of GABA, it does NOT activate GABA receptors directly.



AUTHOR ABDELSIMAR OMAR II, MD (TOP 2 - AUG 2013 MED BOARDS; TOPNOTCH MD - 200 QUESTIONS) AND MARC DENVER TIONGSON, MD (40 QUESTIONS) ABDELSIMAR OMAR II, MD (TOP 2 - AUG 2013 MED BOARDS; TOPNOTCH MD - 200 QUESTIONS) AND MARC DENVER TIONGSON, MD (40 QUESTIONS) ABDELSIMAR OMAR II, MD (TOP 2 - AUG 2013 MED BOARDS; TOPNOTCH MD - 200 QUESTIONS) AND MARC DENVER TIONGSON, MD (40 QUESTIONS) ABDELSIMAR OMAR II, MD (TOP 2 - AUG 2013 MED BOARDS; TOPNOTCH MD - 200 QUESTIONS) AND MARC DENVER TIONGSON, MD (40 QUESTIONS) ABDELSIMAR OMAR II, MD (TOP 2 - AUG 2013 MED BOARDS; TOPNOTCH MD - 200 QUESTIONS) AND MARC DENVER TIONGSON, MD (40 QUESTIONS) ABDELSIMAR OMAR II, MD (TOP 2 - AUG 2013 MED BOARDS; TOPNOTCH MD - 200 QUESTIONS) AND MARC DENVER TIONGSON, MD (40 QUESTIONS) ABDELSIMAR OMAR II, MD (TOP 2 - AUG 2013 MED BOARDS; TOPNOTCH MD - 200 QUESTIONS) AND MARC DENVER TIONGSON, MD (40 QUESTIONS)

TOPNOTCH MEDICAL BOARD PREP PHARMACOLOGY SUPEREXAM Page 64 of 83 For inquiries visit www.topnotchboardprep.com.ph or email us at [email protected]

TOPNOTCH EXAM FINAL EXAM - FEB 2014

FINAL EXAM - FEB 2014

FINAL EXAM - FEB 2014

FINAL EXAM - FEB 2014

FINAL EXAM - FEB 2014

FINAL EXAM - FEB 2014

FINAL EXAM - FEB 2014

TOPNOTCH MEDICAL BOARD PREP PHARMACOLOGY SUPEREXAM For inquiries visit www.topnotchboardprep.com.ph or email us at [email protected] Item # 555

QUESTION

EXPLANATION

AUTHOR

Sisa, 28/F, recently-diagnosed schizophrenic was admited to the Psychiatry ward and started on Risperidone. Three days later, the patient developed fever (39.9C) and rigidity. Patient was notably confused. Vital signs were as follows: BP 170/120, HR 118, RR 22. What could explain Sisa's symptoms? A. Extrapyramidal symptoms B. Tardive dyskinesia C. Serotonin syndrome D. Neuroleptic malignant syndrome E. Malignant hyperthermia Formation of methyltransferases that alter drug binding sites on the 50S ribosomal subunit is the primary mechanism of resistance to which antibiotic: A. Streptomycin B. Tetracycline C. Amikacin D. Azithromycin E. Levofloxacin

Neuroleptic malignant syndrome is characterized by FEVER, RIGIDITY, and AUTONOMIC INSTABILITY. NMS is linked to intake with several atypical antipsychotics.

A 41/M, presenting with high fever and chills, is admitted at the hospital. He reports having had serious allergic reactions to amoxicillin one year ago. Blood GS/CS reveals growth of Gram negative bacteremia. Which of the following antibiotics is MOST appropriatE? A. Cefazolin B. Ampicillin-sulbactam C. Piperacillin-tazobactam D. Aztreonam E. Vancomycin Ceftriaxone is commonly given as an empiric antibiotic in the treatment of bacterial meningitis. However, it must be noted that this antibiotic will NOT cover the following etiologic agent in meningitis: A. Neisseria meningitidis B. Streptococcus pneumoniae C. Listeria monocytogenes D. Group B streptococcus E. C and D

Aztreonam is active only against Gram-(-) organisms and has NO cross-reactivity with penicillins.

559

A 10/F, diagnosed case of HSV encephalitis, was given IV acyclovir. Acyclovir is an efffective antiviral because it inhibits: A. DNA polymerase B. RNA polymerase C. Aspartate protease D. Neuraminidase E. Hemagluttinin

Acyclovir, foscarnet and ganciclovir are DNA polymerase inhibitors.

560

A 55/M comes in due to sudden headache described as the "worst headache of his life." He is a diagnosed hypertensive, with poor compliance to medications. PE reveals nuchal tenderness (positive Kernig's sign), and a left cranial nerve palsy. Brain CT reveals diffuse subarachnoid hemorrhage in the basal cisterns. Which of the following drugs has a proven role in the management of his condition? A. Nimodipine B. Propanolol C. Captopril D. Nifedipine E. Aspirin The drug of choice for clostridium tetani infection because of the drug's excellent antimicrobial activity and the absence of the GABA-antagonistic activity a. Penicillin G b. Metrobidazole c. Vancomycin d. Chloramphenicol

Nimodipine given for 21 days should be started on admission for vasospasm prophylaxis.

ABDELSIMAR OMAR II, MD (TOP 2 - AUG 2013 MED BOARDS; TOPNOTCH MD - 200 QUESTIONS) AND MARC DENVER TIONGSON, MD (40 QUESTIONS) ABDELSIMAR OMAR II, MD (TOP 2 - AUG 2013 MED BOARDS; TOPNOTCH MD - 200 QUESTIONS) AND MARC DENVER TIONGSON, MD (40 QUESTIONS) ABDELSIMAR OMAR II, MD (TOP 2 - AUG 2013 MED BOARDS; TOPNOTCH MD - 200 QUESTIONS) AND MARC DENVER TIONGSON, MD (40 QUESTIONS) ABDELSIMAR OMAR II, MD (TOP 2 - AUG 2013 MED BOARDS; TOPNOTCH MD - 200 QUESTIONS) AND MARC DENVER TIONGSON, MD (40 QUESTIONS) ABDELSIMAR OMAR II, MD (TOP 2 - AUG 2013 MED BOARDS; TOPNOTCH MD - 200 QUESTIONS) AND MARC DENVER TIONGSON, MD (40 QUESTIONS) ABDELSIMAR OMAR II, MD (TOP 2 - AUG 2013 MED BOARDS; TOPNOTCH MD - 200 QUESTIONS) AND MARC DENVER TIONGSON, MD (40 QUESTIONS)

The use of penicillin (10–12 million units IV, given daily for 10 days) has been recommended, but metronidazole (500 mg every 6 h or 1 g every 12 h) is preferred by some experts on the basis of this drug's excellent antimicrobial activity and the absence of the GABA-antagonistic activity seen with penicillin.

BLAKE WARREN ANG, MD (TOP 1 - AUG 2013 MED BOARDS; TOPNOTCH MD)

MIDTERM 2 EXAM - FEB 2014

a second generation Cephalosporin with intrinsic activity against anaerobes: a. cefoxitin b. cefipime c. cefaclor d. ceftriaxone 0

Cefoxitin has anaerobe coverage apart from its activity against gram positive and some gram neg infections

BLAKE WARREN ANG, MD (TOP 1 - AUG 2013 MED BOARDS; TOPNOTCH MD)

MIDTERM 2 EXAM - FEB 2014

556

557

558

561

562

Question could be rephrased into: which acts on the 50s subunit? Macrolides such as azithromycin do.

Organisms NOT covered by cephalosporins are L.A.M.E.: Listeria monocytogenes, Atypicals (chlamydia, mycoplasma), MRSA, and Enterococci.

TOPNOTCH MEDICAL BOARD PREP PHARMACOLOGY SUPEREXAM Page 65 of 83 For inquiries visit www.topnotchboardprep.com.ph or email us at [email protected]

TOPNOTCH EXAM FINAL EXAM - FEB 2014

FINAL EXAM - FEB 2014

FINAL EXAM - FEB 2014

FINAL EXAM - FEB 2014

FINAL EXAM - FEB 2014

FINAL EXAM - FEB 2014

TOPNOTCH MEDICAL BOARD PREP PHARMACOLOGY SUPEREXAM For inquiries visit www.topnotchboardprep.com.ph or email us at [email protected] Item # 563

QUESTION this oral iron compound has the highest elemental iron content: a. ferrous sulfate b. ferrous gluconate c. ferrous fumarate d. iron dextran

EXPLANATION

AUTHOR

TOPNOTCH EXAM MIDTERM 2 EXAM - FEB 2014

Ferrous sulfate (FEOSOL, others) is the hydrated salt, FeSO4×7H2O, which contains 20% iron. Dried ferrous sulfate (32% elemental iron) is also available. Ferrous fumarate (FEOSTAT, others) contains 33% iron and is moderately soluble in water, stable, and almost tasteless. Ferrous gluconate (FERGON, others) also has been successfully used in the therapy of iron-deficiency anemia. The gluconate contains 12% iron. Polysaccharide-iron complex (NIFEREX, others), a compound of ferrihydrite and carbohydrate, is another preparation with comparable absorption. The effective dose of all of these preparations is based on iron content. Phenobarbitals should be avoided in patients diagbosed with porphyria

BLAKE WARREN ANG, MD (TOP 1 - AUG 2013 MED BOARDS; TOPNOTCH MD)

BLAKE WARREN ANG, MD (TOP 1 - AUG 2013 MED BOARDS; TOPNOTCH MD)

MIDTERM 2 EXAM - FEB 2014

564

which anong the following drugs should be avoided in patients with porphyria? a. phenytoin b. secobarbital c. midazolam d. sodium valproate

565

Xenobiotic metabolizing enzymes have historically been grouped into the phase 1 reactions and the phase 2 reactions. Which is NOT TRUE regarding the former? a. enzymes carry out oxidation, reduction, or hydrolytic reactions b. enzymes lead to the introduction of what are called functional groups resulting in a modification of the drug c. the reaction process adds an -OH, -COOH, -SH, -O- or NH2 group. d. the addition of functional groups significantly increase the water solubility of the drug NOT True of Phase 2 enzymes in Xenobiotic metabolism: a. they facilitate the elimination of drugs b. inactivation of electrophilic and potentially toxic metabolites produced by oxidation. c. phase 2 reactions produce a metabolite with improved water solubility d. products have decreased molecular weight majority of phase2 reactions involving Glucoronic acid conjugation are found in what cell compartment? a. ER b. cytosol c. Mitochondrial matrix d. Golgi bodies

Phase 1 reaction does little to the water solubility of a drug metabolized

BLAKE WARREN ANG, MD (TOP 1 - AUG 2013 MED BOARDS; TOPNOTCH MD)

MIDTERM 2 EXAM - FEB 2014

Because of conjugation of these substances, they have a higher molecular weight

BLAKE WARREN ANG, MD (TOP 1 - AUG 2013 MED BOARDS; TOPNOTCH MD)

MIDTERM 2 EXAM - FEB 2014

conjugating enzymes, notably the UGTs, are all located in the endoplasmic reticulum of the cell

BLAKE WARREN ANG, MD (TOP 1 - AUG 2013 MED BOARDS; TOPNOTCH MD)

MIDTERM 2 EXAM - FEB 2014

568

Notable to these drugs are its capability of inducing CP450 enzyme system in the liver except: a. Phenytoin b. Cimetidine c. Carbamazepine d. Griseofulvin

All except Cimetidine induce CYP450 enzyme complex

BLAKE WARREN ANG, MD (TOP 1 - AUG 2013 MED BOARDS; TOPNOTCH MD)

MIDTERM 2 EXAM - FEB 2014

569

Flushing is a unique side effect of Nicotinamide used in hyperlipidemia. The drug that may be used to counter this effect is: a. Acetaminophen b. Aspirin c. Chlorphenamine d. Arginine

The cutaneous effects include flushing and pruritus of the face and upper trunk, skin rashes, and acanthosis nigricans. Flushing and associated pruritus are prostaglandin-mediated. Taking an aspirin each day alleviates the flushing in many patients.

BLAKE WARREN ANG, MD (TOP 1 - AUG 2013 MED BOARDS; TOPNOTCH MD)

MIDTERM 2 EXAM - FEB 2014

570

Local anesthetics block nerve conduction through inhibition of what electrolyte? a. Potassium b. Sodium c. Calcium d. All of the above

Local anesthetics block conduction by decreasing or preventing the large transient increase in the permeability of excitable membranes to Na+ that normally is produced by a slight depolarization of the membrane

BLAKE WARREN ANG, MD (TOP 1 - AUG 2013 MED BOARDS; TOPNOTCH MD)

MIDTERM 2 EXAM - FEB 2014

571

a dreadful side effect, although not common, of this atypical antipsychotic is agranulocytosis. a. Quetiapine b. Clozapine c. Risperidone d. Aripiprazole

Clozapine is notorious for its side effect of agranulocytosis, although uncommon

BLAKE WARREN ANG, MD (TOP 1 - AUG 2013 MED BOARDS; TOPNOTCH MD)

MIDTERM 2 EXAM - FEB 2014

566

567

TOPNOTCH MEDICAL BOARD PREP PHARMACOLOGY SUPEREXAM Page 66 of 83 For inquiries visit www.topnotchboardprep.com.ph or email us at [email protected]

TOPNOTCH MEDICAL BOARD PREP PHARMACOLOGY SUPEREXAM For inquiries visit www.topnotchboardprep.com.ph or email us at [email protected] Item # 572

EXPLANATION

AUTHOR

True of Acetaminophen: a. It covalently binds to COX inhibiting prostaglandin synthesis b. very weak antiinflammatory agent at the typical daily dose of 1000 mg c. incidence of Gastrointestinal adverse effects comparable to Ibuprofen d. its inhibition of thromboxane A2 synthesis discourages its use in dengue cases A condition characterized by the acute onset of encephalopathy, liver dysfunction, and fatty infiltration of the liver and other viscera known as Reye's syndrome is associated with what drug? a. Aminosalicylate b. Long chain fatty acids c. Indomethacin d. nitric oxide

Observational studies suggest that acetaminophen, which is a , very weak antiinflammatory agent at the typical daily dose of 1000 mg is associated with a reduced incidence of gastrointestinal adverse effects compared to tNSAIDs

BLAKE WARREN ANG, MD (TOP 1 - AUG 2013 MED BOARDS; TOPNOTCH MD)

Due to the association with Reye's syndrome, aspirin and other salicylates are contraindicated in children and young adults less than 20 years old with fever associated with viral illness. Reye's syndrome is characterized by the acute onset of encephalopathy, liver dysfunction, and fatty infiltration of the liver and other viscera

BLAKE WARREN ANG, MD (TOP 1 - AUG 2013 MED BOARDS; TOPNOTCH MD)

MIDTERM 2 EXAM - FEB 2014

574

incidence of Reye syndrome among patients less than 20 who took Aspirin concurrent with influenza A/B infection: a. 0.1% b. 1% c. 5% d. 10%

Less than 0.1% of children who took aspirin developed Reye syndrome, but more than 80% of patients diagnosed with Reye syndrome had taken aspirin in the past 3 weeks

BLAKE WARREN ANG, MD (TOP 1 - AUG 2013 MED BOARDS; TOPNOTCH MD)

MIDTERM 2 EXAM - FEB 2014

575

. A good drug combination along with thiazides by virtue of its effect on potassium retention: a. losartan b. amlodipine c. verapamil d. metoprolol

BLAKE WARREN ANG, MD (TOP 1 - AUG 2013 MED BOARDS; TOPNOTCH MD)

MIDTERM 2 EXAM - FEB 2014

576

a highly lipophilic derivative of obiquinone that has activity against plasmodium species and Pneumocystis pneumonia among patients intolerant to Co-trimoxazole: a. Atorvaquone b. Quinine c. Proguanil d. artemisinin

Urinary K+ loss can be a problem with thiazides. Angiotensin converting enzyme (ACE) inhibitors and angiotensin receptor antagonists will attenuate diuretic-induced loss of potassium to some degree, and this is a consideration if a second drug is required to achieve further blood pressure reduction beyond that attained with the diuretic alone. Atorvaquone is promising synthetic derivative with potent activity against Plasmodium species and the opportunistic pathogens Pneumocystis carinii and Toxoplasma gondii. After limited clinical trials, the FDA approved this compound in 1992 for treatment of mild-to-moderate P. carinii pneumonia in patients intolerant to trimethoprim-sulfamethoxazol

BLAKE WARREN ANG, MD (TOP 1 - AUG 2013 MED BOARDS; TOPNOTCH MD)

MIDTERM 2 EXAM - FEB 2014

577

The least nephrotoxic among the aminoglycosides is: a. Tobramycin b. Kanamycin c. Amikacin d. Streptomycin

BLAKE WARREN ANG, MD (TOP 1 - AUG 2013 MED BOARDS; TOPNOTCH MD)

MIDTERM 2 EXAM - FEB 2014

578

A fusion inhibitor used in the treatment of HIV infection a. Efavirenz b. Lamivudine c. Enfuvirtide d. Atazanavir

The nephrotoxic potential varies among individual aminoglycosides. The relative toxicity correlates with the concentration of drug found in the renal cortex in experimental animals. Neomycin, which concentrates to the greatest degree, is highly nephrotoxic in human beings and should not be administered systemically. Streptomycin does not concentrate in the renal cortex and is the least nephrotoxic

BLAKE WARREN ANG, MD (TOP 1 - AUG 2013 MED BOARDS; TOPNOTCH MD)

MIDTERM 2 EXAM - FEB 2014

579

In dermatologic pharmacology, approximately how many grams of topical preparation is needed to cover the body surface? a. 10g b. 20g c. 30g d. 40g

An amount of topical medication sufficient to cover affected body surfaces in repeated applications must be dispensed to the patient. A general rule is that approximately 30 g is required to cover the body surface.

BLAKE WARREN ANG, MD (TOP 1 - AUG 2013 MED BOARDS; TOPNOTCH MD)

MIDTERM 2 EXAM - FEB 2014

580

A topical preparation that are concentric spherical shells of phospholipids in an aqueous medium that may enhance percutaneous absorption. a. Liposome b. Microgrel c. Transfersome d. Micelle

Newer vehicles include liposomes and microgel formulations. Liposomes are concentric spherical shells of phospholipids in an aqueous medium that may enhance percutaneous absorption. Variations in size, charge, and lipid content can influence liposome function substantially. Liposomes penetrate compromised epidermal barriers more efficiently (Korting et al., 1991). Microgels are polymers that may enhance solubilization of certain drugs, thereby enhancing penetration and diminishing irritancy.

BLAKE WARREN ANG, MD (TOP 1 - AUG 2013 MED BOARDS; TOPNOTCH MD)

MIDTERM 2 EXAM - FEB 2014

573

QUESTION

TOPNOTCH MEDICAL BOARD PREP PHARMACOLOGY SUPEREXAM Page 67 of 83 For inquiries visit www.topnotchboardprep.com.ph or email us at [email protected]

TOPNOTCH EXAM MIDTERM 2 EXAM - FEB 2014

TOPNOTCH MEDICAL BOARD PREP PHARMACOLOGY SUPEREXAM For inquiries visit www.topnotchboardprep.com.ph or email us at [email protected] Item # 581

582

583

584

585

586

587

588

589

QUESTION

EXPLANATION

AUTHOR

TOPNOTCH EXAM MIDTERM 1 EXAM - FEB 2014

Theophylline has a pKa of 8.7, what is the predominant form at pH 11.3? A. Ionized and protonated B. Non-ionized and protonated C. Charged and unprotonated D. Uncharged and unprotonated E. None of the above

Above pKa, the unprotonated and uncharged form of a weak base predominates.

TIMOTHY TANG LEE SAY, MD (TOP 4 - AUG 2013 MED BOARDS; TOPNOTCH MD)

Some drugs like magnesium sulfate needs a loading dose in order to reach steady state at a reduced amount of time. The following are used to compute for loading dose EXCEPT? A. Clearance B. Volume of Distribution C. Minimum effective dose D. Bioavailabiloity E. No exception A 5 year old known case of seizure disorder maintained on phenytoin was rushed to the ER due to an asthma attack. IV theophylline was given to control the asthma using usual dose computations. What are the consequences of the drug-drug interactions? A. The patient will be more prone to the toxic sideeffects of theophylline. B. The patient may be at risk for a seizure attack. C. The patient is not being treated adequately for his asthma. D. There will be an increased in plasma levels of phenytoin. E. No drug interactions exist beween theophylline and phenytoin Which of the following substances will increase the available adrenergic neurotransmitter in the synaptic cleft by inibiting reuptake? A. Botulinum B. MAOIs C. Cocaine D. Guanethidine E. Reserpine

Loadign dose = (Vd) (desired plasma concentration)/(bioavailabitlity) the minimum effective dose is usually the desired plasma concentration. Clearance is used to calculate the maintenance dose

TIMOTHY TANG LEE SAY, MD (TOP 4 - AUG 2013 MED BOARDS; TOPNOTCH MD)

MIDTERM 1 EXAM - FEB 2014

Phenytoin is an inducer, which will lower theophylline levels. Asthma may not be adequately treated with this monotherapy.

TIMOTHY TANG LEE SAY, MD (TOP 4 - AUG 2013 MED BOARDS; TOPNOTCH MD)

MIDTERM 1 EXAM - FEB 2014

Cocaine and TCA prevents reuptake of norepinephrine. Review the inhibitors for each step in the process of neurotransmitter release for both the sympathetic and parasympathetic system.

TIMOTHY TANG LEE SAY, MD (TOP 4 - AUG 2013 MED BOARDS; TOPNOTCH MD)

MIDTERM 1 EXAM - FEB 2014

Which of the following drugs for glaucoma increased aqueous humor outflow through the nonconventional pathway without decreaing the secretion of aqueous humor by the ciliary body? A. β-blockers B. Prostaglandin analogues C. α-adrenergic agonist D. Carbonic anhydrase inhibitors E. Mannitol Which of the following beta-blockers demonstrates the relative selectivity in the manner described? A. Betaxolol: relatively selective for β2-receptors B. Timolol: relatively selective for β1-receptors C. Levobunolol: relatively selective for β2-receptors D. Betaxolol: relatively selective for β1-receptors E. Levobunolol: relatively selective for β1-receptors

Beta blockers, CAIs and osmotic agents (mannitol) decreases aqueous humor production. Alpha adrenergic agonist increases outflow and also decreases the secretion of the ciliary body. Prostaglandins increased outflow but has no effect on production.

TIMOTHY TANG LEE SAY, MD (TOP 4 - AUG 2013 MED BOARDS; TOPNOTCH MD)

MIDTERM 1 EXAM - FEB 2014

Because of its relatively β1 selectivity, betaxolol has fewer side effects. Timolol and levobunolol are nonselective β-blockers

TIMOTHY TANG LEE SAY, MD (TOP 4 - AUG 2013 MED BOARDS; TOPNOTCH MD)

MIDTERM 1 EXAM - FEB 2014

What hypertensive drug commonly used for the outpatient treatment of preeclampsia is prominent for its side-effect of hemolytic anemia with a positive Coomb's test? A. α1 adrenergic agonist B. α2 adrenergic antagonist C. α1 adrenergic agonist D. α2 adrenergic antagonist E. non-selective α antagonist Which of the following antiarrhythmics primarily exerts its action on phase 4 of the cardiac action potential, prolonging PR interval? A. Procainamide B. Lidocaine C. Propanolol D. Amiodarone E. Verapamil

Methyldopa is a selective alpha 2 agonist.

TIMOTHY TANG LEE SAY, MD (TOP 4 - AUG 2013 MED BOARDS; TOPNOTCH MD)

MIDTERM 1 EXAM - FEB 2014

Class 2 anti arrhythmics or B blockers exert its effort on phase 4 and prolongs the PR interval

TIMOTHY TANG LEE SAY, MD (TOP 4 - AUG 2013 MED BOARDS; TOPNOTCH MD)

MIDTERM 1 EXAM - FEB 2014

Which of the following loop diuretics is not prone to induce an allergic response? A. Furosemide B. Bumetanide C. Brinzolamide D. Torsemide E. Ethacrynic acid

Ethacrynic acid does not contain a sulfa group.

TIMOTHY TANG LEE SAY, MD (TOP 4 - AUG 2013 MED BOARDS; TOPNOTCH MD)

MIDTERM 1 EXAM - FEB 2014

TOPNOTCH MEDICAL BOARD PREP PHARMACOLOGY SUPEREXAM Page 68 of 83 For inquiries visit www.topnotchboardprep.com.ph or email us at [email protected]

TOPNOTCH MEDICAL BOARD PREP PHARMACOLOGY SUPEREXAM For inquiries visit www.topnotchboardprep.com.ph or email us at [email protected] Item # 590

QUESTION

EXPLANATION

AUTHOR

TOPNOTCH EXAM MIDTERM 1 EXAM - FEB 2014

Which of the following is a major toxicity associalted with gemfibrozil therapy? A. Bloating and constipation B. Cholelithiasis C. Hyperuricemia D. Myopathy E. Severe cardiac arryhtmia

Gemfibrozil is associated with cholelithiasis. Myopathy is only expected in combination with statins.

TIMOTHY TANG LEE SAY, MD (TOP 4 - AUG 2013 MED BOARDS; TOPNOTCH MD)

591

Which of these anesthetics is known to cause acute porphyria? A. Nitrous oxide B. Halothane C. Ketamine D. Thiopental E. Propofol

Barbiturates such as thiopental can cause acute porphyria.

TIMOTHY TANG LEE SAY, MD (TOP 4 - AUG 2013 MED BOARDS; TOPNOTCH MD)

MIDTERM 1 EXAM - FEB 2014

592

Which of the following is the drug of choice for partial seizures? A. Valproic acid B. Phenobarbital C. Lamotrigine D. Ethosuximide E. Topiramate

Carbamazepine, phenytoin and lamotrigine are drug of choices for partial seizures

TIMOTHY TANG LEE SAY, MD (TOP 4 - AUG 2013 MED BOARDS; TOPNOTCH MD)

MIDTERM 1 EXAM - FEB 2014

593

Which drugs reduces the risk of suicide? A. Clozapine B. Clonidine C. Clonazepam D. Clofazimine E. Clofibrate

Clozapine is the anit-psychotic that reduces the risk of suicide.

TIMOTHY TANG LEE SAY, MD (TOP 4 - AUG 2013 MED BOARDS; TOPNOTCH MD)

MIDTERM 1 EXAM - FEB 2014

594

In the acute settings the following drugs are useful for the emergent management of thyroid storm EXCEPT? A. Propanolol B. KI C. Propylthiouracil D. Methimazole E. Hydrocortisone

Propylthiouracil, iodides and propanolol are the first line drugs for thyroid storm. Hydrocortisone may be added for realtive adrenal insufficiency in the acute attack. Methimazole may be used, but is not usually given in the acute therapy of thyroid storm and PTU is preferred.

TIMOTHY TANG LEE SAY, MD (TOP 4 - AUG 2013 MED BOARDS; TOPNOTCH MD)

MIDTERM 1 EXAM - FEB 2014

595

Which of the following insulin preparations mimics the secretion pattern of insulin by the normal pancreas in the fasting state? A. Lispro B. Regular C. NPH D. Glulisine E. Detemir

The long acting insulin mimics the normal secretion of the pancreas in the non-fed state.

TIMOTHY TANG LEE SAY, MD (TOP 4 - AUG 2013 MED BOARDS; TOPNOTCH MD)

MIDTERM 1 EXAM - FEB 2014

596

Which of the following antibiotics does NOT follow time-dependent killing action? A. Amoxicillin B. Aztreonam C. Metronidazole D. Bacitracin E. Vancomycin

The broad class of Beta lactams including the class of monobactams and aztreonams exhibit timedependent killing action. Bacitracin and vancomycin that also act on the cell wall exhibit time dependent killing. Metronidazole, FQ and AG are concentrationdependent.

TIMOTHY TANG LEE SAY, MD (TOP 4 - AUG 2013 MED BOARDS; TOPNOTCH MD)

MIDTERM 1 EXAM - FEB 2014

597

Which of the following anti-malarial is contraindicated for pregnant women? A. Chloroquine B. Quinine C. Quinidine D. Primaquine E. Mefloquine

The fetus is G6PD deficient, so primaquine cannot be used. To achieve radical cure for pregnant women, primaquine is given after delivery of the baby.

TIMOTHY TANG LEE SAY, MD (TOP 4 - AUG 2013 MED BOARDS; TOPNOTCH MD)

MIDTERM 1 EXAM - FEB 2014

598

Which of the following anti-neoplastic drug is highly lipophilic, use for the treatment of cancers that are not readily permeable through vascular channels? A. Bleomycin B. Busulfan C. Carmustine D. Cyclophosphamide E. Carboplatin

Carmustine and Lomustine are highly lipophilic, used to treat brain cancer.

TIMOTHY TANG LEE SAY, MD (TOP 4 - AUG 2013 MED BOARDS; TOPNOTCH MD)

MIDTERM 1 EXAM - FEB 2014

599

Which novel and innovative drug is used to treat rheumatoid arthritis through its action as an antibody against the protein CD20, found in the surface of Bcells? A. Abciximab B. Rituximab C. Omalizumab D. Infliximab E. Trastuzumab

Abciximab - Glycoprotein Iib/IIIa (anti-platelet) Omalizumab - IgE (asthma) Infliximab - TNF (inflammatory bowel disease) Trastuzumba - Her2neu (Breast cancer)

TIMOTHY TANG LEE SAY, MD (TOP 4 - AUG 2013 MED BOARDS; TOPNOTCH MD)

MIDTERM 1 EXAM - FEB 2014

TOPNOTCH MEDICAL BOARD PREP PHARMACOLOGY SUPEREXAM Page 69 of 83 For inquiries visit www.topnotchboardprep.com.ph or email us at [email protected]

TOPNOTCH MEDICAL BOARD PREP PHARMACOLOGY SUPEREXAM For inquiries visit www.topnotchboardprep.com.ph or email us at [email protected] Item # 600

QUESTION

EXPLANATION

Which of the following substances can be adsorbed by activated charcoal, effectively decontaminating the gastrointestinal tract? A. Barbiturates B. HCl C. Lithium D. Cyanide E. Lye

Remember that activated charcoal is effective only to organic substances, so if it is inorganic like ions or acids/bases, it is not effective. Alcohol, though organic does not enhance its elimination with activated charcoal.

TIMOTHY TANG LEE SAY, MD (TOP 4 - AUG 2013 MED BOARDS; TOPNOTCH MD)

Salmeterol has a slower onset of action as compared with formoterol in terms of producing bronchodilation. This is because of: A. Increased lipophilicity of salmeterol B. Faster degradation of salmeterol C. More efficient absorption of formoterol D. Higher affinity of formoterol to beta adrenergic receptors E. A and D If the plasma concentration of a drug declines with “first order kinetics,” this means that: A. The halflife is constant regardless of plasma concentration B. The drug is largely metabolized in the liver after oral administration and has low bioavailability C. The rate of elimination is proportionate to the rate of administration at all times D. The drug has a high volume of distribution E. The drug is eliminated with a constant concentration regardlesss of the plasma concentration A 35 year old female underwent treatment for endometriosis for 12 months. She then developed male-pattern hair growth, deepening of the voice and acne. The drug given is most likely: A. GnRH agonist B. Danazol C. Combined oral contraceptive pills D. Medroxyprogesterone acetate E. GnRH antagonist In the case above, which of the following patient's manifestations may be IRREVERSIBLE? A. Acne B. Male-pattern hair growth C. Deepening of the voice D. All of the above E. B and C

The quickier onset of action of formoterol can be explained by diffusion microkinetic model. This model suggests that formoterol and salmeterol are both retained in the lipid bilayer adjacent to the beta2-adrenergic receptor. Because of salmeterol's lipophilicity, it associates more with the lipid bilayer than with the receptor, accounting for its slower onset of action.

RACHELLE MENDOZA, MD (TOP 9 - AUG 2012 MED BOARDS; TOPNOTCH MD)

FINAL EXAM - FEB 2013

First Order Kinetics : elimination of the Drug is directly proportional to its plasma concerntration. its dependent on its half life, which remains constant. First order implies that no matter how much concerntration of the Drug u give it will be eliminated 50% by its first half- life

RACHELLE MENDOZA, MD (TOP 9 - AUG 2012 MED BOARDS; TOPNOTCH MD)

FINAL EXAM - FEB 2013

Danazol is a derivative of the synthetic steroid ethisterone, a modified testosterone, also known as 17-alpha-ethinyl testosterone. Danazol inhibits ovarian steroidogenesis resulting in decreased secretion of estradiol and may increase androgens. This drug causes androgenic side effects, such as hirsutism, deepening of the voice, acne and oily skin.

RACHELLE MENDOZA, MD (TOP 9 - AUG 2012 MED BOARDS; TOPNOTCH MD)

FINAL EXAM - FEB 2013

All the other side effects of danazol is reversible, except for deepening of the voice.

RACHELLE MENDOZA, MD (TOP 9 - AUG 2012 MED BOARDS; TOPNOTCH MD)

FINAL EXAM - FEB 2013

605

Which of the following anti-hypertensives has been recalled from the market due to its side effect of producing rebound suicide? A. Guanethidine B. Phenoxybenzamine C. Reserpine D. Trimetaphan

RACHELLE MENDOZA, MD (TOP 9 - AUG 2012 MED BOARDS; TOPNOTCH MD)

FINAL EXAM - FEB 2013

606

Organs involved in xenobiotic metabolism are as follows, EXCEPT: A. Liver B. Small intestines C. Lung D. Nasal mucosa E. None of the above

RACHELLE MENDOZA, MD (TOP 9 - AUG 2012 MED BOARDS; TOPNOTCH MD)

FINAL EXAM - FEB 2013

607

A pure opioid antagonist with a greater affinity for μ receptors and used for acute opioid overdose: A. Morphine B. Naloxone C. Codeine D. Dextrometorphan E. Diphenoxylate

Reserpine is an indole alkaloid antipsychotic and antihypertensive drug that has been used for the control of high blood pressure and for the relief of psychotic symptoms. Reserpine mediated depletion of monoamine neurotransmitters in the synapses is often cited as evidence to the theory that depletion of the neurotransmitters causes subsequent depression in humans Xenobiotic metabolizing enzymes are found in most tissues in the body with the highest levels located in the tissues of the gastrointestinal tract (liver, small and large intestines). Other organs that contain significant xenobiotic-metabolizing enzymes include the tissues of the nasal mucosa and lung, which play important roles in the first-pass metabolism of drugs that are administered through aerosol sprays. Naloxone is a pure opioid anatagonist and is used to counter the effects of opiate overdose, for example heroin or morphine overdose. The other drugs are opioid agonists.

RACHELLE MENDOZA, MD (TOP 9 - AUG 2012 MED BOARDS; TOPNOTCH MD)

FINAL EXAM - FEB 2013

608

Given a hypothetical species of bacteria that is resistant to antibacterial agents that prevent translocation during translation, which of the following can be used effectively for this bacterial infection? A. Chloramphenicol B. Clindamycin C. Erythromycin D. A and B E. B and C

Chloramphenicol inhibits peptidyl transferase. Clindamycin and erythromycin inhibits translocation.

RACHELLE MENDOZA, MD (TOP 9 - AUG 2012 MED BOARDS; TOPNOTCH MD)

FINAL EXAM - FEB 2013

601

602

603

604

AUTHOR

TOPNOTCH MEDICAL BOARD PREP PHARMACOLOGY SUPEREXAM Page 70 of 83 For inquiries visit www.topnotchboardprep.com.ph or email us at [email protected]

TOPNOTCH EXAM MIDTERM 1 EXAM - FEB 2014

TOPNOTCH MEDICAL BOARD PREP PHARMACOLOGY SUPEREXAM For inquiries visit www.topnotchboardprep.com.ph or email us at [email protected] Item # 609

610

611

612

613

614

615

616

617

QUESTION

EXPLANATION

AUTHOR

TOPNOTCH EXAM FINAL EXAM - FEB 2013

The drug of choice for the treatment of bladder and bowel atony is: A. Neostigmine B. Metoclopramide C. Pilocarpine D. Bethanecol E. None of the above

Bethanechol is a muscarinic agonist that is administered orally to promote GI and bladder motility, usually used in post-op ileus and diabetic neurogenic bladder.

RACHELLE MENDOZA, MD (TOP 9 - AUG 2012 MED BOARDS; TOPNOTCH MD)

Beta-lactamase inhibitors are usually given with betalactam containing antibiotics, such as penicillins. This is done to produce a favorable drug interaction, specifically: A. Synergism B. Potentiation C. Additive D. Antagonism E. None of the above A 67-year old male, on warfarin maintenance after undergoing valve replacement surgery, was brought to the ER due to unresolving epistaxis and easy bruising noted for the past week. In asking the history, the following substances are also being taken by the patient. Which of them contributed to the patient's condition? A. Cimetidine B. Digoxin C. Griseofulvin D. Procainamide Clopidogrel exerts its antithrombotic effect through which of the following mechanisms: A. Irreversible inhibition of ADP receptor B. Inhibition of thromboxane synthesis C. Reversible blockade of glycoprotein IIb/IIIa D. Conversion of plasminogen to plasmin E. Posttranslational modification of vitamin Kdependent clotting factors Sympathomimetic drug that causes mydriasis without cycloplegia: A. Atropine B. Tropicamide C. Phenyephrine D. Pilocarpine E. Timolol

Synergism is giving two active drugs and the combined effect is greater than the sum of their effects (1+1=3). Potentiation is the addition of an inactive drug to an active drug, leading to increased effect of the active drug (1+0=3).

RACHELLE MENDOZA, MD (TOP 9 - AUG 2012 MED BOARDS; TOPNOTCH MD)

FINAL EXAM - FEB 2013

Cimetidine inhibits metabolism of warfarin, thereby prolonging its effect.

RACHELLE MENDOZA, MD (TOP 9 - AUG 2012 MED BOARDS; TOPNOTCH MD)

FINAL EXAM - FEB 2013

Clopidogrel works by irreversibly inhibiting a receptor called P2Y12, an adenosine diphosphate (ADP) chemoreceptor on platelet cell membranes.

RACHELLE MENDOZA, MD (TOP 9 - AUG 2012 MED BOARDS; TOPNOTCH MD)

FINAL EXAM - FEB 2013

Atropine and tropicamide cause mydriasis and cycloplegia. Timolol has no mydriatic effect.

RACHELLE MENDOZA, MD (TOP 9 - AUG 2012 MED BOARDS; TOPNOTCH MD)

FINAL EXAM - FEB 2013

A 33-year old farmer, after 1 hour of spraying insecticides, started to note generalized body weakness, drowsiness, shortness of breath, numbness and severe epigastric pain. After 10 hours, he was brought to the ER due to loss of consciousness. He was soaking in sweat, drooling with saliva and diarrheic. What should be administered to this patient? A. Atropine B. Pralidoxime C. Neostigmine D. A and B E. B and C Lactulose is the most frequently administered laxative in post-MI patients. It is classified as: A. Bulk-forming laxative B. Osmotic laxative C. Stool softener D. Emollient E. None of the above

The patient had an insecticide poisoning. Insecticides usually contain organophosphates or carbamates. Before 6 hours, pralidoxime and atropine should be both administered. In this patient, 10 hours has already elapsed, hence pralidoxime can no longer be effective.

RACHELLE MENDOZA, MD (TOP 9 - AUG 2012 MED BOARDS; TOPNOTCH MD)

FINAL EXAM - FEB 2013

Lactulose is an osmotic agent. Bulk forming agents include insoluble fibers (psyllium). Stool softeners include docusate Na. Emollients include mineral oil.

RACHELLE MENDOZA, MD (TOP 9 - AUG 2012 MED BOARDS; TOPNOTCH MD)

FINAL EXAM - FEB 2013

A hypertensive patient was found to have elevated triglycerides, normal LDL and decreased HDL levels. Which of the following is most efficacious for this patient? A. Simvastatin B. Ezetimibe C. Colestipol D. Fenofibrate E. Orlistat Which of the following cephalosporins is highly effective against pseudomonas? A. Cefazolin B. Cefuroxime C. Ceftazidime D. Cefaclor E. Ceftriaxone

Fibrates are indicated for hypertriglyceridemia. Fibrates typically lower triglycerides by 20% to 50%. Level of the good cholesterol HDL is also increased. Fibrates may decrease LDL, though generally to a lesser degree than statins.

RACHELLE MENDOZA, MD (TOP 9 - AUG 2012 MED BOARDS; TOPNOTCH MD)

FINAL EXAM - FEB 2013

Ceftazidime, 3rd gen cephalosporin, is considered to be most efficacious agaianst pseudo monas among cephalosporins.

RACHELLE MENDOZA, MD (TOP 9 - AUG 2012 MED BOARDS; TOPNOTCH MD)

FINAL EXAM - FEB 2013

TOPNOTCH MEDICAL BOARD PREP PHARMACOLOGY SUPEREXAM Page 71 of 83 For inquiries visit www.topnotchboardprep.com.ph or email us at [email protected]

TOPNOTCH MEDICAL BOARD PREP PHARMACOLOGY SUPEREXAM For inquiries visit www.topnotchboardprep.com.ph or email us at [email protected] Item # 618

QUESTION

EXPLANATION

AUTHOR

TOPNOTCH EXAM FINAL EXAM - FEB 2013

Which of the following antivirals is a reverse transcriptase inhibitor that is used in the treatment of Hepatitis B infection? A. Amantadine B. Ganciclovir C. Lamivudine D. Interferon alpha E. Acyclovir

Lamivudine is used in hepatitis B as well as in HIV infection.

RACHELLE MENDOZA, MD (TOP 9 - AUG 2012 MED BOARDS; TOPNOTCH MD)

619

Which of the following is NOT true about insulin action? A. It stimulates glycogen synthesis in muscle fiber B. It inhibits lipolysis in the adipocyte C. It stimulates fatty acid synthesis in the hepatocytes D. It binds to G receptor to increase intracelluar cAMP

The insulin receptor is a transmembrane receptor that is activated by insulin, IGF-I, IGF-II and belongs to the large class of tyrosine kinase receptors.

RACHELLE MENDOZA, MD (TOP 9 - AUG 2012 MED BOARDS; TOPNOTCH MD)

FINAL EXAM - FEB 2013

620

Disseminated intravascular coagulation (DIC) is a condition characterized by widespread stimulation of thrombosis. Which of the following is NOT consistent with the clinical findings in DIC? A. Increased PTT B. Increased PT C. Decreased fibrinogen D. Increased fibrin split products E. None of the above A 53 year old woman with newly diagnosed type 2 diabetes presents to mergency department complaining of vomiting, severe headache, dizziness, blurry vision, and DOB. She says that she had been at a cocktail party when the symptoms began. her skin is notably flushed on physical examination. Which of the following medications is responsible for this reaction? A. Acarbose B. Glipizide C. Glyburide D. Metformin E. Tolbutamide A 65 year old man comes to the physician because he awakens to urintae several times per night and has developed problems starting and stopping his stream of urine. A biopsy of the prostate shows enlargement and dilation of the prostate glands but no dysplasia. Which of the following is the most appropriate pharmacological treatment for this patient? A. Finasteride B. Flutamide C. Ketoconazole D. Spirinolactone E. Yohimbine A 45 year old man who has received long treatment for schizophrenia has recently been noted to display involuntary movements that include lateral deviations of the jaw and "fly catching" motions in his tongue. Which of the following agents is the most likely cause of his involuntary movements? A. Clozapine B. Fluphenazine C. Lithium D. Risperidone E. selegiline Which of the following drugs can be used to reduce intraocular pressure in the treatment of glaucoma? A. Pilocarpine B. Acetazolamide C. Neostigmine D. Timolol E. All of the above

DIC clinical findings: ↓ Fibrinogen, ↑ fibrin split products, ↓ platelets, ↑ PT, ↑ PTT, fragmented red blood cells on peripheral smear

RACHELLE MENDOZA, MD (TOP 9 - AUG 2012 MED BOARDS; TOPNOTCH MD)

FINAL EXAM - FEB 2013

This is a disulfiram like reaction after drinking alcohol at a cocktail party. TOLBUTAMIDE is associated with this kind of reaction

VON ANDRE MEDINA, MD (TOP 4 - FEB 2012 MED BOARDS; TOPNOTCH MD)

DIAGNOSTIC EXAM - AUG 2012

BPH. Finasteride is a 5 alpha reductase inhibitor that inhibits conversion of testosterone to DHT.

VON ANDRE MEDINA, MD (TOP 4 - FEB 2012 MED BOARDS; TOPNOTCH MD)

DIAGNOSTIC EXAM - AUG 2012

this is TARDIVE DYSKINESIA, which is a complication of long term antipsychotic therapy with the older agents like, FLUPHENAZINE.

VON ANDRE MEDINA, MD (TOP 4 - FEB 2012 MED BOARDS; TOPNOTCH MD)

DIAGNOSTIC EXAM - AUG 2012



VON ANDRE MEDINA, MD (TOP 4 - FEB 2012 MED BOARDS; TOPNOTCH MD)

DIAGNOSTIC EXAM - AUG 2012

Dantrolene is the drug of choice to treat malignant hyperthermia caused by succinylcholine because? A. It blocks Ca release from sarcoplasmic reticulum B. It induces contraction of skeletal muscle C. It increases the rate of succinylcholine metabolism D. Succinylcholine binding to nicotinic receptors is antagonized by dantrolene E. Dantrolene acts centrally to reduce fever. In the kidney, which one of the following is inhibited by thiazide diuretics? A. Sodium chloride reabsorption in the early part of the distal convuluted tubule B. Water removal from intracellular space by osmosis C. Reabsorption of Calcium D. Aldosterone action on the nephron E. Excretion of chloride



VON ANDRE MEDINA, MD (TOP 4 - FEB 2012 MED BOARDS; TOPNOTCH MD)

DIAGNOSTIC EXAM - AUG 2012

Thiazide diuretics inhibit sodium chloride reabsorption in DCT.

VON ANDRE MEDINA, MD (TOP 4 - FEB 2012 MED BOARDS; TOPNOTCH MD)

DIAGNOSTIC EXAM - AUG 2012

621

622

623

624

625

626

TOPNOTCH MEDICAL BOARD PREP PHARMACOLOGY SUPEREXAM Page 72 of 83 For inquiries visit www.topnotchboardprep.com.ph or email us at [email protected]

TOPNOTCH MEDICAL BOARD PREP PHARMACOLOGY SUPEREXAM For inquiries visit www.topnotchboardprep.com.ph or email us at [email protected] Item # 627

QUESTION

EXPLANATION

AUTHOR

TOPNOTCH EXAM DIAGNOSTIC EXAM - AUG 2012

Which of the following agents will increase myocardial contractility with the LEAST effect on total peripheral resistance? A. Epinephrine B. Phenylephrine C. Terbutaline D. Dobutamine E. Carbachol

Dobutamine increases myocardial contractility, but it has little effect on TPR. Beta 1 selective.

VON ANDRE MEDINA, MD (TOP 4 - FEB 2012 MED BOARDS; TOPNOTCH MD)

Which of the following antihypertive agents commonly used in hypertension in pregnancy is contraindicated to patients having myocardial ischemia? A. Methyldopa B. Methergine C. Clonidine D. Hydralazine E. Magnesium sulfate Which of the following beta blocker drugs has/have intrinsic sympathomimetic activity? A. Timolol B. Acebutolol C. Pindolol D. Betaxolol E. B and C

Hydralazine is CI in ptients with myocardial ischemia.

VON ANDRE MEDINA, MD (TOP 4 - FEB 2012 MED BOARDS; TOPNOTCH MD)

DIAGNOSTIC EXAM - AUG 2012

Pindolol and Acebutolol have intrinsic sympathomimetic activity.

VON ANDRE MEDINA, MD (TOP 4 - FEB 2012 MED BOARDS; TOPNOTCH MD)

DIAGNOSTIC EXAM - AUG 2012

630

Which of the following nonselective , irreversible alpha blocker is the drug of choice as a preoperative agent used in Pheochromocytoma? A. Phentolamine B. Phenoxybenzamine C. Prazosin D. Yohimbine E. Clonidine

DOC for pheochromocytoma is Phenoxybenzamine, it is a nonselective, IRREVERSIBLE alpha blocker.. Phentolamine is a reversible alpha blocker.

VON ANDRE MEDINA, MD (TOP 4 - FEB 2012 MED BOARDS; TOPNOTCH MD)

DIAGNOSTIC EXAM - AUG 2012

631

Succor Mendiola was diagnosed to have glaucoma. Which of the following drugs induces ciliary muscle contraction thereby opening the trabecular meshwork and increasing the outflow? A. Timolol B. Mannitol C. Epinephrine D. Latanoprost E. Pilocarpine Which of the following vasodilators has the common side effect of headache, hypotension and CYANIDE toxicity? A. Fenoldopam B. Nitroprusside C. Nifedipine D. Dopamine E. Minoxidil

Pilocarpine induces ciliary muscle contraction thereby opening the trabecular meshwork and increasing the outflow

VON ANDRE MEDINA, MD (TOP 4 - FEB 2012 MED BOARDS; TOPNOTCH MD)

DIAGNOSTIC EXAM - AUG 2012

Nitroprusside is a parenteral vasodilator known to cause CYANIDE toxicity.

VON ANDRE MEDINA, MD (TOP 4 - FEB 2012 MED BOARDS; TOPNOTCH MD)

DIAGNOSTIC EXAM - AUG 2012

633

Which of the following drug/s is/are notorius in causing gingival hyperplasia? A. Nifedipine B. Cyclosporine C. Phenytoin D. Verapamil E. All of the above



VON ANDRE MEDINA, MD (TOP 4 - FEB 2012 MED BOARDS; TOPNOTCH MD)

DIAGNOSTIC EXAM - AUG 2012

634

Which of the following statements concerning barbiturates is true? A. Barbiturates can increase bleeding time when administered to patients taking anticoagulants B. Barbiturates are contraindicated in patients with acute intermittent porphyria C. Patients tolerant to the therapeutic actions of barbiturates are also tolerant to analgesic effect of morphine D. Barbiturates are used to prevent withdrawal symptoms associated with heroin dependence E. All of the above Which of the following has the lowest incidence of extrapyramidal symptoms but is known to cause agranulocytosis? A. Clozapine B. Fluphanazine C. Chlorpromazine D. Pimozide E. Haloperidol



VON ANDRE MEDINA, MD (TOP 4 - FEB 2012 MED BOARDS; TOPNOTCH MD)

DIAGNOSTIC EXAM - AUG 2012

Clozapine is a new drug with least extrapyramidal effect but is known to cause agranulocytosis.

VON ANDRE MEDINA, MD (TOP 4 - FEB 2012 MED BOARDS; TOPNOTCH MD)

DIAGNOSTIC EXAM - AUG 2012

628

629

632

635

TOPNOTCH MEDICAL BOARD PREP PHARMACOLOGY SUPEREXAM Page 73 of 83 For inquiries visit www.topnotchboardprep.com.ph or email us at [email protected]

TOPNOTCH MEDICAL BOARD PREP PHARMACOLOGY SUPEREXAM For inquiries visit www.topnotchboardprep.com.ph or email us at [email protected] Item # 636

QUESTION

EXPLANATION

AUTHOR

TOPNOTCH EXAM DIAGNOSTIC EXAM - AUG 2012

Which of the following statements about the mechanism of action of ipratropium? A. It acts cebtrally to decrease vagal Ach release B. It inhibits pulmonary Ach receptors C. It decreases mast cell release of histamine D. It blocks the action of histamine at H1 receptors E. None of the above



VON ANDRE MEDINA, MD (TOP 4 - FEB 2012 MED BOARDS; TOPNOTCH MD)

637

Which of the following anti TB drugs is known to cause hyperuricemia and is noted to be the most hepatotoxic? A. Rifampicin B. Ethambutol C. Pyrazinamide D. Streptomycin E. Isoniazid



VON ANDRE MEDINA, MD (TOP 4 - FEB 2012 MED BOARDS; TOPNOTCH MD)

DIAGNOSTIC EXAM - AUG 2012

638

A 5 year old boy with Trisomy 21 was diagnosed to Asparaginase, depletes serum asparagine. It is the have acute lymphoblastic leukemia. What is the most main and the most impt drug in the management od important drug in the therapeutic regimen of this ALL. disease entity? A. Rituximab B. Cytarabine C. Idarubucin D. Asparaginase E. Prednisone Which of the following ergot alkaloids is known to cause retroperitoneal fibrosis? A. Methylsergide B. Dihydroergotamine C. Methylergonovine D. Sumatriptan E. None of the above

VON ANDRE MEDINA, MD (TOP 4 - FEB 2012 MED BOARDS; TOPNOTCH MD)

DIAGNOSTIC EXAM - AUG 2012

VON ANDRE MEDINA, MD (TOP 4 - FEB 2012 MED BOARDS; TOPNOTCH MD)

DIAGNOSTIC EXAM - AUG 2012

640

Which of the following medication-rescue therapy pair is INCORRECT? A. Methotrexate : Leucovorin B. Cyclophosphamide : MESNA C. Doxorubicin : Dexrazoxane D. Cisplatin : Amifostine E. None of the above



VON ANDRE MEDINA, MD (TOP 4 - FEB 2012 MED BOARDS; TOPNOTCH MD)

DIAGNOSTIC EXAM - AUG 2012

641

Allen Dimo, is a 38 y/o G8P8(6208), known hypertensive and diabetic, turned out to be pregnant for 5 weeks already. What anti-hypertensive medication should you AVOID to give to the patient? A. Propranolol B. Captopril C. Hydralazine D. Methyldopa E. None of the above Assuming that the patient above is NOT pregnant. What is the best medication for her? A. Propranolol B. Captopril C. Hydralazine D. Methyldopa E. None of the above

Captopril is an ACE inhibitor and its use during gestation may result in IUGR, acute fetal renal failure, PDA, and even fetal death.

LITO JAY MACARAIG, MD (TOP 8 - FEB 2013 MED BOARDS; TOPNOTCH MD)

DIAGNOSTIC EXAM - AUG 2013

ACE inhibitors block the formation of Angiotensin II which is a potent vasoconstrictor of the afferent and efferent Renal blood vessels. When administered to the patient, it prevents diabetic nephropathy by increasing blood flow to the kidneys.

LITO JAY MACARAIG, MD (TOP 8 - FEB 2013 MED BOARDS; TOPNOTCH MD)

DIAGNOSTIC EXAM - AUG 2013

Excessive use of Nitroprusside may result in Methemoglobinemia. The antidote for this condition is? A. Methylene blue B. Silver Nitrate C. 100% oxygen D. Dialysis E. Blood transfusion This is the anti-Asthma drug which is only of worth if given as prophylaxis. A. Salbutamol B. Budesonide C. Salmeterol D. Cromolyn E. Hydrocortisone

Methylene blue

LITO JAY MACARAIG, MD (TOP 8 - FEB 2013 MED BOARDS; TOPNOTCH MD)

DIAGNOSTIC EXAM - AUG 2013



LITO JAY MACARAIG, MD (TOP 8 - FEB 2013 MED BOARDS; TOPNOTCH MD)

DIAGNOSTIC EXAM - AUG 2013

One of the following drugs causes an increase in metabolism of other drugs when administered simultaneously. A. Valproic Acid B. Rifampicin C. Venlafaxine D. Ritonavir E. Ketoconazole

Rifampicin is an inducer. All other choices are inhibitors.

LITO JAY MACARAIG, MD (TOP 8 - FEB 2013 MED BOARDS; TOPNOTCH MD)

DIAGNOSTIC EXAM - AUG 2013

639

642

643

644

645

TOPNOTCH MEDICAL BOARD PREP PHARMACOLOGY SUPEREXAM Page 74 of 83 For inquiries visit www.topnotchboardprep.com.ph or email us at [email protected]

TOPNOTCH MEDICAL BOARD PREP PHARMACOLOGY SUPEREXAM For inquiries visit www.topnotchboardprep.com.ph or email us at [email protected] Item # 646

QUESTION

EXPLANATION

Before intubation, prior to surgery, a 66 year-old male Asthmatic was injected with Succinylcholine. He was noted to remain apneic and paralyzed for an extended period of time. The patient most likely A. Received an excessive dose of the drug B. has been recieving aminoglycosides C. has impaired renal function D. had an allergic reaction E. has pseudocholinesterase deficiency What is the drug of choice for trigeminal neuralgia? A. Lamotrigine B. Gabapentin C. Pregabalin D. Carbamazepine E. B or C

Succinylcholine is being metabolized by the enzyme pseudocholinesterase, hence deficiency or lack of this enzyme will lead to extended drug effect of Succinyhlcholine.

LITO JAY MACARAIG, MD (TOP 8 - FEB 2013 MED BOARDS; TOPNOTCH MD)

SIMILAR TO PREVIOUS BOARD EXAM CONCEPT/PRINCIPLE

LITO JAY MACARAIG, MD (TOP 8 - FEB 2013 MED BOARDS; TOPNOTCH MD)

DIAGNOSTIC EXAM - AUG 2013

648

Which among the lipid-lowering medicatins causes flushing? A. Statins B. Niacin C. Cholestyramine D. A and B E. B and C



LITO JAY MACARAIG, MD (TOP 8 - FEB 2013 MED BOARDS; TOPNOTCH MD)

DIAGNOSTIC EXAM - AUG 2013

649

Which anti-parasitic drug exerts its actions on the microtubules of helminths? A. Mebendazole B. Stibogluconate C. Ivermectin D. Diloxanide Furoate E. Metronidazole

SIMILAR TO PREVIOUS BOARD EXAM CONCEPT/PRINCIPLE

LITO JAY MACARAIG, MD (TOP 8 - FEB 2013 MED BOARDS; TOPNOTCH MD)

DIAGNOSTIC EXAM - AUG 2013

650

Zidovudine is a very remarkable drug for HIV. Howeve,r precaution for its use should be observed because of this side effect. A. pancreatitis B. diarrhea C. myelosuppression D. Pulmonary fibrosis E. Optic neuritis

SIMILAR TO PREVIOUS BOARD EXAM CONCEPT/PRINCIPLE

LITO JAY MACARAIG, MD (TOP 8 - FEB 2013 MED BOARDS; TOPNOTCH MD)

DIAGNOSTIC EXAM - AUG 2013

651

A 50 year old female consulted due to palpitations. This was accompanied by frequent bowel movements and heat intolerance. You gave her Radioactive iodine and you know that its most common side effect is? A. Papillary Thyroid CA B. Medullary Thyroid CA C. Rebound hyperthyroidism D. Permanent hypothyroidism E. Rebound tachycardia

RAI may induce production of cancer cells but ALL patients administered with RAI will suffer from permanent hypothyroidism and will require exogenous thyroxine as supplement

LITO JAY MACARAIG, MD (TOP 8 - FEB 2013 MED BOARDS; TOPNOTCH MD)

DIAGNOSTIC EXAM - AUG 2013

652

Which among the following diuretics can also be used for mountain sickness? A. Acetazolamide B. Furosemide C. Thiazide D. Mannitol E. None

Carbonic anhydrase inhibitors

LITO JAY MACARAIG, MD (TOP 8 - FEB 2013 MED BOARDS; TOPNOTCH MD)

DIAGNOSTIC EXAM - AUG 2013

653

Cimetidine is used as anti-PUD drug and it acts as a/an? A. mucuprotectant B. H1 blocker C. pH neutralizer D. A and B E. None

Cimetidine is a H2 blocker

LITO JAY MACARAIG, MD (TOP 8 - FEB 2013 MED BOARDS; TOPNOTCH MD)

DIAGNOSTIC EXAM - AUG 2013

654

Which among the following is/are opioid agonist? A. Hyoscine B. Metoclopramide C. Loperamide D. Attapulgite E. AlMg(OH)

Loperamide is an opioid that acts on Kappa receptors to increase gastric emptying time.

LITO JAY MACARAIG, MD (TOP 8 - FEB 2013 MED BOARDS; TOPNOTCH MD)

DIAGNOSTIC EXAM - AUG 2013

647

AUTHOR

TOPNOTCH MEDICAL BOARD PREP PHARMACOLOGY SUPEREXAM Page 75 of 83 For inquiries visit www.topnotchboardprep.com.ph or email us at [email protected]

TOPNOTCH EXAM DIAGNOSTIC EXAM - AUG 2013

TOPNOTCH MEDICAL BOARD PREP PHARMACOLOGY SUPEREXAM For inquiries visit www.topnotchboardprep.com.ph or email us at [email protected] Item # 655

QUESTION

EXPLANATION

AUTHOR

TOPNOTCH EXAM DIAGNOSTIC EXAM - AUG 2013

After the oath-taking, you decided to establish a clinic on your province. A 63 year-old farmer was brought to you, soaked in sweat and salivating. When you asked his son about the history, you were told that he was found in that condition at the middle of the field, which is an 8-hour travel from your clinic, with a bottle of Malathion on his side. Assuming you have all the following medications in hand, what is the most appropriate to give? A. Pralidoxime B. Physostigmine C. Atropine D. Scopolamine E. Atracurium On your clinic, an 18 month old infant was brought for consult due to "blank stares" with a duration of 510 seconds. The best medication to be given for this case is A. Phenobarbital B. Valproic acid C. Ethosuximide D. Levepiracetam E. Carbamazepine You encountered a 2 year old patient suffering from an acute asthmatic attack. You immediately nebulized him with Salbutamol. Knowing its mode of action, you would expect that the patient may have A. Seizures B. Dry mouth C. Tachycardia D. Somnolence E. Excessive salivation Furosemide is a loop diuretic that is known for its ototoxicity. Which among the following drugs should you AVOID administering it simultaneously? A. Penicillin B. Ceftriaxone C. Clindamycin D. Kanamycin E. Erythromycin

The antidote for organophosphate poisoning is Pralidoxime. BUT the golden period of 6 hours had already passed. And the best medication to give in this case is ATROPINE.

LITO JAY MACARAIG, MD (TOP 8 - FEB 2013 MED BOARDS; TOPNOTCH MD)

This is absence seizure and DOC is Ethosuximide

LITO JAY MACARAIG, MD (TOP 8 - FEB 2013 MED BOARDS; TOPNOTCH MD)

DIAGNOSTIC EXAM - AUG 2013

Salbutamol is a short-acting beta agonist and may stimulate beta 1 receptors of the heart, leading to transient tachycardia.

LITO JAY MACARAIG, MD (TOP 8 - FEB 2013 MED BOARDS; TOPNOTCH MD)

DIAGNOSTIC EXAM - AUG 2013

Kanamycin is an aminoglycoside is very ototoxic

LITO JAY MACARAIG, MD (TOP 8 - FEB 2013 MED BOARDS; TOPNOTCH MD)

DIAGNOSTIC EXAM - AUG 2013

659

Which among the following drugs is commonly used in controlling the manic phase of a bipolar patient? A. Lithium B. Selegiline C. Reserpine D. Haloperidol E. Biperiden



LITO JAY MACARAIG, MD (TOP 8 - FEB 2013 MED BOARDS; TOPNOTCH MD)

DIAGNOSTIC EXAM - AUG 2013

660

Being an IM resident, you are assigned to watch over a patient who is undergoing chemotherapy. Which drug is best to be given to prevent post-chemo vomiting? A. Metoclopramide B. Hyoscine N-Butyl-bromide C. Ondansteron D. Methylergonovine E. Methyrgine This is time of drug solubility and absorption and clearance? A. Ionized drugs are better absorbed in the GIT B. Polar drugs have decreased clearance C. Non-ionized drugs are better absorbed in the GIT D. Non-polar drugs have increased clearance E. None of the above

Ondansetron is a 5TH-3 anatagonist and is the drug of choice for post-chemo vomiting.

LITO JAY MACARAIG, MD (TOP 8 - FEB 2013 MED BOARDS; TOPNOTCH MD)

DIAGNOSTIC EXAM - AUG 2013

polar, ionized - less soluble in lipids, excreted; non - polar, unionized - more soluble, absorbed

HAZEL KAREN RAZ, MD (TOP 6 - FEB 2013 MED BOARDS; TOPNOTCH MD)

MIDTERM 2 - AUG 2013

662

Route of administration with instantaneous absorption, bypasses first pass effect with I00% bioavailabilty? A. IV B. SL C. Oral D. Rectal E. None of the above

Compared with other routes of administration, the intravenous route is the fastest way to deliver fluids and medications throughout the body.

HAZEL KAREN RAZ, MD (TOP 6 - FEB 2013 MED BOARDS; TOPNOTCH MD)

MIDTERM 2 - AUG 2013

663

Drugs which undergo zero order kinetics, except? A. Warfarin B. Heparin C. Hydralazine D. Aspirin E. None of the above

WHATPET - Warfarin, Heparin, ASA, Tolbutamide, Phenytoin, Ethanol, Theophylline

HAZEL KAREN RAZ, MD (TOP 6 - FEB 2013 MED BOARDS; TOPNOTCH MD)

MIDTERM 2 - AUG 2013

656

657

658

661

TOPNOTCH MEDICAL BOARD PREP PHARMACOLOGY SUPEREXAM Page 76 of 83 For inquiries visit www.topnotchboardprep.com.ph or email us at [email protected]

TOPNOTCH MEDICAL BOARD PREP PHARMACOLOGY SUPEREXAM For inquiries visit www.topnotchboardprep.com.ph or email us at [email protected] Item # 664

QUESTION

EXPLANATION

AUTHOR

TOPNOTCH EXAM MIDTERM 2 - AUG 2013

In this phase of clinical trials, the drig is evaluated in a moderate number of target patients to test dose efficacy of the drug? A. I B. II C. III D. IV E. None of the above

I - normal volunteers, pharmacokinteics and dose sensitive acute effects II - moderate number of PATIENTS, dose efficacy III - large # of patients, toxicities IV - post - marketing, idiosyncratic reactions

HAZEL KAREN RAZ, MD (TOP 6 - FEB 2013 MED BOARDS; TOPNOTCH MD)

665

Which of the following drugs is a direct acting cholinoninetic used for treatment of glaucoma, sjoqren and sicca syndrome? A. Betanechol B. Pilocarpine C. Physostigmine D. Neostigmine E. None of the above

Bethanecol for bowel and bladder atony

HAZEL KAREN RAZ, MD (TOP 6 - FEB 2013 MED BOARDS; TOPNOTCH MD)

MIDTERM 2 - AUG 2013

666

Muscarinic antagonists used for the treatment of parkinsonism, except? A. Trinexyphenidyl B. Triamterene C. Benztropine D. Biperiden E. None of the above

Triamterene - potassium sparing diuretic

HAZEL KAREN RAZ, MD (TOP 6 - FEB 2013 MED BOARDS; TOPNOTCH MD)

MIDTERM 2 - AUG 2013

667

Drug of choice for uncomplicated essential hypertension? A. Furosemide B. Metoprolol C. HCTZ D. Captopril E. None of the above

Thiazide diuretics are first line of treatment for uncomplicated hypertension.

HAZEL KAREN RAZ, MD (TOP 6 - FEB 2013 MED BOARDS; TOPNOTCH MD)

MIDTERM 2 - AUG 2013

668

B - Blocker with intrinsic sympathomimetic activity? A. Acebutolol B. Atenolol C. esmolol D. metroprolol E. None of the above

Acebutolol and Pindolo has ISA.

HAZEL KAREN RAZ, MD (TOP 6 - FEB 2013 MED BOARDS; TOPNOTCH MD)

MIDTERM 2 - AUG 2013

669

What is the MOST-SLECTIVE alpha 1 selective antagonist used in the treatment of BPH? A. Prazosin B. tamsulosin C. Terazosin D. Doxazosin E. all of the above

MIDTERM 2 - AUG 2013

670

Mechanism of action of cyanide poisoning? A. Inhibits complex III of ETC B. Uncoupler C. Inhibits oxidative bursts D. Inhibits complex IV of ETC E. None of the above

HAZEL KAREN However, all of the mentioned drugs are alpha-1 RAZ, MD (TOP 6 - selective antagonists used in the treatment of BPH FEB 2013 MED (Prazosin, Tamsulosin, Terazosin, Doxazosin). BOARDS; Tamsulosin is the most selective alpha-1 selective TOPNOTCH MD) antagonist. . Tamsulosin is a selective α1 receptor antagonist that has preferential selectivity for the α1A receptor in the prostate versus the α1B receptor in the blood vessels. When alpha 1 receptors in the bladder neck and the prostate are blocked, this causes a relaxation in smooth muscle and therefore less resistance to urinary flow. Due to this the pain associated with BPH can be reduced. Complex I - amytal, rotenone; Complex II - malonate; HAZEL KAREN Complex III - antimycin A, Dimercaprol; Complex IV - RAZ, MD (TOP 6 - CN, CO, Na azide; Complex V - Oligomycin FEB 2013 MED Uncoupler - ASA, 2,4 - DNP BOARDS; TOPNOTCH MD)

671

An anticoagulant which has a more selective effect on factor Xa? A. hepanin B. Lepirodin C. enoxaparin D. warfarin E. None of the above

It is an anticoagulant used to prevent and treat deep vein thrombosis or pulmonary embolism, and is given as a subcutaneous injection (by a health care provider or the patient).

HAZEL KAREN RAZ, MD (TOP 6 - FEB 2013 MED BOARDS; TOPNOTCH MD)

MIDTERM 2 - AUG 2013

672

Antihyperlipidemic agent which is contraindicated in patients prone to GB stone formation? A. Genifibrozil B. Niacin C. Ezetimibe D. Simvastatin E. None of the above

Fibrates and cholestipol increases risk of forming gallstones.

HAZEL KAREN RAZ, MD (TOP 6 - FEB 2013 MED BOARDS; TOPNOTCH MD)

MIDTERM 2 - AUG 2013

TOPNOTCH MEDICAL BOARD PREP PHARMACOLOGY SUPEREXAM Page 77 of 83 For inquiries visit www.topnotchboardprep.com.ph or email us at [email protected]

MIDTERM 2 - AUG 2013

TOPNOTCH MEDICAL BOARD PREP PHARMACOLOGY SUPEREXAM For inquiries visit www.topnotchboardprep.com.ph or email us at [email protected] Item # 673

QUESTION

EXPLANATION

AUTHOR

TOPNOTCH EXAM MIDTERM 2 - AUG 2013

Mechanism of action of chloroquine except:? A. Suppression of B-Lymphocyte response to mitogens B. Free-Radical Trapping C. Lysosomal enzyme stabilization D. Decreased leukocyte chemotaxis E. None of the above

Chloroquine suppress T - lymphocyte response

HAZEL KAREN RAZ, MD (TOP 6 - FEB 2013 MED BOARDS; TOPNOTCH MD)

674

Mechanism of action of diazepam? A. Increases duration of Cl- channel opening B. Increases frequency of Cl- channel opening C. Agonist at 5HT1a and D2 receptors D. Blocks voltage gated Na+ channels E. None of the above

Increases frequency of channel opening

HAZEL KAREN RAZ, MD (TOP 6 - FEB 2013 MED BOARDS; TOPNOTCH MD)

MIDTERM 2 - AUG 2013

675

Drug of choice for partial seizures? A. Diazepam B. Phenobarbital C. Valproic Acid D. Carbamazepine E. None of the above

Diazepam - s. epilepticus Phenobarbital - s. epilepticus Valproic Acid - absence, myoclonic Carbamazepine - partial seizures

HAZEL KAREN RAZ, MD (TOP 6 - FEB 2013 MED BOARDS; TOPNOTCH MD)

MIDTERM 2 - AUG 2013

676

Causes analgesia, amnesia and catatonia with retained conciousness? A. Midazolam B. Ketamine C. Thiopental D. Fentanyl E. None of the above

Dissociative anesthesia

HAZEL KAREN RAZ, MD (TOP 6 - FEB 2013 MED BOARDS; TOPNOTCH MD)

MIDTERM 2 - AUG 2013

677

Early morning hyperglycemia characterized by development of hypoglycemia around 3AM and prebreakfast hyperglycemia:? A. Waning of insulin dose B. Dawn Phenomenon C. Somogyi Effect D. Combined Waning + Dawn E. None of the above

Waning of insulin dose - low dose insulin Dawn Phenomenon - wrong type of insulin Somogyi Effect - high dose of insulin

HAZEL KAREN RAZ, MD (TOP 6 - FEB 2013 MED BOARDS; TOPNOTCH MD)

MIDTERM 2 - AUG 2013

678

Antibiotic which inhibits transpeptidation by binding to 50s subunit, used in the treatment of meningitis secondary to strep, H.inf, Neisseria neningitidis, causes aplastic anemia as its most common idiosyncratic effect? A. Chloramphenicol B. Erythromycin C. Linezolid D. Clindamycin E. None of the above

Chloramphenicol is a bacteriostatic drug that stops bacterial growth by inhibiting protein synthesis. Chloramphenicol prevents protein chain elongation by inhibiting the peptidyl transferase activity of the bacterial ribosome.

HAZEL KAREN RAZ, MD (TOP 6 - FEB 2013 MED BOARDS; TOPNOTCH MD)

MIDTERM 2 - AUG 2013

679

Mechanism of action of mebendazole? A. Inhibits microtubule assembly B. Inhibits glucose uptake in rematudes C. Causes dipolarization induced paralysis D. Increase permeability of membrane to Ca2+ causing muscle paralysis E. None of the above

HAZEL KAREN RAZ, MD (TOP 6 - FEB 2013 MED BOARDS; TOPNOTCH MD)

MIDTERM 2 - AUG 2013

680

Drug of choice for prostatic carcinoma? A. gemcitabine B. Paditaxel C. Cisplatin D. Leuprolide E. None of the above

HAZEL KAREN RAZ, MD (TOP 6 - FEB 2013 MED BOARDS; TOPNOTCH MD)

MIDTERM 2 - AUG 2013

681

Doxylamine is an ethanolamine H1 antagonist drug previously utilized as a possible treatment of nausea and vomiting of pregnancy. Now, this drug is still available over-the-counter as sleep aid. What Bvitamin is a component of Doxylamine? A. Vitamin B2 B. Vitamin B3 C. Vitamin B6 D. Vitamin B12

Mebendazole causes degenerative alterations in the tegument and intestinal cells of the worm by binding to the colchicine-sensitive site of tubulin, thus inhibiting its polymerization or assembly into microtubules. The loss of the cytoplasmic microtubules leads to impaired uptake of glucose by the larval and adult stages of the susceptible parasites, and depletes their glycogen stores. Leuprolide acts as an agonist at pituitary GnRH receptors. By interrupting the normal pulsatile stimulation of, and thus desensitizing, the GnRH receptors, it indirectly down regulates the secretion of gonadotropins luteinizing hormone (LH) and follicle-stimulating hormone (FSH), leading to hypogonadism and thus a dramatic reduction in estradiol and testosterone levels in both sexes. Answer: C. Vitamin B6 (pp. 389 [.pdf file], Katzung Pharmacology, 9th edition) Notes: Several H1 antagonist drugs have been studied for possible use in treating "morning sickness." The piperazine derivatives were withdrawn from such use when it was demonstrated that they have teratogenic effects in rodents. Doxylamine, an ethanolamine H1 antagonist, was promoted for this application as a component of Bendectin, a prescription medication that also contained pyridoxine. * SIMILAR TO PREVIOUS BOARD EXAM CONCEPT/PRINCIPLE

MICHELLE JAY FRANCISCO, MD (TOP 9 - FEB 2013 MED BOARDS; TOPNOTCH MD)

MIDTERM 1 - AUG 2013

TOPNOTCH MEDICAL BOARD PREP PHARMACOLOGY SUPEREXAM Page 78 of 83 For inquiries visit www.topnotchboardprep.com.ph or email us at [email protected]

TOPNOTCH MEDICAL BOARD PREP PHARMACOLOGY SUPEREXAM For inquiries visit www.topnotchboardprep.com.ph or email us at [email protected] Item # 682

683

684

685

686

687

QUESTION

EXPLANATION

AUTHOR

A few substances that are almost completely inert in the chemical sense nevertheless have significant pharmacologic effects. An example of these substances is Simethicone, which is included in many antacid preparations in order to act as A. Antispasmodic B. Antiflatulent C. Antiemetic D. Antiminth 3. Future clinicians may screen every patient for a variety of individual differences in drug response. Differences in the rate of acetylation of isoniazid among groups of individuals exemplifies A. Pharmacodynamics B. Posology C. Pharmacogenetics D. Biochemorphology A college friend consults you regarding the suitability of the medication given by his doctor for severe hypertension. He complains of postural and exercise hypotension (“dizziness”), some diarrhea, and problems with ejaculation during sexual activity. Given those adverse effects stated, which of the following mechanisms of action is most likely related to the drug described by your friend? A. A drug which inhibits the release of norepinephrine from sympathetic nerve endings. B. A drug which decreases blood pressure as a result of a decrease in cardiac output due to its nonselective β-blockade. C. An oral vasodilator which relaxes smooth muscles of arterioles, thereby decreasing the systemic vascular resistance. D. A drug which inhibits the converting enzyme peptidyl dipeptidase that hydrolyzes angiotensin I to angiotensin II and also inactivates bradykinin. The Board of Medicine decided to change the program of examination for the August 2013 Physician Licensure Examination (PLE) changing the first subject to Pharmacology. Because of this, a nonprepared Non-Topnotch examinee suddenly got an attack of diarrhea while on his way to the testing site. If he will stop at a nearby drugstore to buy an overthe-counter opioid with an antidiarrheal action, he will be asking for A. Codeine B. Dextromethorphan C. Loperamide D. Diphenoxylate Sources of drugs can either be natural or synthetic. These might be derived from plant sources, animal or marine products, minerals, microorganisms and others from laboratory sources. Many antibiotics were derived from Genus Streptomyces, Penicillium, Micromonospora, etc. Which of the following drugs is a polyene antibiotic derived from Streptomyces nodosus? A. Netilmicin B. Amikacin C. Streptomycin D. Amphotericin B

Answer: B. Antiflatulent (pp. 1520 [.pdf file], Katzung Pharmacology, 9th edition) Notes: Some antacid preparations include simethicone, an antiflatulent to relieve symptoms of bloating and pressure. *SIMILAR TO PREVIOUS BOARD EXAM CONCEPT/PRINCIPLE

MICHELLE JAY FRANCISCO, MD (TOP 9 - FEB 2013 MED BOARDS; TOPNOTCH MD)

Answer: C. Pharmacogenetics (pp. 3 and 82 [.pdf file] Katzung Pharmacology, 9th edition)

MICHELLE JAY FRANCISCO, MD (TOP 9 - FEB 2013 MED BOARDS; TOPNOTCH MD)

MIDTERM 1 - AUG 2013

Answer: A. A drug which inhibits the release of norepinephrine from sympathetic nerve endings. The adverse effects described pertains to Guanethidine (pp. 237, 241, 244 and 251 [.pdf file], Katzung Pharmacology, 9th edition). Notes: • A drug which decreases blood pressure as a result of a decrease in cardiac output due to its nonselective β-blockade – Propranolol • An oral vasodilator which relaxes smooth muscles of arterioles, thereby decreasing the systemic vascular resistance – Hydralazine • A drug which inhibits the converting enzyme peptidyl dipeptidase that hydrolyzes angiotensin I to angiotensin II and also inactivates bradykinin – Captopril

MICHELLE JAY FRANCISCO, MD (TOP 9 - FEB 2013 MED BOARDS; TOPNOTCH MD)

MIDTERM 1 - AUG 2013

Answer: C. Loperamide (pp. 713 [.pdf file], Katzung Pharmacology, 9th edition) – must know, MPL 1.0

MICHELLE JAY FRANCISCO, MD (TOP 9 - FEB 2013 MED BOARDS; TOPNOTCH MD)

MIDTERM 1 - AUG 2013

Answer: D. Amphotericin B (pp. 1104 [.pdf file], Katzung Pharmacology, 9th edition.) Notes: incomplete list Streptomycin à Strept. griseus Neomycin à Strept. fradiae Paromomycin à Strept. rimosus Kanamycin (der. Amikacin) à Strept.kanamyceticus Tobramycin à Strept. tenebrarius Ivermectin à Strept. avermitilis Mitomycin à Strept. caespitosus Erythromycin à Strept. erythreus Lincomycin (der. Clindamycin) à Strept. lincolnensis Rifamycin (der. Rifampicin) à Strept. mediterranei Gentamicin à Micromonospora purpurea Sisomicin (der. Netilmicin) à Micromonospora inyoensis

MICHELLE JAY FRANCISCO, MD (TOP 9 - FEB 2013 MED BOARDS; TOPNOTCH MD)

MIDTERM 1 - AUG 2013

Mrs. T. Anga, 25 yo, G1P1 was misdiagnosed to have preeclampsia because of the inaccuracy of her history and was given an anti-hypertensive to control her blood pressure. Fortunately, patient had no postpartum complications and BP was controlled. She continued taking the previously given antihypertensive but was lost to follow-up. After maintaining the drug for more than 12 months, patient developed intermittent episodes of headache, myalgias, jaundice and passage of dark brown urine. She finally went back to her physician for consult and laboratory testing revealed a positive Antiglobulin Test. If the condition was drug-induced, what is the most likely offending agent? A. Methyldopa B. Hydralazine C. Clonidine D. Magnesium Sulfate

Answer: A. Methyldopa (pp. 235 [.pdf file], Katzung Pharmacology, 9th edition). Notes: The symptoms experienced by the patient are consistent with Autoimmune Hemolytic Anemia which is a rare adverse effect of methyldopa. Methyldopa is a drug which lowers blood pressure chiefly by reducing the peripheral vascular resistance and variably reduces heart rate and cardiac output. Ten to twenty percent of patients undergoing therapy for longer than 12 months from methyldopa will have positive Coomb’s Test (or Antiglobulin Test).

MICHELLE JAY FRANCISCO, MD (TOP 9 - FEB 2013 MED BOARDS; TOPNOTCH MD)

MIDTERM 1 - AUG 2013

TOPNOTCH MEDICAL BOARD PREP PHARMACOLOGY SUPEREXAM Page 79 of 83 For inquiries visit www.topnotchboardprep.com.ph or email us at [email protected]

TOPNOTCH EXAM MIDTERM 1 - AUG 2013

TOPNOTCH MEDICAL BOARD PREP PHARMACOLOGY SUPEREXAM For inquiries visit www.topnotchboardprep.com.ph or email us at [email protected] Item # 688

QUESTION

EXPLANATION

AUTHOR

TOPNOTCH EXAM MIDTERM 1 - AUG 2013

Baby B. Ulate, suffers from infestation of the “unholy trinity” of roundworms, namely Ascaris lumbricoides, Trichuris trichiura and Ancylostoma duodenale. What is the mechanism of action of the drug suitable for the patient’s condition? A. Increases permeability to calcium causing muscle paralysis, vacuolization and death. B. Selectively inhibits microtubule synthesis and glucose uptake in nematodes. C. Immobilizes the parasite and alters their surface structure displacing them from the tissues and making them more susceptible to destruction by host defense mechanisms. D. Intensifies GABA-mediated neurotransmission in nematodes immobilizing the parasite.

Answer: B. Selectively inhibits microtubule synthesis and glucose uptake in nematodes is the mechanism of action of Mebendazole, the drug of choice for Ascariasis (pp. 40 of part2, Pharmacology, Topnotch Handouts). Notes: • Increases permeability to calcium causing muscle paralysis, vacuolization and death – Praziquantel (DOC for Trematodes and Cestodes) • Immobilizes the parasite and alters their surface structure displacing them and making them more susceptible to destruction by host defense mechanisms – Diethylcarbamazine (DOC for Filariasis) • Intensifies GABA-mediated neurotransmission in nematodes immobilizing the parasite – Ivermectin (DOC for Strongyloidiasis) • “Unholy trinity” of roundworms – Ascaris, Whipworm and Hookworms Answer: A. Vinblastine (pp 41 [part 1], Pharmacology, Topnotch Handouts). Notes: A – Adriamycin (Doxorubicin) B – Bleomycin V – Vincristine (Oncovin) D – Dacarbazine

MICHELLE JAY FRANCISCO, MD (TOP 9 - FEB 2013 MED BOARDS; TOPNOTCH MD)

MICHELLE JAY FRANCISCO, MD (TOP 9 - FEB 2013 MED BOARDS; TOPNOTCH MD)

MIDTERM 1 - AUG 2013

689

The following drugs are included in the ABVD regimen for Hodgkin’s Lymphoma, except: A. Vinblastine B. Doxorubicin C. Dacarbazine D. Oncovin E. Bleomycin

690

The following drugs accelerate substrate metabolism causing a decrease in the pharmacologic action of the co-administered drug, except; A. Griseofulvin B. Rifampicin C. Ethanol D. Cimetidine

Answer: D. Cimetidine (pp. 8 [part 1], Pharmacology, Topnotch Handouts; pp. 77 [.pdf file], Katzung Pharmacology, 9th edition). The question defines a CYP450 Inducer. The only CYP450 inhibitor among the choices is Cimetidine.

MICHELLE JAY FRANCISCO, MD (TOP 9 - FEB 2013 MED BOARDS; TOPNOTCH MD)

MIDTERM 1 - AUG 2013

691

Mr. A. Lipin, a farmer from a distant hacienda was brought to the ED and admitted due to unconsciousness, profuse salivation and shallow breathing. It took 7 hours of travel time from the hacienda to the nearest Provincial Hospital. BP 140/90 mmHg, PR 52 bpm, temp 30OC and with constricted pupils. As an ED physician, what medication will you initially give that will most likely benefit the patient? A. Pralidoxime B. Atropine C. Pralidoxime + Atropine D. Physostigmine A patient in PACU is in no apparent distress. The V/S are stable except for HR of 128 bpm that is irregular with no p wave. Which of the following treatment option would not be appropriate as initial therapy? A. Metoprolol B. Diltiazem C. Digoxin D. Adenosine

Answer: B. Atropine (pp. 141, 149 and 162, Katzung Pharmacology, 9th edition) Notes: Atropine is the prototype antimuscarinic drug to be given initially in this patient suffering from organophosphate poisoning. It is best to give both Atropine and Pralidoxime if the golden period of 6 hours was not violated. The likelihood of aging of the phosphate enzyme complex occurs after 6 hours; however, recent reports suggest that administration of multiple doses of pralidoxime over several days may be useful in severe poisoning.

MICHELLE JAY FRANCISCO, MD (TOP 9 - FEB 2013 MED BOARDS; TOPNOTCH MD)

MIDTERM 1 - AUG 2013

Answer: D Patient has atrial fibrillation w/ rapid ventricular response. B-blocker, Ca Channel blocker or Digoxin may be used. Supraventricular tachycardia, not a.fib responds to adenosine.

MICHELLE JAY FRANCISCO, MD (TOP 9 - FEB 2013 MED BOARDS; TOPNOTCH MD)

MIDTERM 1 - AUG 2013

693

This factor indicates the spread of local anesthetic in CSF? A. Addition of narcotic to local anesthetic. B. Density of local anesthetic C. Patient’s body surface area D. Dose administered

Answer: B Density of local anesthetic in relation to density of CSF at normal body temp.is termed as baricity. Degree of spread is determnined primarily by density of anesthetic & patient’s position.

MICHELLE JAY FRANCISCO, MD (TOP 9 - FEB 2013 MED BOARDS; TOPNOTCH MD)

MIDTERM 1 - AUG 2013

694

A young, male patient presents with mental deterioration and tremors. He has brown pigmentation in a ring surrounding the periphery of his cornea and altered liver function tests. Which treatment modality should he receive? A. Dimercaprol B. Penicillamine C. EDTA D. Deferoxamine In the treatment of CHF, this drug is characterized as having a selective beta-1 antagonistic activity and devoid of dopaminergic activity: A. Fenoldopam B. Dobutamine C. Dopamine D. Isoproterenol

Answer: B Wilson’s disease: excess copper deposition (cornea = Kayser-Fleischer ring)

MICHELLE JAY FRANCISCO, MD (TOP 9 - FEB 2013 MED BOARDS; TOPNOTCH MD)

MIDTERM 1 - AUG 2013

Answer: B Dobutamine is a relatively b-1 selective adrenoreceptor agonist that causes peripheral vasodilation in some vascular beds useful for many cases of acute heart failure.

MICHELLE JAY FRANCISCO, MD (TOP 9 - FEB 2013 MED BOARDS; TOPNOTCH MD)

MIDTERM 1 - AUG 2013

692

695

TOPNOTCH MEDICAL BOARD PREP PHARMACOLOGY SUPEREXAM Page 80 of 83 For inquiries visit www.topnotchboardprep.com.ph or email us at [email protected]

TOPNOTCH MEDICAL BOARD PREP PHARMACOLOGY SUPEREXAM For inquiries visit www.topnotchboardprep.com.ph or email us at [email protected] Item # 696

697

698

699

700

QUESTION

EXPLANATION

AUTHOR

A 7 year-old male child with nephritic syndrome on steroid therapy was exposed to a sister with measles. You would administer: A. Measles vaccine B. Human immunoglobulin C. Antisera D. Vitamin A

Answer: B After susceptible person is exposed to measles, either measles vaccine given within 72 hrs or immune globulin given within 6 days can prevent or modify disease. However, px has nephritic syndrome & is immunocompromised so an active measles vaccine may even be harmful.

MICHELLE JAY FRANCISCO, MD (TOP 9 - FEB 2013 MED BOARDS; TOPNOTCH MD)

At the E.R., you saw a 24-yr old woman complaining of severe, acute, generalized headache. PE revealed T 400C with (+) nuchal rigidity. Her CSF showed gramneg diplococci & 200 leukocytes/mm3 (98% PMNs). Administration of which of these is the next important management: A. IV sulphonamides B. Intrathecal penicillin C. IV penicillin D. Intratechal chloramphenicol An asthmatic patient whose symptoms occur less than once a week, lasts a few hours to a few days and has nighttime symptoms twice a month is best managed with an: A. Inhaled long-acting B2 agonist at H.S. B. Inlahed short-acting B2 agonist PRN C. Inhaled steroid OD D. Oral B2 agonist TID The absorption of most orally administered drug, which occur in the small intestine, may be enhanced by A. Taking agents that accelerate gastric emptying time B. The presence of food C. Stimulation of hepatic microsomal CYP3A4 enzymes D. Increasing intestinal motility Adverse events associated with the use of quinolones include all of the following except: A. Tendinitis and possible tendon rupture B. Seizures C. Arthropathy in children D. Clostridium difficile colitis E. Narrowing of QT interval

Answer: C Lab picture: N.meningitides. Tx = Pen G 4 million units IV q4h

MICHELLE JAY FRANCISCO, MD (TOP 9 - FEB 2013 MED BOARDS; TOPNOTCH MD)

MIDTERM 1 - AUG 2013

Answer: B Px’s asthma is classified as intermittent. *Review GINA guidelines

MICHELLE JAY FRANCISCO, MD (TOP 9 - FEB 2013 MED BOARDS; TOPNOTCH MD)

MIDTERM 1 - AUG 2013

Answer: A Increasing gastric emptying time allow drugs to reach the small intestine sooner, hence increasing the rate of drug absorption.

MICHELLE JAY FRANCISCO, MD (TOP 9 - FEB 2013 MED BOARDS; TOPNOTCH MD)

MIDTERM 1 - AUG 2013

Answer: E Quinolones inhibit topoisomerase II (DNA gyrase) & IV impairing DNA synthesis. Adverse effects include GI intolerance, antibiotic-asso.colitis, cutaneous rxns, hepatotoxicity, prolongation of QT interval and Achilles tendon rupture.

MICHELLE JAY FRANCISCO, MD (TOP 9 - FEB 2013 MED BOARDS; TOPNOTCH MD)

MIDTERM 1 - AUG 2013



TOPNOTCH MEDICAL BOARD PREP PHARMACOLOGY SUPEREXAM Page 81 of 83 For inquiries visit www.topnotchboardprep.com.ph or email us at [email protected]

TOPNOTCH EXAM MIDTERM 1 - AUG 2013

TOPNOTCH MEDICAL BOARD PREP PHARMACOLOGY SUPEREXAM For inquiries visit www.topnotchboardprep.com.ph or email us at [email protected] Item # ANSWER 1 B 2 D 3 C 4 D 5 E 6 B 7 D 8 A 9 BONUS - no correct answer 10 D 11 E 12 A 13 E 14 B 15 B 16 E 17 E 18 E 19 B 20 B 21 D 22 A 23 B 24 B 25 B 26 C 27 E 28 C 29 A 30 D 31 C 32 C 33 C 34 E 35 B 36 B 37 D 38 B 39 D 40 B 41 E 42 E 43 C 44 A 45 B 46 B 47 D 48 B 49 D 50 A 51 B 52 B 53 B 54 A 55 A 56 A 57 B 58 C 59 B 60 B 61 B 62 C 63 A, D 64 B 65 D 66 C 67 A 68 B 69 E 70 C 71 B 72 A 73 D 74 B 75 B 76 E 77 D 78 C 79 B 80 B 81 A 82 A 83 C 84 C 85 B 86 E

87 88 89 90 91 92 93 94 95 96 97 98 99 100 101 102 103 104 105 106 107 108 109 110 111 112 113 114 115 116 117 118 119 120 121 122 123 124 125 126 127 128 129 130 131 132 133 134 135 136 137 138 139 140 141 142 143 144 145 146 147 148 149 150 151 152 153 154 155 156 157 158 159 160 161 162 163 164 165 166 167 168 169 170 171 172 173 174

C B B A D D B D B B E A C E C D A C E D B E D E C A B A C D B A B C D A A E A A E C C A D B D B A A B B D D B C A B A D D A C D E C A D B D B D B D A C B D E D A D E A C D E B

175 176 177 178 179 180 181 182 183 184 185 186 187 188 189 190 191 192 193 194 195 196 197 198 199 200 201 202 203 204 205 206 207 208 209 210 211 212 213 214 215 216 217 218 219 220 221 222 223 224 225 226 227 228 229 230 231 232 233 234 235 236 237 238 239 240 241 242 243 244 245 246 247 248 249 250 251 252 253 254 255 256 257 258 259 260 261 262

B D A C E D B A D C E C C E E B D A A C A A A B D C B C C C E B A B A C A D D B C B D C E D C D D C B C D B C A A D B D E E D B E A B A D A B D C D A D E D C D B C A E C A D B

263 264 265 266 267 268 269 270 271 272 273 274 275 276 277 278 279 280 281 282 283 284 285 286 287 288 289 290 291 292 293 294 295 296 297 298 299 300 301 302 303 304 305 306 307 308 309 310 311 312 313 314 315 316 317 318 319 320 321 322 323 324 325 326 327 328 329 330 331 332 333 334 335 336 337 338 339 340 341 342 343 344 345 346 347 348 349 350

A E A B B C A E D A D E C B E B D A C A D C D E A D D A C B E B D E B A B C B E D C C C A B C D C B D D D A A B A A A A B D D D E B D C C A D B A D D C A A B B B C B A C A C D

TOPNOTCH MEDICAL BOARD PREP PHARMACOLOGY SUPEREXAM Page 82 of 83 For inquiries visit www.topnotchboardprep.com.ph or email us at [email protected]

351 352 353 354 355 356 357 358 359 360 361 362 363 364 365 366 367 368 369 370 371 372 373 374 375 376 377 378 379 380 381 382 383 384 385 386 387 388 389 390 391 392 393 394 395 396 397 398 399 400 401 402 403 404 405 406 407 408 409 410 411 412 413 414 415 416 417 418 419 420 421 422 423 424 425 426 427 428 429 430 431 432 433 434 435 436 437 438

D C B C B A B C A D E E D C E B B A C D D B B E A A B D C A B B C D A B C C B B D B C A A C A B D B A D E D D C A B A C D C A B A A B C C E C A E C D B C B D B C A B D C D B A

TOPNOTCH MEDICAL BOARD PREP PHARMACOLOGY SUPEREXAM For inquiries visit www.topnotchboardprep.com.ph or email us at [email protected] 439 440 441 442 443 444 445 446 447 448 449 450 451 452 453 454 455 456 457 458 459 460 461 462 463 464 465 466 467 468 469 470 471 472 473 474 475 476 477 478 479 480 481 482 483 484 485 486 487 488 489 490 491 492 493 494 495 496 497 498 499 500 501 502 503 504 505 506 507 508 509 510 511 512 513 514 515 516 517 518 519 520 521 522 523 524 525 526 527

B D E C E B E B C B E B D C D E A C B A B C B A C B D A C B E D B E C B E C D A C A A D C B A A D B C D D A D A B C B A D D A C E A E B D B A D E B C B D A B D B E C B D B C D D

528 529 530 531 532 533 534 535 536 537 538 539 540 541 542 543 544 545 546 547 548 549 550 551 552 553 554 555 556 557 558 559 560 561 562 563 564 565 566 567 568 569 570 571 572 573 574 575 576 577 578 579 580 581 582 583 584 585 586 587 588 589 590 591 592 593 594 595 596 597 598 599 600 601 602 603 604 605 606 607 608 609 610 611 612 613 614 615 616

C C C A D A C C B D A D C E E A D C A B E B B E D C D D D D C A A B A C B D D A B B B B B A A A A D C C A D A C C B D C C E B D C A D E C D C B A A A B C C E B A D B A A C A B D

617 618 619 620 621 622 623 624 625 626 627 628 629 630 631 632 633 634 635 636 637 638 639 640 641 642 643 644 645 646 647 648 649 650 651 652 653 654 655 656 657 658 659 660 661 662 663 664 665 666 667 668 669 670 671 672 673 674 675 676 677 678 679 680 681 682 683 684 685 686 687 688 689 690 691 692 693 694 695 696 697 698 699 700

C C D E E A B E A A D D E B E B E B A B C D A E B B A D B E D B A C D A E C C C C D A C C A C B B B C A B D C A A B D B C A A D C B C A C D A B A D B D B B B B C B A E

TOPNOTCH MEDICAL BOARD PREP PHARMACOLOGY SUPEREXAM Page 83 of 83 For inquiries visit www.topnotchboardprep.com.ph or email us at [email protected]

Related Documents


More Documents from "Abdelrahman Mamdouh"